Curbside Consultation in Neuro-Ophthalmology : 49 Clinical Questions, Second Edition [1 ed.] 9781630911515, 9781617116377

Curbside Consultation in Neuro-Ophthalmology has been updated into a Second Edition!The Second Edition contains new ques

202 36 12MB

English Pages 237 Year 2015

Report DMCA / Copyright

DOWNLOAD PDF FILE

Recommend Papers

Curbside Consultation in Neuro-Ophthalmology : 49 Clinical Questions, Second Edition [1 ed.]
 9781630911515, 9781617116377

  • 0 0 0
  • Like this paper and download? You can publish your own PDF file online for free in a few minutes! Sign Up
File loading please wait...
Citation preview

Lee

Second

Second

UPDATED

UPDATED

Curbside Consultation in Neuro-Ophthalmology 49 Clinical Questions

Curbside Consultation in Neuro-Ophthalmology: 49 Clinical Questions has been updated into a Second Edition! Curbside Consultation in Neuro-Ophthalmology: 49 Clinical Questions, Second Edition contains new questions and brief, practical, evidence-based answers to the most frequently asked questions that are posed during a “curbside consultation” between surgical colleagues.  Dr. Andrew G. Lee and associate editors Dr. Paul W. Brazis and Dr. Lanning B. Kline have designed this unique reference in which neuro-ophthalmologists offer expert advice, preferences, and opinions on tough clinical questions commonly associated with neuro-ophthalmology. The unique Q&A format provides quick access to current information related to neuro-ophthalmology with the simplicity of a conversation between two colleagues. Images, diagrams, and references are included to enhance the text and to illustrate common clinical dilemmas. Some of the questions that are answered inside the Second Edition include: • What is the evaluation for papilledema? • What is the work up for third nerve palsy? • What is the treatment for giant cell arteritis? • What is the evaluation for optic disc edema with a macular star? • What is the evaluation for anisocoria? Curbside Consultation in Neuro-Ophthalmology: 49 Clinical Questions, Second Edition  provides information basic enough for residents while also incorporating expert pearls that even high-volume ophthalmologists will appreciate. Residents, fellows, and practicing physicians alike will benefit from the user-friendly, casual format and the expert advice contained within. Contributors Include: Anne Abel • Benin Barahimi • M. Tariq Bhatti • Valérie Biousse • Mark Borchert • Swaraj Bose • Emily M. Bratton • Jodie M. Burton • Tom Carlow • Sophia Chung • Kimberly Cockerham • James Corbett • Wayne Cornblath • Fiona Costello • Roberto A. Cruz • Kathleen B. Digre • Marc Dinkin • James A. Dixon • Eric Eggenberger • Angelina Espino • Julie Falardeau • Steven E. Feldon • Rod Foroozan • Deborah I. Friedman • James A. Garrity • Christopher C. Glisson • Karl Golnik • Aaron Grant • Jennifer K. Hall • Steven R. Hamilton • Andrew R. Harrison • Jonathan C. Horton • Randy Kardon • David I. Kaufman • Aki Kawasaki • Sachin Kedar • Melissa W. Ko • Gregory S. Kosmorsky • Byron L. Lam • Jacqueline Leavitt • Michael S. Lee • Robert L. Lesser • Leah Levi • Grant T. Liu • Amina Malik • Timothy James McCulley • Neil R. Miller • Lina Nagia • Nancy J. Newman • Steve Newman • Victoria S. Pelak • Janet C. Rucker • Alfredo A. Sadun • Robert H. Spector • Madhura A. Tamhankar • Rosa Ana Tang • Matthew J. Thurtell • Robert L. Tomsak • Roger E. Turbin • Michael S. Vaphiades • Nicholas Volpe • Michael Wall • Michelle Y. Wang • Sushma Yalamanchili

®

MEDICAL/Ophthalmology

®

I N C O R P O R A T E D

I N C O R P O R A T E D

SLACK

SLACK

Curbside Consultation in Neuro-Ophthalmology: 49 Clinical Questions, Second Edition

Edition

Curbside Consultation in Neuro-Ophthalmology Edition

49 Clinical Questions

Editor Andrew G. Lee Associate Editors Paul W. Brazis Lanning B. Kline Series Editor David F. Chang

SLACK Incorporated

Second

UPDATED Edition

49 Clinical Questions

Curbside Consultation in Ophthalmology Series Series Editor, David F. Chang, MD

Second

UPDATED Edition

49 Clinical Questions Editor Andrew G. Lee, MD Chair, Department of Ophthalmology, Houston Methodist Hospital, Houston, Texas Professor of Ophthalmology, Neurology and Neurosurgery, Weill Cornell Medical College, New York, New York Adjunct Professor, Ophthalmology, Baylor College of Medicine, Houston, Texas Adjunct Professor, Ophthalmology, University of Texas Medical Branch, Galveston, Texas Adjunct Professor, Ophthalmology, MD Anderson Cancer Center, Houston, Texas Adjunct Professor, Ophthalmology, University of Iowa, Iowa City, Iowa

Associate Editors Paul W. Brazis, MD Professor of Neurology Consultant in Neurology and Ophthalmology Mayo Clinic—Jacksonville Jacksonville, Florida

Lanning B. Kline, MD Professor Department of Ophthalmology University of Alabama School of Medicine Birmingham, Alabama

www.Healio.com/books Copyright © 2015 by SLACK Incorporated All rights reserved. No part of this book may be reproduced, stored in a retrieval system or transmitted in any form or by any means, electronic, mechanical, photocopying, recording or otherwise, without written permission from the publisher, except for brief quotations embodied in critical articles and reviews. The procedures and practices described in this publication should be implemented in a manner consistent with the professional standards set for the circumstances that apply in each specific situation. Every effort has been made to confirm the accuracy of the information presented and to correctly relate generally accepted practices. The authors, editors, and publisher cannot accept responsibility for errors or exclusions or for the outcome of the material presented herein. There is no expressed or implied warranty of this book or information imparted by it. Care has been taken to ensure that drug selection and dosages are in accordance with currently accepted/recommended practice. Off-label uses of drugs may be discussed. Due to continuing research, changes in government policy and regulations, and various effects of drug reactions and interactions, it is recommended that the reader carefully review all materials and literature provided for each drug, especially those that are new or not frequently used. Some drugs or devices in this publication have clearance for use in a restricted research setting by the Food and Drug and Administration or FDA. Each professional should determine the FDA status of any drug or device prior to use in their practice. Although the editors are members of the Ophthalmic Knowledge Assessment Program (OKAP) committee, the Residency Review Committee (RRC) for ophthalmology, and/or examiners for the American Board of Ophthalmology (ABO), this work and its content shall not be marketed or construed as board or OKAP preparatory material and the views expressed herein do not necessarily reflect those of any organization listed above. Any review or mention of specific companies or products is not intended as an endorsement by the author or publisher. SLACK Incorporated uses a review process to evaluate submitted material. Prior to publication, educators or clinicians provide important feedback on the content that we publish. We welcome feedback on this work. Published by:

SLACK Incorporated 6900 Grove Road Thorofare, NJ 08086 USA Telephone: 856-848-1000 Fax: 856-848-6091 www.Healio.com/books

Contact SLACK Incorporated for more information about other books in this field or about the availability of our books from distributors outside the United States. Library of Congress Cataloging-in-Publication Data Curbside consultation in neuro-ophthalmology : 49 clinical questions / editor, Andrew G. Lee ; associate editors, Paul W. Brazis, Lanning B. Kline. -- Second edition. p. ; cm. -- (Curbside consultation in ophthalmology) Includes bibliographical references and index. ISBN 978-1-61711-637-7 (alk. paper) I. Lee, Andrew G., editor. II. Brazis, Paul W., editor. III. Kline, Lanning B., editor. IV. Series: Curbside consultation in ophthalmology series. [DNLM: 1. Optic Nerve Diseases--diagnosis. 2. Optic Nerve Diseases--therapy. 3. Ocular Motility Disorders-diagnosis. 4. Ocular Motility Disorders--therapy. WW 280] RE725 617.7'32--dc23 2014044756 For permission to reprint material in another publication, contact SLACK Incorporated. Authorization to photocopy items for internal, personal, or academic use is granted by SLACK Incorporated provided that the appropriate fee is paid directly to Copyright Clearance Center. Prior to photocopying items, please contact the Copyright Clearance Center at 222 Rosewood Drive, Danvers, MA 01923 USA; phone: 978-750-8400; Web site: www.copyright.com; email: [email protected]

Dedication This book is dedicated to my immigrant parents, Alberto and Rosalind Lee, who believed in the supremacy of education and the hope for a better life for their children and made their way bravely to the United States with nothing but ambition, work ethic, and an unshakable faith in the American dream.

Contents Dedication ...........................................................................................................................................v Acknowledgments ...............................................................................................................................xi About the Editor ...............................................................................................................................xiii About the Associate Editors ...............................................................................................................xiv Contributing Authors .................................................................................................................... ....xv Introduction ..................................................................................................................................... xix Question 1

How Should a Childhood Optic Nerve Glioma Be Worked Up? ......................1 Mark Borchert, MD

Question 2

How Should a Meningioma of the Optic Nerve Sheath Be Managed? .............5 Neil R. Miller, MD

Question 3

When Should You Consider the Diagnosis of Neuromyelitis Optica? .............. 9 Fiona Costello, MD, FRCPC and Jodie M. Burton, MD, MSc, FRCPC

Question 4

How Should I Evaluate and Manage Suspected Optic Neuritis? .................... 13 Eric Eggenberger, DO, MSEpi

Question 5

What Is the Treatment of Optic Neuritis? ....................................................... 17 Melissa W. Ko, MD

Question 6

What Is the Workup and Treatment for Neuroretinitis? ................................. 21 Karl Golnik, MD and Amina Malik, MD

Question 7

How Do You Evaluate and Treat Nonarteritic Anterior Ischemic Optic Neuropathy? ............................................................................................ 25 M. Tariq Bhatti, MD

Question 8

How Do You Differentiate Arteritic From Nonarteritic Anterior Ischemic Optic Neuropathy? ............................................................................................ 29 Rod Foroozan, MD

Question 9

How Should I Treat Giant Cell Arteritis? ....................................................... 33 Jacqueline Leavitt, MD

Question 10

What Is the Evaluation of Traumatic Optic Neuropathy? ............................... 37 Nicholas Volpe, MD and Jennifer K. Hall, MD

Question 11

What Is the Evaluation and Management for Papilledema? ........................... 41 Michael Wall, MD

Question 12

Is There a Difference in the Management of Pseudotumor Cerebri in Pregnancy? ......................................................................................................45 Kathleen B. Digre, MD

Question 13

What Is the Evaluation and Management of Low- and High-Flow Carotid Cavernous Fistulas? ...............................................................................49 Victoria S. Pelak, MD; Emily M. Bratton, MD; and James A. Dixon, MD

Question 14

How Do I Work Up and Manage an Internuclear Ophthalmoplegia?.............55 Christopher C. Glisson, DO, MS and David I. Kaufman, DO

Question 15

What Is the Workup and Treatment of Myasthenia Gravis?........................... 59 Leah Levi, MBBS

viii  Contents Question 16

How Do You Manage Visual Loss in Thyroid Eye Disease? ...........................65 James A. Garrity, MD

Question 17

When Do You Use Radiation or Steroids in Thyroid Eye Disease?.................69 Steven E. Feldon, MD, MBA

Question 18

How Do You Manage Diplopia in Thyroid Eye Disease? ................................73 Kimberly Cockerham, MD, FACS

Question 19

What Are the Evaluations and Treatments for Acquired Nystagmus? ............79 Janet C. Rucker, MD

Question 20

What Is the Evaluation for Anisocoria?.............................................................85 Sophia Chung, MD and Aaron Grant, MD

Question 21

How and When Should I Work Up Horner Syndrome? ..................................89 Sachin Kedar, MBBS, MD and Valérie Biousse, MD

Question 22

What Should I Do With a Dilated Pupil?.........................................................97 Randy Kardon, MD, PhD

Question 23

What Is the Evaluation for Episodic Anisocoria? ...........................................101 Aki Kawasaki, MD, PhD

Question 24

How Do You Manage Toxic and Nutritional Optic Neuropathies? ...............105 Alfredo A. Sadun, MD, PhD and Michelle Y. Wang, MD, PhD

Question 25

What Are Visual Processing Defects and How Can I Recognize Them? .....111 Swaraj Bose, MD

Question 26

How Do I Manage Patients With Headache Syndromes Who Come to Me as an Ophthalmologist?..............................................................................115 Deborah I. Friedman, MD, MPH

Question 27

What Is the Evaluation of Optic Atrophy?......................................................119 Julie Falardeau, MD

Question 28

How Do I Treat a Child With Asymmetric Nystagmus? .............................. 123 Madhura A. Tamhankar, MD and Grant T. Liu, MD

Question 29

What Is Opsoclonus and How Do I Manage It? ........................................... 125 Steve Newman, MD

Question 30

What Is Wernicke Encephalopathy and How Does It Affect the Eye? ........ 127 Lina Nagia, DO and James Corbett, MD

Question 31

How Do I Manage Postoperative Visual Loss After Nonocular Surgery? .....131 Wayne Cornblath, MD

Question 32

How Do I Manage Transient Monocular Visual Loss in a Young, Otherwise Healthy Patient? .................................................................135 Rosa Ana Tang, MD, MPH and Roberto A. Cruz, MD

Question 33

What Is the Evaluation for Transient Monocular Visual Loss in an Older Adult?......................................................................................................139 Byron L. Lam, MD

Question 34

What Is the Evaluation for a Homonymous Hemianopia? .............................143 Jonathan C. Horton, MD, PhD

Question 35

What Is the Evaluation for a Painful Third Nerve Palsy Without a Fixed and Dilated Pupil but With Anisocoria (Partial Pupillary Involvement)? .....149 Michael S. Vaphiades, DO

Contents  ix Question 36

How Do You Manage an Isolated and Presumed Vasculopathic, Pupil-Sparing Third Nerve Palsy? ...................................................................151 Tom Carlow, MD

Question 37

What Is the Appropriate Evaluation of a Fourth Nerve Palsy? ......................155 Robert L. Tomsak, MD, PhD and Matthew J. Thurtell, MBBS, FRACP

Question 38

What Is the Appropriate Evaluation in a Patient Suspected of Having a Sixth Nerve Palsy? ............................................................................................159 Steven R. Hamilton, MD

Question 39

How Do I Evaluate a Patient With Multiple Ocular Motor Cranial Nerve Palsies? ....................................................................................................163 Marc Dinkin, MD and Gregory S. Kosmorsky, DO

Question 40

What Is Blepharospasm? ..................................................................................169 Sushma Yalamanchili, MD and Angelina Espino, MD

Question 41

What Is Hemifacial Spasm? .............................................................................173 Andrew R. Harrison, MD; Benin Barahimi, MD; and Michael S. Lee, MD

Question 42

How Do You Deal With Nonorganic Visual Loss? ........................................177 Robert L. Lesser, MD

Question 43

How Do You Diagnose and Manage Migraine Aura? ....................................183 Robert H. Spector, MD

Question 44

How Do I Recognize Leber Hereditary Optic Neuropathy?..........................187 Nancy J. Newman, MD

Question 45

How Do I Manage an Orbital Apex Syndrome?.............................................191 Roger E. Turbin, MD, FACS

Question 46

How Do I Evaluate and Manage Idiopathic Orbital Inflammatory Syndrome? .................................................................................197 Timothy James McCulley, MD

Question 47

How Do I Manage the Low-Flow Carotid Cavernous Fistula? .....................201 Leah Levi, MBBS

Question 48

How Do I Evaluate and Treat Radiation Optic Neuropathy? ....................... 205 Anne Abel, MD and Michael S. Lee, MD

Question 49

What Should I Do With a Seventh Nerve Palsy? .......................................... 209 Matthew J. Thurtell, MBBS, FRACP

Financial Disclosures ....................................................................................................................... 213

Acknowledgments Dr. Lee would like to acknowledge his father (Alberto C. Lee, MD), mother (Rosalind G. Lee, MD), wife (Hilary A. Beaver, MD), and sister (Amy Lee Wirts, MD), who are all physicians, for their patience, love, support, and input and for helping to “keep it real” for an academic physician communicating with comprehensive and general ophthalmologists who are on the front lines of care. He also thanks and recognizes the valuable input and support from his brother, Richard B. Lee, who has been a source of inspiration, intellectual exercise, and passionate debate about topics in and outside of medicine. He particularly thanks the mentors and role models in his career and in life, Neil R. Miller, MD, Anthony C. Arnold, MD, and Karl C. Golnik, MD, who have made it fun as well as educational. Lastly, Dr. Lee thanks his co-editors (Paul Brazis, MD and Lanning Kline, MD) for helping make this book a success, and he sends a shout out to all of his former medical students, residents, and fellows who are the true inspiration for creating curbside consult content that can help neuro-ophthalmic patients in the real world.

About the Editor Andrew G. Lee, MD is chair of the Department of Ophthalmology at the Jack S. Blanton Eye Institute of the Houston Methodist Hospital in the Texas Medical Center. He is Professor of Ophthalmology, Neurology, and Neurosurgery at Weill Cornell Medical College in New York; Adjunct Professor of Ophthalmology at the University of Iowa in Iowa City and at Baylor College of Medicine in Houston, Texas; and Clinical Professor at the University of Texas Medical Branch in Galveston, the University of Texas MD Anderson Cancer Center in Houston, and the University of Buffalo, SUNY in New York. Dr. Lee has served on the national and international editorial board of 15 journals including Archives of Ophthalmology, American Journal of Ophthalmology, Canadian Journal of Ophthalmology, Japanese Journal of Ophthalmology, Journal of Neuro-Ophthalmology, Survey of Ophthalmology, and Eye and is the Editor-in-Chief of the Journal of Clinical and Academic Ophthalmology. He has published over 380 peer-reviewed publications, 40 book chapters, and 9 full textbooks in ophthalmology. He has been the invited speaker at over 350 national and international eye meetings and has given 12 named lectureships. Dr. Lee has a special interest in graduate medical education and has received the resident teaching award 6 times at 5 different academic institutions.

About the Associate Editors Paul W. Brazis, MD received his undergraduate degree from the University of Notre Dame in Indiana and his medical degree from Loyola University-Stritch School of Medicine in Maywood, Illinois. After his medical internship at Presbyterian-St. Lukes Hospital in Chicago, he returned to Loyola University-Stritch School of Medicine for a neurology residency. His fellowship in neuro-ophthalmology was at Johns Hopkins Medical Center-Wilmer Eye Center in Baltimore, Maryland. Dr. Brazis is presently a Professor of Neurology in the Mayo Clinic School of Medicine and a consultant in neurology and neuro-ophthalmology for the Mayo Clinic at Jacksonville, Florida. He was voted the Outstanding Faculty Member in 1999, 2001, and 2002 for his lectures with the Mayo Clinic School of Continuing Medical Education and received the Distinguished Mayo Clinician Award for the Jacksonville Mayo Clinic in 2001. He has published over 90 peerreviewed articles, 36 book chapters, and 14 textbooks in neurology and neuro-ophthalmology. Lanning B. Kline, MD was born in Edmonton, Alberta, Canada. He received his Bachelor of Arts degree from the University of Alberta and graduated from Duke University School of Medicine in Durham, North Carolina. He served an internship at Duke University Department of Medicine and a residency in the department of Ophthalmology, McGill University, Montreal, Quebec, Canada. Dr. Kline completed fellowships in neuro-ophthalmology at the Montreal Neurological Institute and at the Bascom Palmer Eye Institute, Miami, Florida. Dr. Kline has been a faculty member all of his career in the Department of Ophthalmology, University of Alabama School of Medicine in Birmingham. In 1998, he became Professor and Chairman of the department, and in 2000 was appointed to the EyeSight Foundation of Alabama Endowed Chair in Ophthalmology. He served in that capacity until 2011. Dr. Kline is certified by the American Board of Ophthalmology, is a member of the North American Neuro-Ophthalmology Society, and is a Fellow of the American College of Surgeons. Currently, he serves as Editor-in-Chief of the Journal of Neuro-Ophthalmology.

Contributing Authors Anne Abel, MD (Question 48) Department of Ophthalmology Hennepin County Medical Center Minneapolis, Minnesota

Tom Carlow, MD (Question 36) Emeritus Professor, Neurology, Ophthalmology, Radiology and Neurosurgery University of New Mexico School of Medicine Eye Associates of New Mexico Albuquerque, New Mexico

Benin Barahimi, MD (Question 41) Assistant Professor Department of Ophthalmology Vanderbilt University Nashville, Tennessee M. Tariq Bhatti, MD (Question 7) Professor Departments of Ophthalmology and Neurology Duke Eye Center and Duke University Medical Center Durham, North Carolina Valérie Biousse, MD (Question 21) Professor of Ophthalmology and Neurology Cyrus H. Stoner Professor of Ophthalmology Emory University School of Medicine Emory Eye Center Atlanta, Georgia Mark Borchert, MD (Question 1) Associate Professor, Ophthalmology Neurology Children's Hospital Los Angeles USC Keck School of Medicine Los Angeles, California

and

Kimberly Cockerham, MD, FACS (Question 18) Adjunct Associate Clinical Professor Stanford Department of Ophthalmology Palo Alto, California James Corbett, MD (Question 30) Department of Neurology University of Mississippi Medical Center Jackson, Mississippi Wayne Cornblath, MD (Question 31) Professor, Departments of Ophthalmology & Visual Sciences and Neurology University of Michigan School of Medicine Ann Arbor, Michigan Fiona Costello, MD, FRCPC (Question 3) Associate Professor, Departments of Clinical Neurosciences and Surgery University of Calgary Calgary, Alberta, Canada

Swaraj Bose, MD (Question 25) Attending Physician Neuro-Ophthalmology and Orbital Surgery Cedars Sinai Medical Center Los Angeles, California Emily M. Bratton, MD (Question 13) University of Colorado Orbital and Oculofacial Plastic Reconstructive Surgery Denver, Colorado

Sophia Chung, MD (Question 20) Professor of Ophthalmology, Neurology & Psychiatry, and Neurosurgery Saint Louis University School of Medicine Saint Louis, Missouri

Roberto A. Cruz, MD (Question 32) Neuro-Ophthalmology of Texas, PLLC Houston, Texas

and

Jodie M. Burton, MD, MSc, FRCPC (Question 3) Department of Clinical Neurosciences and Department of Community Health Sciences University of Calgary Calgary, Alberta, Canada

Kathleen B. Digre, MD (Question 12) Professor of Neurology, Ophthalmology Adjunct Professor of Obstetrics and Gynecology Moran Eye Center Departments of Neurology and Ophthalmology University of Utah Salt Lake City, Utah

xvi  Contributing Authors Marc Dinkin, MD (Question 39) Director of Neuro-Ophthalmology Assistant Professor of Ophthalmology, Neurology and Neurosurgery Weill Cornell Medical College New York Presbyterian Hospital New York, New York James A. Dixon, MD (Question 13) Kaiser Permanente Denver, Colorado Eric Eggenberger, DO, MSEpi (Question 4) Department of Neurology & Ophthalmology Michigan State University East Lansing, Michigan Angelina Espino, MD (Question 40) Centro Medico Zambrano Hellion San Pedro Garza García, Nuevo León, Mexico Julie Falardeau, MD (Question 27) Associate Professor of Ophthalmology Casey Eye Institute Oregon Health and Science University Portland, Oregon Steven E. Feldon, MD, MBA (Question 17) Chair, Department of Ophthalmology Director, David & Ilene Flaum Eye Institute Professor of Ophthalmology, Neurology, Neurosurgery, and Visual Science University of Rochester School of Medicine & Dentistry Rochester, New York

Christopher C. Glisson, DO, MS (Question 14) Medical Director, Neuro-Ophthalmology Program Mercy Health Hauenstein Neurosciences Grand Rapids, Michigan Assistant Professor, Department of Neurology and Ophthalmology Michigan State University East Lansing, Michigan Karl Golnik, MD (Question 6) University of Cincinnati The Cincinnati Eye Institute Cincinnati, Ohio Aaron Grant, MD (Question 20) Pediatric and Neuro-Ophthalmologist Wilford Hall Ambulatory Surgical Center Lackland Air Force Base, Texas Jennifer K. Hall, MD (Question 10) Swarthmore, Pennsylvania Steven R. Hamilton, MD (Question 38) Neuro-Ophthalmic Consultants Northwest Swedish Neuroscience Institute Seattle, Washington Andrew R. Harrison, MD (Question 41) Director, Oculoplastic and Orbital Surgery Associate Professor University of Minnesota Departments of Ophthalmology and Otolaryngology Minneapolis, Minnesota

Rod Foroozan, MD (Question 8) Associate Professor of Ophthalmology Baylor College of Medicine Houston, Texas

Jonathan C. Horton, MD, PhD (Question 34) William F. Hoyt Professor of Ophthalmology Beckman Vision Center, UCSF San Francisco, California

Deborah I. Friedman, MD, MPH (Question 26) Professor, Neurology & Neurotherapeutics and Ophthalmology University of Texas Southwestern Medical Center Dallas, Texas

Randy Kardon, MD, PhD (Question 22) Professor and Director of Neuro-Ophthalmology Director of Iowa City VA Center for Prevention and Treatment of Visual Loss Pomerantz Family Chair in Ophthalmology Department of Ophthalmology and Visual Sciences Department of Veterans Affairs Hospital University of Iowa Hospital and Clinics Iowa City, Iowa

James A. Garrity, MD (Question 16) Whitney and Betty MacMillan Professor of Ophthalmology Mayo Clinic Rochester, Minnesota

Contributing Authors  xvii David I. Kaufman, DO (Question 14) Professor and Chair Department of Neurology and Ophthalmology Michigan State University East Lansing, Michigan Aki Kawasaki, MD, PhD (Question 23) Department of Ophthalmology University of Lausanne Hôpital Ophtalmique Jules Gonin Lausanne, Switzerland Sachin Kedar, MBBS, MD (Question 21) Associate Professor in Neurology Ophthalmology University of Nebraska Medical Center Stanley M. Truhlsen Eye Institute Omaha, Nebraska

and

Melissa W. Ko, MD (Question 5) Associate Professor, Neurology and Ophthalmology University Health Care Center Neurology SUNY Upstate Medical University Syracuse, New York Gregory S. Kosmorsky, DO (Question 39) Head, Section of Neuro-Ophthalmology Cleveland Clinic Cole Eye Institute Cleveland, Ohio Byron L. Lam, MD (Question 33) Professor Bascom Palmer Eye Institute University of Miami School of Medicine Miami, Florida Jacqueline Leavitt, MD (Question 9) Associate Professor Department of Ophthalmology Mayo Clinic Rochester Rochester, Minnesota Michael S. Lee, MD (Questions 41, 48) Director, Neuro-Ophthalmology Professor University of Minnesota Departments of Ophthalmology, Neurology and Neurosurgery Minneapolis, Minnesota

Robert L. Lesser, MD (Question 42) Clinical Professor of Ophthalmology & Visual Science and Neurology Yale University School of Medicine New Haven, Connecticut Clinical Professor of Neurology and Surgery University of Connecticut School of Medicine Farmington, Connecticut Leah Levi, MBBS (Questions 15, 47) Director of Neuro-Ophthalmology Scripps Clinic Division of Ophthalmology La Jolla, California Clinical Professor Emerita University of California San Diego School of Medicine San Diego, California Grant T. Liu, MD (Question 28) Department of Neurology and Ophthalmology Hospital of University of Pennsylvania Philadelphia, Pennsylvania Amina Malik, MD (Question 6) Department of Ophthalmology Houston Methodist Hospital Houston, Texas Timothy James McCulley, MD (Question 46) Vice Chair for Clinical Strategic Planning Director, Division of Neuro-Ophthalmology Director, ASOPRS Oculoplastic Surgery Fellowship The Wilmer Eye Institute Johns Hopkins School of Medicine Baltimore, Maryland Neil R. Miller, MD (Question 2) Johns Hopkins University School of Medicine Baltimore, Maryland Lina Nagia, DO (Question 30) Department of Ophthalmology University of Alabama Birmingham Birmingham, Alabama Nancy J. Newman, MD (Question 44) LeoDelle Jolley Professor of Ophthalmology Professor of Ophthalmology and Neurology Instructor in Neurological Surgery Emory University School of Medicine Atlanta, Georgia

xviii  Contributing Authors Steve Newman, MD (Question 29) Professor of Ophthalmology University of Virginia Health Systems Charlottesville, Virginia Walter Reed National Medical Center Bethesda, Maryland

Robert L. Tomsak, MD, PhD (Question 37) Professor of Ophthalmology and Neurology Wayne State University School of Medicine Specialist in Neuro-Ophthalmology Kresge Eye Institute Detroit, Michigan

Victoria S. Pelak, MD (Question 13) Associate Professor, Neurology and Ophthalmology University of Colorado School of Medicine Anschutz Medical Campus Department of Neurology Aurora, Colorado

Roger E. Turbin, MD, FACS (Question 45) Associate Professor of Ophthalmology Neuro-Ophthalmology, Orbit and Oculoplastic Surgery Director of Ocular Trauma Rutgers, New Jersey Medical School Newark, New Jersey

Janet C. Rucker, MD (Question 19) Bernard A. and Charlotte Marden Professorship of Neurology Division and Fellowship Director, NeuroOphthalmology Associate Professor, Department of Neurology NYU Langone Medical Center New York, New York Alfredo A. Sadun, MD, PhD (Question 24) F. Thornton Chair, Doheny Eye Institute Vice-Chair of Ophthalmology, David Geffen School of Medicine at UCLA Los Angeles, California Robert H. Spector, MD (Question 43) Retired Neuro-Ophthalmologist Atlanta, Georgia Madhura A. Tamhankar, MD (Question 28) Department of Ophthalmology and Neurology Scheie Eye Institute University of Pennsylvania Philadelphia, Pennsylvania Rosa Ana Tang, MD, MPH (Question 32) Director, Neuro-Ophthalmology of Texas, PLLC Research Professor Director, Multiple Sclerosis Eye CARE Center Clinical Assistant Professor, Department of Physical Medicine and Rehabilitation and Neurosurgery University of Houston Houston, Texas Matthew J. Thurtell, MBBS, FRACP (Questions 37, 49) Department of Ophthalmology & Visual Sciences and Department of Neurology University of Iowa Department of Veterans Affairs Medical Center Iowa City, Iowa

Michael S. Vaphiades, DO (Question 35) Callahan Eye Hospital Birmingham, Alabama Nicholas Volpe, MD (Question 10) Tarry Professor and Chairman Department of Ophthalmology Northwestern Medicine Northwestern Feinberg School of Medicine Chicago, Illinois Michael Wall, MD (Question 11) Department of Neurology Carver College of Medicine University of Iowa Iowa City, Iowa Professor of Neurology and Ophthalmology Iowa City Veterans Affairs Medical Center Iowa City, Iowa Michelle Y. Wang, MD, PhD (Question 24) Department of Ophthalmology Kaiser Permanente Oakland, California Sushma Yalamanchili, MD (Question 40) Methodist Eye Associates Houston, Texas Assistant Professor of Clinical Ophthalmology Weill-Cornell Medical College New York, New York Adjunct Professor of Clinical Ophthalmology Department of Ophthalmology and Visual Science The University of Texas Medical Branch Galveston, Texas

Introduction Curbside Consultation in Neuro-Ophthalmology, Second Edition is meant to be enjoyed as a quick reference text for the clinician on the go. Three neuro-ophthalmologists, Andrew G. Lee MD, Lanning B. Kline MD, and Paul W. Brazis MD serve as your guide through various common clinical scenarios. Curbside consults are a ubiquitous and important part of day-to-day medical communications between general ophthalmologists and subspecialty neuro-ophthalmologists. We chose this format to emphasize the collegial and informal but informative nature of this real world interaction. This book offers brief, concise, and practical answers to those questions that are often left unanswered by traditional texts and references in neuro-ophthalmology. The presenters use a case-based format to pose and answer questions of neuro-ophthalmic interest to the general ophthalmologist. We hope that you enjoy reading this book as much as we enjoyed writing and editing it.

1

QUESTION

HOW SHOULD A CHILDHOOD OPTIC NERVE GLIOMA BE WORKED UP? Mark Borchert, MD An 8-year-old boy presents with vision of 20/40 right eye (OD) and 20/20 left eye (OS). He has a right relative afferent pupillary defect and a pale optic nerve OD. The left fundus is normal. Magnetic resonance imaging (MRI) findings are consistent with a right optic nerve glioma. How should this patient be evaluated? Let us be frank. There are no treatments with demonstrated effectiveness in curing optic gliomas or preventing the vision loss that may be associated with them. Thus, the main management decision is determining if and when the various unproven and, in some cases, experimental therapies should be started. This decision depends on understanding the prognosis for untreated optic nerve gliomas, which, in turn, depends on the age of the patient and whether he or she is afflicted with neurofibromatosis type 1 (NF-1). In general, patients with NF-1 have a better visual prognosis, and their optic nerve gliomas rarely cause progressive vision loss beyond the age of 12 years. An assessment of afflicted children for café-au-lait spots, Lisch nodules, and family history of possible NF-1 is essential. Genotyping is rarely necessary but is available and, in questionable cases, should be discussed with a geneticist. You should be more reluctant to pull the treatment trigger on a patient with NF-1 because the untreated prognosis is relatively good and there is a theoretical risk of inducing malignant tumors in patients who have a growth control gene mutation. This is of particular concern if your oncologist is considering radiation therapy. Regardless of whether the afflicted patient has NF-1, I believe that no treatment should be started without documented progression of vision loss or hypothalamic dysfunction. The fact that the patient presents with diminished vision is not sufficient. The diminished vision may have been stable for years, even for life. It is also not sufficient for a single decrement in visual acuity from one exam to the next to be considered progressive vision loss. Many patients have fluctuating vision or full recovery of vision regardless of whether the tumor is changing in size on the MRI scan. Consequently, there should be some documentation of progressive vision loss over at least 2 to 3 examinations and this loss should be approaching the threshold for a change in functional significance (eg, losing reading ability or independent mobility).

1

Lee AG, ed. Curbside Consultation in Neuro-Ophthalmology: 49 Clinical Questions, Second Edition (pp 1-3). © 2015 SLACK Incorporated.

2  Question 1 Since your particular patient is 8 years old and literate, you have many monitoring tools at your disposal to help in your decision as to when to refer for chemotherapy. Besides measuring best corrected visual acuity, you can monitor visual fields. This is best done with Goldmann perimetry because it is more reliable than automated perimetry at this age and the examiner can focus on areas of known field loss before the patient’s attention is lost. You should also consider monitoring color vision. I would not base treatment decisions on color vision, but an isolated drop in color vision would cause me to increase the examination frequency. Finally, baseline optic disc photos and/or optical coherence tomography should be obtained and repeated for confirmation if there is suspicion of worsening optic atrophy. How often should you follow this patient? In general, the frequency of visits depends on the stability of the examination and the age of the patient. The longer you follow a patient with a stable ophthalmic examination, the less frequent need be the visits. I will usually see an 8-year-old patient 2 to 3 months following the first examination, then semiannually for 2 to 3 years, then annually. I will follow children who present at less than age 3 years more frequently. The role of repeat neuroimaging is controversial. I do not find it useful in making treatment decisions unless I suspect nonophthalmic consequences of tumor enlargement, such as hypopituitarism or impending hydrocephalus. Nonetheless, parents seem to find comfort in documenting neuroradiographic stability or even shrinkage. Spontaneous regression of optic gliomas with or without improvement in vision has also been documented on serial MRI scans. As I previously mentioned, treatment options are limited. Most neuro-oncologists will treat with some combination of chemotherapy that includes vincristine, carboplatin, or temozolomide. The side effects of these medications are similar to, but generally less severe than, those used to treat most childhood malignancies. These agents are believed to arrest progression and, in some cases, cause tumor regression based on MRI. However, the radiographic response to chemotherapy does not correlate with vision outcomes, and no controlled studies using vision outcomes have been done. Surgical excision of unilateral optic gliomas has been advocated in some cases to prevent progression to chiasmal involvement or to relieve unsightly exophthalmos. Obviously, this treatment causes blindness, so it should be reserved for patients with severely impaired vision. The effectiveness of surgical excision is controversial. There is no evidence that optic gliomas actually spread to uninvolved parts of the visual pathways. There have been numerous documented cases of histological involvement of the chiasm despite the fact that the tumor appeared confined to the orbit on MRI scan. Radiation therapy results in tumor shrinkage but can also cause worsening vision loss. It should best be avoided, if possible, in very young children because of the concomitant risk of cognitive impairment and hypothalamic injury. In anecdotal reports, progressive vision or visual field loss following chemotherapy and proton beam radiation for optic gliomas has been rescued with bevacizumab. This agent has been well tolerated in phase I studies in children with solid tumors, but its effectiveness in optic gliomas has yet to be systematically studied. Fortunately, progressive vision loss and nonophthalmic complications are the exception rather than the rule in most cases of optic glioma. In my experience, parents are relieved to hear this and, once the facts about optic gliomas and their treatment are calmly explained, can accept expectant observation as a management option over immediate treatment.

How Should a Childhood Optic Nerve Glioma Be Worked Up?  3

Summary ●

Optic nerve glioma is typically a tumor of childhood.



Some patients have NF-1.



No treatment has been proven to be effective.



Observation for clinical or radiographic stability or progression is the first line of management.





Chemotherapy and radiation therapy have been used in patients with progressive disease, but their usefulness is unproven. Surgical excision of an optic nerve glioma has a limited role for patients with blind eyes and cosmetically unacceptable proptosis.

Bibliography Avery RA, Hwang EI, Jakacki RI, Packer RJ. Marked recovery of vision in children with optic pathway gliomas treated with bevacizumab. JAMA Ophthalmol. 2014;132(1):111-114. Borit A, Richardson EP Jr. The biological and clinical behavior of pilocytic astrocytomas of the optic pathways. Brain. 1982;105:161-187 Fisher MJ, Loguidice M, Gutmann DH, et al. Visual outcomes in children with neurofibromatosis type 1–associated optic pathway glioma following chemotherapy: a multicenter retrospective analysis. Neuro Oncol. 2012;14:790–797. Glade Bender JL, Adamson PC, Reid JM, et al. Phase I trial and pharmacokinetic study of bevacizumab in pediatric patients with refractory solid tumors: a Children's Oncology Group Study. J Clin Oncol. 2008;26(3):399-405. Hwang JM, Cheon JE, Wang KC. Visual prognosis of optic glioma. Childs Nerv Syst. 2008; 24: 693-698. Parsa CF, Hoyt CS, Lesser RL, et al. Spontaneous regression of optic gliomas: thirteen cases documented by serial neuroimaging. Arch Ophthalmol. 2001;119:516–529.

2

QUESTION

HOW SHOULD A MENINGIOMA OF THE OPTIC NERVE SHEATH BE MANAGED? Neil R. Miller, MD A 50-year-old otherwise healthy woman has had progressive visual loss for 2 years in her right eye. Her MRI scan showed a meningioma of the right optic nerve sheath. Her vision is 20/40 OD; there is a relative afferent pupillary defect and an inferior visual field defect OD. The left eye is normal. The right optic nerve is pale and there is a retinochoroidal venous collateral on the optic nerve head. What should be done now? Based on this patient’s history, clinical findings, and neuroimaging, she almost certainly has an optic nerve sheath meningioma (ONSM). Although so-called retinochoroidal venous collaterals are most often seen after central retinal vein occlusion, the triad of decreased vision, a pale optic disc, and one or more retinochoroidal venous collaterals almost certainly indicates an ONSM. These lesions have three main appearances on imaging: tubular, fusiform, or globular. In all three types, CT scanning typically shows enlargement of the optic nerve with an increased density peripherally and decreased density centrally (the “tram-track” sign) (Figure 2-1). These changes are particularly well seen after intravenous injection of iodinated contrast material. In addition, some ONSMs contain calcifications surrounding the nerve, which may be masked by contrast enhancement and thus are best identified on precontrast soft tissue and bone-windowed images. MRI generally provides much better detail of ONSMs than CT. In particular, the soft tissue component of the tumor is readily visible, especially when T1-weighted images are viewed after intravenous injection of a paramagnetic contrast agent and fat saturation techniques are used. The appearance of the optic nerve on coronal MRI after gadolinium is most often that of a hypo- to isointense signal (the nerve) surrounded by an enhancing tubular, fusiform, or globular peripheral ring of tissue (the tumor) (Figure 2-2). In addition, on careful examination, rather than having a perfectly smooth outline, most ONSMs have very fine extensions into the orbit that are obvious on coronal views (Figures 2-2B, 2-2D, and 2-3). MRI also provides sufficient tissue detail that can serve to determine not only how close the tumor is to the globe, but also the degree of canalicular and intracranial extension if any. Ultrasound of the orbit can also be helpful in the diagnosis of an ONSM. Echographic evaluation of an ONSM characteristically shows an enlargement of the nerve’s diameter with predominantly medium-high reflectivity and an irregular acoustic structure.

5

Lee AG, ed. Curbside Consultation in Neuro-Ophthalmology: 49 Clinical Questions, Second Edition (pp 5-8). © 2015 SLACK Incorporated.

6  Question 2 Figure 2-1. CT scan, axial view, showing the appearance of left optic nerve sheath meningioma. Note thickening of nerve with the “tram-track” sign.

Figure 2-2. MRI: T1weighted fat-suppressed contrast-enhanced axial (A, C) and coronal (B, D) views of two types of optic nerve sheath meningioma. (A, B) Tubular type. (C, D) Fusiform type. Note that in all images there is a hypo- to isointense, nonenhancing optic nerve surrounded by enhancing tumor tissue. Also note the fine extensions of tumor into the orbit, best seen on the coronal views.

Figure 2-3. MRI: T1-weighted fat-suppressed contrast-enhanced coronal view of a left optic nerve sheath meningioma. Note the enhancing tumor tissue surrounding the hypo- to isointense nonenhancing optic nerve. The tumor has irregular borders with fine extensions into the orbit.

In addition, shadowing from internal calcification might be seen. In many cases, a 30-degree test will reveal solid thickening of the nerve, whereas in others the tumor is located more posteriorly and the test will indicate anterior enlargement of the nerve due to the trapping of cerebrospinal fluid by the posteriorly located tumor. These studies obviate the need for tissue biopsy in most

How Should a Meningioma of the Optic Nerve Sheath Be Managed?  7

Figure 2-4. MRI: T1-weighted contrast-enhanced axial views of two ONSMs. (A) Fat-suppressed image shows tumor that extends from the globe to the orbital apex. Note flattening of the globe by the tumor. Radiation therapy of this tumor has a higher likelihood of being complicated by subsequent radiation retinopathy. (B) A non-fat-suppressed image in another ONSM reveals that the tumor is located in the mid-to-posterior orbit. Radiation of this tumor is unlikely to produce radiation retinopathy.

cases, making an early diagnosis possible without potentially damaging the optic nerve during surgery. The retinochoroidal venous collaterals present in your patient eliminate other entities that could cause her clinical presentation; if they were not present, however, you might consider such entities as metastatic infiltration of the optic nerve and optic nerve sheath, lymphoma, and sarcoid in the differential diagnosis. The first things to remember in managing a patient with a presumed ONSM are that the tumor is not lethal, will not cause a catastrophic neurologic deficit, and is highly unlikely to spread across the planum sphenoidale and cause loss of vision in the opposite eye. Thus, your management should be aimed at restoring or at least preserving visual function in the affected eye. To date, trials of medical therapy for ONSMs, such as those using estrogen or progesterone antagonists, have not been successful, and attempts to excise these tumors while keeping the optic nerve itself intact are usually unsuccessful, with most patients becoming blind in the eye following surgery. Indeed, the only treatment currently capable of preserving and occasionally improving vision in patients with ONSMs in both the short and long term (ie, > 10 years) is radiation therapy. Several types of radiation therapy have been utilized, including conventional fractionated radiation therapy, stereotactic radiosurgery, hypofractionated stereotactic radiosurgery, and stereotactic fractionated radiation therapy (SFR). Unfortunately, all radiation therapy has significant risks. The major concern with radiotherapy for ONSMs is late toxicity. Radiation can damage not only the optic nerve itself, but also the adjacent tissues, including the retina, pituitary gland, and whitematter tracts of the brain. Retinal injury may occur, particularly if the tumor extends to the globe (Figure 2-4) and a total dose of more than 50 Gy is used, but coexistence of diabetes mellitus may lower the threshold for retinal or optic nerve damage to 45 Gy. The threshold for radiation damage to the optic nerve has been estimated to be 8 to 10 Gy for a single dose. Because lower doses of radiation are thought to have a more uncertain effect on benign tumors such as ONSMs and a large single dose of radiation is associated with a high risk of tissue damage, single-dose stereotactic radiosurgery is not widely used to treat ONSMs. Although hypofractionated stereotactic radiosurgery (5 sessions rather than just one, with the total doses for the five sessions equaling the single dose for standard stereotactic radiosurgery), there are no long-term data. SFR, on the other hand, appears to offer the potential for delivering a sufficient amount of radiation to an ONSM in a more focused manner than conventional fractionated radiation therapy, thus minimizing (but not eliminating) the complications from exposure of the surrounding tissue to high doses of radiation. The first step in patient management is for you to determine whether she believes that her visual loss is sufficient for her to accept the potential risks of radiation. This will be related, at least in part, to the extent of the lesion. If your patient wishes to undergo treatment and accepts

8  Question 2 its risks, then SFR is the treatment of choice. SFR requires complex planning, which is facilitated by sophisticated software and three-dimensional imaging. Fortunately the newer linear accelerator units—such as the Cyberknife (Accuray), Novalis Tx (Novalis Radiosurgery), and Truebeam STx (Varian Medical Systems)—use tracking systems that eliminate the need for rigid immobilization of the patient during treatment. Unlike conventional radiation therapy, these systems deliver radiation in non-coplanar fields that take into account the characteristics of the surrounding tissue. Every beam is size- and shape-adjusted by different devices, with microleaf collimators being the most advanced way of achieving a high degree of conformity to the tumor, thus minimizing irradiation of the surrounding tissue. Nevertheless, the more extensive the lesion, the larger the field that will have to be covered by the radiation and the greater the risk of damage to the globe. I believe it likely that your patient may be a candidate for SFR; please let me know how she does.

Summary ●



Primary ONSMs typically present with slowly progressive unilateral visual loss associated with signs of an anterior optic neuropathy or retrobulbar optic neuropathy. It is fine to observe patients with presumed ONSMs as long as the patients are monitored at regular intervals both clinically and with neuroimaging.



There is no currently effective medical therapy for ONSMs.



In general, any attempt at surgical removal of an ONSM will cause blindness in the eye.



Stereotactic or 3-dimensional conformal fractionated radiotherapy is the current standard of care for most patients with ONSMs who require treatment.

Bibliography Andrews DW, Faroozan R, Yang BP, et al. Fractionated stereotactic radiotherapy for the treatment of optic nerve sheath meningiomas: preliminary observations of 33 optic nerves in 30 patients with historical comparison to observation with or without prior surgery. Neurosurgery. 2002;51:890-902. Lesser RL, Knisely JP, Wang SL, Yu JB, Kupersmith MJ. Long-term response to fractionated radiotherapy of presumed optic nerve sheath meningioma. Br J Ophthalmol. 2010;94:559-563. Marchetti M, Bianchi S, Milanesi I, et al. Multisession radiosurgery for optic nerve sheath meningiomas—an effective option: preliminary results of a single-center experience. Neurosurgery. 2011;69:1116-1123. Metellus P, Kapoor S, Kharkar S, et al. Fractionated conformal radiotherapy for management of optic nerve sheath meningiomas: long-term outcomes of tumor control and visual function at a single institution. Int J Radiat Oncol Biol Phys. 2011;80:185-192. Subramanian PS, Bressler NM, Miller NR. Radiation retinopathy after fractionated stereotactic radiotherapy for optic nerve sheath meningioma. Ophthalmology. 2004;111:565-567.

3

QUESTION

WHEN SHOULD YOU CONSIDER THE DIAGNOSIS OF NEUROMYELITIS OPTICA? Fiona Costello, MD, FRCPC and Jodie M. Burton, MD, MSc, FRCPC

A colleague seeks your opinion regarding the case of a 42-year-old woman who presented with painless vision loss in both eyes. Her symptoms began a month earlier and she has demonstrated no visual recovery (current best-corrected Snellen acuity measured 20/200 in both eyes). She had a prior history of transverse myelitis 1 year ago and was left with lower extremity paralysis. Cranial MRI studies have been normal.

What Are the Typical Features of Optic Neuritis Associated With Multiple Sclerosis? Most cases of optic neuritis (ON) referred to an ophthalmology clinic are sporadic events or represent the first clinical manifestation of multiple sclerosis (MS). The Optic Neuritis Treatment Trial (ONTT) showed that typical ON patients are Caucasian (85%), are women (77%), and have symptom onset at 32 years. Among adults, the majority of ON cases are unilateral, but occasionally bilateral simultaneous vision loss is observed. Typical ON patients report subacute-onset vision loss that worsens over hours to days, and 90% experience pain, frequently provoked by eye movements, within 1 to 2 weeks of symptom onset. There are several localizing features on initial examination that can help secure the diagnosis. The severity of vision loss in the eye with ON may range from mild (Snellen visual acuity equivalent of 20/20) to no light perception. In patients with unilateral optic nerve involvement, a relative afferent pupillary defect is observed in the affected eye. Cecocentral, altitudinal, and arcuate patterns of vision field loss are frequent, and decreased color vision is also common. In cases of retrobulbar ON, the fundus examination is initially normal, whereas patients with anterior ON manifest mild to moderate optic disc swelling acutely. The ONTT demonstrated that the presence of severe optic disc edema, vitreous cells, and hemorrhage are uncommon findings in typical ON. High-dose intravenous methylprednisolone

9

Lee AG, ed. Curbside Consultation in Neuro-Ophthalmology: 49 Clinical Questions, Second Edition (pp 9-11). © 2015 SLACK Incorporated.

10  Question 3 initiated within 2 weeks of symptom onset has been shown to hasten the clinical recovery but not to change the ultimate visual outcome in patients with ON. Initially, visual recovery from typical ON occurs within a matter of weeks; further improvement may be seen up to 1 year after the acute episode. In the post-acute phase, most patients with ON develop optic disc pallor as a “footprint” of the original injury. Regardless of whether patients receive high-dose corticosteroids, the prognosis for recovery after typical ON is generally good. The mean visual acuity 1 year after entry into the ONTT was approximately 20/20 (Snellen equivalent), with less than 10% having a visual acuity worse than 20/40.

What Is Neuromyelitis Optica? Occasionally, ON occurs as the initial manifestation of neuromyelitis optica (NMO), an inflammatory process of the optic nerves and spinal cord typically characterized by marked disability, morbidity, and early mortality. Relative to their multiple sclerosis counterparts, patients with NMO are typically older at disease onset (median age, 39 years), show a strong female preponderance, and have a relatively poor prognosis for visual recovery. In NMO, stepwise neurological impairment arises secondary to relapses as opposed to interval progression, with the latter being much more common in MS. Thus early diagnosis is paramount. For many patients, immunosuppressive therapy is required to obtain some degree of control over the disease. Within 5 years, approximately half of patients with NMO are either blind in one or both eyes or require a walking aid, in sharp contrast to the relatively milder impact of MS on affected individuals with the same disease duration.

How Does the Optic Neuritis Associated With Neuromyelitis Optica Differ From Typical Optic Neuritis? While many ON mimics will declare themselves through the detection of atypical signs and symptoms, ON associated with NMO can easily be overlooked if there is not a high degree of suspicion because these patients often present with clinical features that, for all intents and purposes, mimic typical ON except that visual outcomes tend to be poor. In a retrospective study of 175 NMO patients, Jarius et al reported that among those with both ON and myelitis, isolated ON was the presenting event in 58% of cases. The disease began with isolated myelitis in 25% of patients, with simultaneous myelitis and ON in 13% and with brainstem encephalitis without concomitant myelitis or ON in 4%. Furthermore, this study showed that there was a marked delay to diagnosis in patients with NMO, which ranged from 16 months from onset if the disease started with myelitis to 55 months if ON was the initial clinical event. Unfortunately, a high proportion of NMO patients (43%) in this study were incorrectly diagnosed with MS initially, and over half of the patients were treated inappropriately as a result. Specifically, these 53% were treated at least once with interferon-beta, a drug that is safe and effective in MS but considered to be harmful in NMO. Patients with poor visual recovery 1 month after symptom onset, a history of recurrent ON, bilateral ON, or symptoms of transverse myelitis should be investigated for NMO.

When Should You Consider the Diagnosis of Neuromyelitis Optica?  11

What Steps Can Be Taken to Diagnose Neuromyelitis Optica? Previously, the diagnosis of NMO was made predominantly on clinical grounds and with evidence of extensive longitudinal lesions of three or more vertebral body segments visualized with spinal MRI. Recently, an autoantibody (anti-NMO IgG) has been identified that targets aquaporin-4, a water channel protein. The anti-NMO IgG antibody facilitates the diagnosis of NMO with a specificity of 94% to 100% and a sensitivity 76% to 91%. This antibody has been incorporated into the diagnostic criteria for the disease, although it is neither a necessary nor a sufficient feature.

How Should Patients With Optic Neuritis Associated With Neuromyelitis Optica Be Treated? In the study by Jarius et al, complete remission was reported in 32% of ON attacks, partial remission in 49%, and no remission in approximately 18%. Classically, NMO attacks are treated with high-dose steroid therapy and/or plasma exchange treatments. Maintenance therapy often begins with prednisone and azathioprine, although mycophenolate mofetil is also used. Recently, rituximab has shown itself to be a moderately effective agent in the treatment of NMO and has become a maintenance therapy in many centers.

Summary ●

Optic neuritis may be a manifestation of MS or NMO.



NMO is associated with significant morbidity; therefore, early diagnosis is important.





Optic neuritis patients manifesting bilateral involvement, poor recovery, and/or features of myelitis should be investigated for NMO. NMO patients require long-term immunosuppressive therapy.

Bibliography Beck RW, Cleary PA, Anderson MM Jr, et al. A randomized, controlled trial of corticosteroids in the treatment of acute optic neuritis. The Optic Neuritis Study Group. N Engl J Med. 1992;326:581-588. Costello F, Burton JM. An approach to optic neuritis: the initial presentation. Expert Rev Ophthalmol. 2013;8(6):539-551. Hickman SJ, Dalton CM, Miller DH, Plant GT. Management of acute optic neuritis. Lancet. 2002;360:1953-1962. Jarius S, Ruprecht K, Wildeman B., et al. Contrasting disease patterns in seropositive and seronegative neuromyelitis optica: a multicentre study of 175 patients. Neuroinflammation. 2012;9:14. Lennon VA, Wingerchuk DM, Kryzer TJ, et al. A serum autoantibody marker of neuromyelitis optica: distinction from multiple sclerosis. Lancet. 2004;364:2106-2112. Wingerchuk DM, Lennon VA, Pittock SJ, et al. Revised diagnostic criteria for neuromyelitis optica. Neurology. 2006;66:1485-1489.

4

QUESTION

HOW SHOULD I EVALUATE AND MANAGE SUSPECTED OPTIC NEURITIS? Eric Eggenberger, DO, MSEpi A 25-year-old woman presents with a sudden loss of vision OD, pain with eye movement, a right relative afferent pupillary defect, and a normal fundus examination. Optic neuritis OD is suspected. What would you recommend for a workup? The first step is to firmly establish the diagnosis; you have done most of the clinical work already in that you mention the existence of an afferent pupillary defect and normal fundus. I would certainly want to know the visual acuity, color vision, and visual field for follow-up purposes, but everything you have mentioned thus far points to a retrobulbar optic neuropathy. Together with the history, I agree with you that this is consistent with retrobulbar optic neuritis. In particular, periorbital pain is very common (92% in the Optic Neuritis Treatment Trial [ONTT]), and a history of pain with eye movements is often elicited. Despite the fact that this sounds archetypal for retrobulbar optic neuritis, it is worth keeping in mind the possibility of other retrobulbar optic neuropathies if the subsequent clinical course mandates. Nevertheless we do not order routine labs as part of the initial workup in such patients. Working on your diagnosis of retrobulbar optic neuritis, management requires magnetic resonance imaging (MRI); then we need to make an acute therapeutic decision (the use of steroids) followed by a chronic therapeutic decision (consideration of multiple sclerosis [MS] immunomodulatory therapy). I typically order an MRI of the brain and orbits with and without contrast (Figures 4-1 and 4-2); the orbital portion helps rule out structural mimics and assists in confirmation (the optic nerve typically enhances if the scan is obtained within the first 3 to 4 weeks). You should be aware that gadolinium-based contrast agents used in MRI may be associated with a progressive fibrosing disease (nephrogenic systemic fibrosis) in patients with renal failure; additional contraindications to gadolinium contrast include known allergy to the agent or pregnancy. In my opinion, a lumbar puncture is not necessary in the majority of typical optic neuritis cases, although the presence of unique spinal fluid oligoclonal bands may be an independent risk factor for the development of MS, especially if the initial MRI is normal.

13

Lee AG, ed. Curbside Consultation in Neuro-Ophthalmology: 49 Clinical Questions, Second Edition (pp 13-16). © 2015 SLACK Incorporated.

14  Question 4 Figure 4-1. Axial fluid-attenuation inversion recovery (FLAIR) MRI study shows periventricular multifocal hyperintense white matter lesions consistent with demyelination.

Figure 4-2. (A) Axial T1-weighted MRI postgadolinium shows enhancing white matter lesions bilaterally. (B) Axial T2-weighted MRI shows periventricular multifocal hyperintense white matter lesions consistent with demyelination.

The ONTT and its follow-up study, the Longitudinal Optic Neuritis Study (LONS), provide the evidence base for the prognosis of optic neuritis and the use of steroids; they also provide data pertaining to the risk of subsequent MS development. The ONTT randomized patients with acute optic neuritis (< 8 days’ duration) to intravenous methylprednisolone (250 mg every 6 hours for 3 days followed by a prednisone taper), oral prednisone (1 mg/kg), or placebo. Regardless of treatment group, the prognosis for visual recovery after optic neuritis is generally quite good, with 95% of patients achieving Snellen acuity of 20/40 or better at 12 months; however, the use of oral prednisone in a dose of 1 mg/kg in the ONTT was associated with twice the risk of recurrent optic neuritis and is therefore contraindicated. Accordingly, our therapeutic decision involves high-dose

How Should I Evaluate and Manage Suspected Optic Neuritis?  15 methylprednisolone or no steroid therapy. High-dose steroids in the ONTT were associated with more rapid recovery of visual function compared to placebo; they may also provide partial protection against a subsequent demyelinating event over the subsequent 2 to 3 years, and often help to resolve pain issues rapidly. The potential side effects of steroids are well known and include hyperglycemia, gastrointestinal symptoms, mood alteration (including rare psychosis), insomnia, and the infrequent occurrence of avascular necrosis. Keeping in mind that high-dose steroids merely enhance the rate of recovery (not the ultimate extent of recovery), I typically will discuss the risks, benefits, side effects, and alternatives with each patient in light of his or her unique medical history and individual preferences. I tend to offer high-dose methylprednisolone (1 g intravenously [IV] every day for 3 days followed by oral prednisone 1 mg/kg every day for 11 days followed by a 4-day taper) to most patients unless there is a contraindication or patient preference dictates otherwise. Optic neuritis has a well-known association with MS and is a common first symptom of the disease. The brain portion of the MRI is essential to stratify this patient’s risk of subsequent MS and is a key factor in the chronic/prophylactic therapeutic decision we need to make regarding immunomodulatory therapy. The LONS demonstrated that a normal MRI is associated with a 5-year MS risk of 16%, while an abnormal MRI (at least 3 T2-weighted MRI hyperintensities typical of demyelination) corresponds to a 5-year MS risk of 51% (see Figures 4-1 and 4-2). At 10 years, a normal MRI is associated with an MS risk of 22%, while any T2-weighted MRI lesion corresponds to an MS risk of 56%. At 15 years, a normal MRI is associated with an MS risk of 25%, while any T2-weighted MRI abnormality translates to an MS risk of 72%. A normal MRI in combination with certain clinical characteristics also defines a cohort with a very low MS risk; anterior optic neuritis (ie, papillitis) in a male with a normal MRI is associated with a very low risk of MS (only 4% at 15 years). In addition, with a normal MRI, several features have strong prognostic value, including severe disc edema, disc hemorrhage, painless onset, the presence of a macular star figure, or no light perception at onset; no subject with these features converted to MS, even after a 15-year follow-up. Trials using interferon beta-1a, interferon beta-1b, or glatiramer acetate in clinically isolated syndrome patients at high risk for MS development have all reported significantly delayed disease progression when these agents are given early (CHAMPS, BENEFIT, and PreCISe trials). The CHAMPS trial randomized patients after a first demyelinating event typical of MS and an MRI with at least 2 T2-weighted MRI lesions to either weekly interferon beta-1a intramuscular or placebo, and approximately 50% of the subjects entered the trial with optic neuritis as their initial demyelinating event. Subjects randomized to interferon beta-1a had a significantly lower conversion rate (42% decreased) to MS compared with subjects on placebo. Very similar findings have since been shown for interferon beta-1b SC every other day and glatiramer acetate; it is not illogical to expect that other MS medications would have similar effects.

Summary ●



A patient presenting with optic neuritis requires an evaluation that includes clinical history and examination; a targeted MRI scan; and discussion concerning the risk, benefits, side effects, and alternatives regarding treatment with high-dose steroids. In appropriate patients with high-MS risk MRI findings, referral to an MS expert or a similar discussion concerning institution of anti-MS immunomodulatory therapy such as interferons is important for long-term neurological health.

16  Question 4

Bibliography CHAMPS Study Group. Interferonb-1a for optic neuritis patients at high risk for multiple sclerosis. Am J Ophthalmol. 2001;132:463-471. Jacobs LD, Beck RW, Simon JH, et al; CHAMPS Study Group. Intramuscular interferon beta-1a therapy initiated during a first demyelinating event in multiple sclerosis. N Engl J Med. 2000;343:898-904. Kaufman DI, Trobe JD, Eggenberger ER, Whitaker JN. Practice parameters: the role of corticosteroids in the management of acute monosymptomatic optic neuritis. Neurology. 2000;54:2039-2044. Optic Neuritis Study Group. The 5-year risk of MS after optic neuritis: experience of the Optic Neuritis Treatment Trial. Neurology. 1997;49:1404-1413. Optic Neuritis Study Group. The clinical profile of acute optic neuritis: experience of the Optic Neuritis Treatment Trial. Arch Ophthalmol. 1991;109:1673-1678.

5

QUESTION

WHAT IS THE TREATMENT OF OPTIC NEURITIS? Melissa W. Ko, MD The patient is a 25-year-old woman with optic neuritis. What are the best treatment options for her? Could you tell me how, when, and why treatment should be given for optic neuritis?

What Is the Evaluation of Optic Neuritis? The diagnosis of optic neuritis is established primarily by clinical history and examination findings. Optic neuritis can occur in isolation or be associated with multiple sclerosis (MS) or neuromyelitis optica (NMO). The first question to be addressed is whether the clinical features of the optic neuritis are typical or atypical. In typical optic neuritis, patients describe loss of vision over 7 to 10 days, pain with eye movements, and some recovery of vision within 30 days of onset. For these patients, a brain MRI with and without contrast should be included in the workup, primarily in order to assess the future risk of MS. Patients with optic neuritis and white matter abnormalities on MRI have the greatest risk of developing MS. In the 15-year follow-up of patients in the Optic Neuritis Treatment Trial (ONTT), the risk of MS was 72% for those with one or more white matter lesions on brain MRI versus 25% for those with a normal baseline MRI. Atypical optic neuritis (ie, infectious, other inflammatory type, or infiltrative) may require different treatments than those used for idiopathic or demyelinating optic neuritis. Some atypical features for you to consider at presentation include retinal exudates, retinal hemorrhages, severe edema of the optic disc, no light perception, and the absence of pain. When these atypical features are present, in addition to an MRI of the brain/orbits, I would recommend consideration for further evaluation, including an antinuclear antibody test, fluorescent treponemal antibody absorption, angiotensin converting enzyme level, Lyme titer, chest x-ray, and lumbar puncture to look for other causes of optic neuropathy. Patients with normal brain MRI and history of recurrent optic neuritis or transverse myelitis may suggest neuromyelitis optica, and obtaining the aquaporin-4‒specific serum autoantibody may be useful (Figure 5-1). Optical coherence tomography and electrophysiological studies may be helpful in select cases where the differentiation of a retinal condition from an optic nerve process is difficult. The results of these tests might alter your decision to treat.

17

Lee AG, ed. Curbside Consultation in Neuro-Ophthalmology: 49 Clinical Questions, Second Edition (pp 17-20). © 2015 SLACK Incorporated.

18  Question 5

1. Clinical features typical? No Yes

MRI brain/orbits w/contrast Consider CSF analysis/serological studies Treatment as appropriate

2. MRI brain w/ contrast High risk for MS (characteristic white matter lesions) No

Yes

Consider intravenous methylprednisolone on individual basis to hasten visual recovery Suggest repeat brain MRI w/contrast in 6 months’ time. If study suggestive of MS lesions, recommend initiation of disease-modifying therapy.

3. Treatment a. Intravenous methylprednisolone sodium succinate (1 g IV/day for 3 days) followed by oral prednisone (1 mg/kg per day for 11 days) with 4-day taper (20 mg on day 1, 10 mg on days 2 and 4), followed by: b. Interferon beta-1a (Avonex 30 μg intramuscularly [IM] weekly, or Rebif 22 or 44 μg weekly or 44 μg subcutaneously [SQ] 3 times/week), interferon beta-1b (Betaseron 250 μg SQ every other day), or glatiramer acetate (Copaxone 20 mg SQ daily) have been demonstrated to significantly reduce the risk of MS and the development of clinically silent MRI lesions in high-risk patients within 2 to 3 years’ follow-up.

Figure 5-1. Algorithm for evaluation of patient with optic neuritis.

What Is the Treatment of Optic Neuritis? The ONTT, the largest and most comprehensive study of the optic neuritis patient population, has provided important data about the clinical presentation and course of acute demyelinating optic neuritis. The primary objective of the ONTT was to determine the effect of intravenous and oral corticosteroid therapy on visual outcomes in acute demyelinating optic neuritis. Patients were randomized to 1 of 3 treatment groups: (1) oral prednisone (1 mg/kg per day for 14 days with a 4-day taper); (2) intravenous methylprednisolone sodium succinate (250 mg every 6 hours for 3 days followed by oral prednisone [1 mg/kg per day] for 11 days with a 4-day taper); or (3) oral placebo.

What Is the Treatment of Optic Neuritis?  19 Patients treated with intravenous methylprednisolone experienced faster recovery of visual function, particularly in the first 2 weeks after onset of visual loss, than did those in the oral prednisone and placebo groups. However, at 1-year follow-up, there were no significant differences in visual function between the treatment groups. Meta-analysis of clinical trials of steroid treatment in MS and optic neuritis have not conclusively determined visual recovery benefit following corticosteroids administration. Oral corticosteroids in conventional doses of 60 mg/day should not be used because there was an associated increased risk of recurrent optic neuritis (41%) compared with those who received intravenous methylprednisolone or placebo (25% in both groups). At 10 years from the initial optic neuritis event, the data continued to show that the risk of recurrent optic neuritis was still higher in the oral prednisone (44%) versus the intravenous group (29%), but there was no longer a statistical difference between the oral prednisone and the placebo groups. Thus, the decision to use corticosteroids for visual recovery alone needs to be individualized, and intravenous steroid is the recommended regimen. If oral corticosteroids need to be used (eg, insurance issues, access problems) in a patient with acute optic neuritis, I would recommend a high-dose oral regimen of 500 to 1,000 mg methylprednisolone for 3 to 5 days; however, this treatment regimen has not been adequately studied in a large cohort of patients. For patients with optic neuritis and 2 or more high-signal abnormalities on baseline brain MRI, intravenous corticosteroids may delay MS onset. However, the effect is short-lived and, by 3 years, treated versus untreated groups had a similar risk of developing MS. Nonetheless, it is common practice to administer high-dose IV corticosteroids to patients with optic neuritis at high risk for MS irrespective of the extent of their visual impairment. There have been several important studies involving treatment with disease-modifying therapies (DMTs) in patients experiencing a first clinical event suggestive of MS. In the CHAMPS study, patients randomized to the treatment group of weekly interferon beta-1a (Avonex) 30 μg IM had a 44% reduction in the 3-year risk of developing clinically definite multiple sclerosis (CDMS) compared with placebo. The ETOMS study found that patients randomized to weekly interferon beta-1a (Rebif, Serono Pharmaceutical) 22 μg SC also demonstrated a lower risk for CDMS (34% versus 45%, P = .047) and a lower annual relapse rate (0.33 versus 0.43, P = .045) compared with placebo. Treatment group patients also demonstrated a decreased lesion burden on MRI and a reduced rate of brain volume loss over 2 years. The REFLEX trial also found that at 2 years, patients with a clinically isolated syndrome treated with either weekly or thrice weekly interferon beta-1a (Rebif) had a lower probability of being diagnosed with MS compared to placebo. The BENEFIT study showed that patients randomized to treatment with every other day interferon beta-1b (Betaseron, Bayer Healthcare Pharmaceutical) had delayed development of CDMS compared to the placebo group. At 2-year follow-up, the probability of developing CDMS was 28% in the treatment group versus 45% in the placebo group. Risk for developing MS was reduced from 85% to 69%. This study also demonstrated a reduction of disability progression in the immediately treated group compared with those who received delayed interferon therapy. In the PreCISe trial, patients randomized to subcutaneous glatiramer acetate (20 mg/day) or placebo for 3 years were noted by intention-to-treat analysis to have a significantly reduced risk of conversion to CDMS (hazard ratio: .55, 95% CI: .40 ‒ .77). For 25% of patients, there was a prolonged time to convert to CDMS (772 days versus 336 for placebo) and a reduced frequency of conversion to CDMS (25% versus 43% with placebo). Efficacy among the high-dose, high-frequency interferons (Betaseron, Rebif) and glatiramer acetate are similar, and head-to-head studies (REGARD, BEYOND) did not show differences in primary outcome measures between the two groups. However, comparisons suggest that Avonex (Biogen Idec) may be inferior in reducing relapse rates and active brain lesions on MRI compared with other DMTs (INCOMIN, EVIDENCE, CombiRx trials). While several new oral agents have been approved recently for treating relapsing/remitting multiple sclerosis (fingolimod,

20  Question 5 dimethyl fumarate, teriflunomide), there is currently insufficient evidence to support their use in patients with only optic neuritis. With the advent of long-term therapy options, it is reasonable to consider whether it is necessary or beneficial to begin indefinite therapy for a chronic disease with a variable natural course. Many physicians argue that we cannot consistently identify such patients and that we may risk long-term, irreversible disability by delaying treatment. Some physicians advocate initial surveillance to better determine the patient’s disease progression before initiating therapy, advocating that many patients could avoid long-term therapy without affecting their prognoses. Until we can reliably determine which patients can safely defer treatment, I recommend offering long-term immunomodulatory therapy to all patients with a first demyelinating event whose MRI suggests a high risk for MS. For those patients with a normal brain MRI following an initial typical attack of optic neuritis, I suggest a repeat brain MRI with contrast in 6 months’ time to reevaluate for the interval development of brain lesions. Ophthalmologists may consider referring such patients to a neurologist for continued surveillance for MS or guidance for initiating MS treatment.

Summary ●







In the setting of typical acute optic neuritis: The use of corticosteroids must be individualized to the patient. If there are white matter lesions suspicious for demyelination, the use of intravenous corticosteroids is suggested. Such agents can hasten visual recovery and delay the onset of MS, but the effect is short-lived, and by 3 years treated versus placebo groups have a similar risk of developing MS. There is no evidence-based role for oral corticosteroids at conventional doses because studies have demonstrated an increased risk of the recurrence of optic neuritis. MRI of the brain with contrast is an important part of the evaluation for assessing the future risk of developing MS. In those patients with optic neuritis with high-signal white matter lesions typical of demyelination on brain MRI, initiation of DMT is recommended. Multiple trials have demonstrated that DMT reduces the risk of developing CDMS. In addition, some trials have shown that immediate treatment following the initial demyelinating event reduced both the probability of developing CDMS and the development of clinically silent MRI lesions in high-risk patients for several years following the event.

Bibliography Beck RW, Cleary PA, Anderson MM, et al. A randomized, controlled trial of corticosteroids in the treatment of acute optic neuritis. N Engl J Med. 1992;326:581-588. Gal RL, Vedula SS, Beck R. Corticosteroids for treating optic neuritis. Cochrane Database Syst Rev. 2012;18:4. The Optic Neuritis Study Group. Multiple sclerosis risk after optic neuritis: final optic neuritis treatment trial follow-up. Arch Neurol. 2008;65(6):727-732.

6

QUESTION

WHAT IS THE WORKUP AND TREATMENT FOR NEURORETINITIS? Karl Golnik, MD and Amina Malik, MD

A 26-year-old man presents with the complaint of having had blurry vision in the right eye for the previous 10 days. He has had no pain and no other symptoms. Examination shows visual acuity of 20/100 OD, a right relative afferent pupillary defect, and a moderately swollen right optic disc with a macular star of exudate. How should I evaluate this patient?

What Is Neuroretinitis? Neuroretinitis (NR) is defined as inflammation of vasculature of the optic disc, causing exudation of fluid into the peripapillary retina. Lipid-rich fluid flows into the outer plexiform layer, where a loose radial configuration leads to the formation of a macular star (Figure 6-1).

How Is Neuroretinitis Classified? NR is broadly classified as infectious, inflammatory, or idiopathic. Most infectious cases are due to cat-scratch disease (CSD) caused by Bartonella species. In one series, two-thirds of cases of NR were due to CSD. Other infectious etiologies of NR include syphilis, Lyme disease, histoplasmosis, brucellosis, HIV, toxoplasmosis, Epstein-Barr virus, hepatitis B and C, and tuberculosis. The reported average age of onset is 24.5 years, with a range from 4 to 64 years and a female predilection (1.8:1 female-to-male incidence). Of these patients, 73% manifest systemic symptoms, while only 7.7% report eye pain. Initial visual acuity is 20/200 or worse in 52.2% of patients, while final visual acuity is 20/40 or better in 93%. Inflammatory etiologies include sarcoidosis or polyarteritis nodosa. NR has also been described in association with idiopathic retinal vasculitis and diffuse unilateral subacute neuroretinitis.

21

Lee AG, ed. Curbside Consultation in Neuro-Ophthalmology: 49 Clinical Questions, Second Edition (pp 21-24). © 2015 SLACK Incorporated.

22  Question 6 Figure 6-1. (A) Two days after initial symptoms, the optic disc is swollen; there is macular edema but no star of exudate. (B) Ten days after initial symptoms, a macular star of exudate has formed.

A

B

NR is considered idiopathic when an infectious or inflammatory etiology is not found, which occurs in about half of all cases. These patients have an average age of onset of 28 years, with an age range from 8 to 55 years. More than 50% have a preceding flu-like illness, usually an upper respiratory infection. Most cases of visual loss are unilateral and painless, with a usual range from 20/50 and 20/200, although vision has been shown to range from 20/20 to light perception only. The most common pattern of visual field loss is a central or cecocentral scotoma from edema of the papillomacular bundle. Most patients recover excellent visual acuity with or without treatment; a 20/40 or better final outcome occurred in 90% of reported cases.

How Does Neuroretinitis Appear Clinically? NR is a clinical diagnosis. Initially, inflammation of the optic disc may be the only sign, as the macular star can take up to 2 weeks to manifest (see Figure 6-1). The disc swelling resolves over weeks, leaving either a normal disc or an optic atrophy; however, the macular exudate can persist for months. Optical coherence tomography (OCT) may show retinal thickening, subretinal fluid, and fluid or exudates within the outer plexiform layer. OCT may also be useful in detecting a serous retinal detachment before a macular star forms.

What Is the Differential Diagnosis of Neuroretinitis? The differential diagnosis for optic nerve swelling with a macular star includes hypertensive retinopathy, papilledema, sarcoidosis, toxoplasmosis, syphilis, tuberculosis, Lyme disease, anterior ischemic optic neuropathy, diabetic papillopathy, posterior vitreous traction, optic disc

What Is the Workup and Treatment for Neuroretinitis?  23 and juxtapapillary tumors, and, rarely, toxic etiologies, including bis(chloroethyl), nitrosourea, and procarbazine. In hypertensive optic neuropathy, retinopathy, and most cases of papilledema, fundus abnormalities are bilateral, while NR is typically unilateral. Also, in hypertensive retinopathy, cotton wool spots will often be seen scattered in the posterior pole. Macular exudates are sometimes seen with papilledema; the presence of symptoms and signs of increased intracranial pressure (ICP) should lead to the correct diagnosis. Brain neuroimaging and lumbar puncture may be needed for confirmation. Rarely, a macular star is seen in nonarteritic anterior ischemic optic neuropathy (NAION), but the exudation is typically mild and the star is often incomplete. The presence of vitreous cells would be consistent with NR rather than with NAION. Clinical factors, including age and vascular risk factors, should also be considered to help differentiate these causative etiologies.

What Is the Appropriate Workup? Many of the causative agents are treatable, so accurate diagnosis is important. A complete workup should include a thorough history and medical evaluation. A detailed social history should be obtained, including recent travel, sexual contacts, unpasteurized and uncooked foods, and animal contacts, including cats. Although CSD is most often identified as the cause of NR, not everyone will have had or will remember exposure to cats. Physical examination should be performed, with particular attention to rashes and inoculation sites; it should also include a blood pressure check. Laboratory testing should be guided by clinical suspicion and tailored to history and physical examination. Because Bartonella is so often the cause, Bartonella titers should be obtained. Fluorescent treponemal antibody, rapid plasma reagin, purified protein derivative skin testing, Lyme serology, angiotensin converting enzyme, and chest x-ray could also be included in baseline testing. If an infectious etiology is suspected but acute serologies are negative, retesting at 6 weeks for rising IgG convalescent titers is appropriate.

What Is the Appropriate Treatment? The treatment of NR is directed at the underlying etiology. Suspected infectious cases may require broad-spectrum antibiotics while results are pending. If the causative agent is found to be CSD, a variety of treatment methods are suggested in the literature, including no treatment, antibiotics only, antibiotics and steroids, and steroids only. It has been well documented that most patients will improve spontaneously. However, a multitude of studies suggest different treatment regimens to shorten the disease course. Recommended antibiotics change yearly but might include doxycycline, ciprofloxacin, or azithromycin for adults and azithromycin or sulfamethoxazoletrimethoprim for children. In idiopathic NR, most patients have spontaneous, excellent visual recovery. Although unproven, high-dose oral corticosteroids can be administered to shorten the disease course.

Summary ●



Neuroretinitis is defined as inflammation of the neural retina and optic nerve. Etiology can be infectious (most commonly due to cat scratch disease from Bartonella henselae), inflammatory, or idiopathic.

24  Question 6 ●



Examination shows isolated disc edema early, with macular edema occurring up to 2 weeks later. Most patients with idiopathic neuroretinitis recover excellent visual acuity with or without treatment.

Acknowledgment Work on this chapter was supported in part by a grant from Research to Prevent Blindness.

Bibliography Cunningham ET, Koehler JE. Ocular bartonellosis. Am J Ophthalmol. 2000;130(3):340-349. Gass JDM. Diffuse unilateral subacute neuroretinitis. In: Stereoscopic atlas of macular disease: diagnosis and treatment. 4th ed. St. Louis, MO: CV Mosby; 1997:622-628. Nourinia R, Montahai T, Amoohashemi N, et al. idiopathic retinal vasculitis, aneurysms and neuroretinitis syndrome associated with positive perinuclear antineutrophil cytoplasmic antibody. J Ophthalm Vis Res. 2011;6(4): 330-333. Purvin V, Sundaram S, Kawasaki A. Neuroretinitis: review of the literature and new observations. J Neuroophthalmol. 2011;31: 58-68. Ray S, Gragoudas E. Neuroretinitis. Int Ophthalmol Clin. 2001;41(10):83-102. Suhler EB, Lauer AK, Rosenbaum JT. Prevalence of serologic evidence of cat scratch disease in patients with neuroretinitis. Ophthalmology. 2000;107:871-876.

7

QUESTION

HOW DO YOU EVALUATE AND TREAT NONARTERITIC ANTERIOR ISCHEMIC OPTIC NEUROPATHY? M. Tariq Bhatti, MD My patient is a 45-year-old man with acute onset visual loss OD. He has a right relative afferent pupil defect and a swollen optic nerve OD. The left eye is normal. He has some pain but not with eye movement. Can you tell me how you would go about making the diagnosis of nonarteritic anterior ischemic optic neuropathy versus optic neuritis or another optic neuropathy? A middle-aged man with a right optic neuropathy associated with pain, which I assume is in the region of the right eye, and a swollen optic nerve—that is a tough case. At 45 years of age, the 2 most common causes of a swollen optic nerve are nonarteritic anterior ischemic optic neuropathy (NAION) and optic neuritis (ON). However, it should be noted that the differential diagnosis of a swollen optic nerve is extensive and includes other inflammatory, vascular, orbital, toxic/ metabolic, hereditary, infiltrative, and compressive causes. If we are to assume this is NAION, the diagnosis is based on the history, clinical examination, and disease course. In other words, NAION is a clinical diagnosis. If laboratory evaluation and neuroimaging are performed, they are often done to exclude other possibilities, such as ON and the multitude of vascular, infectious, neoplastic, and inflammatory optic neuropathies. In this patient, a key piece of information that is helpful in the diagnosis is the fact that he has pain but not with eye movement. This is important because nearly 90% of patients with ON will have eye pain exacerbated by eye movement, compared with only 10% of patients with NAION who note mild eye pain without eye movement. If we were dealing with someone older than age 55 years, it would be critical to ascertain a history of headache, scalp tenderness, jaw claudication, weight loss, or neck pain for the possibility of arteritic anterior ischemic optic neuropathy due to temporal arteritis. The configuration of the fellow optic nerve should also be noted in any patient suspected of NAION. In the majority of cases, a small cup-to-disc ratio is present, termed “the disc at risk” (Figure 7-1). On formal visual field testing, an inferior altitudinal defect is the most common visual field defect seen with NAION (Figure 7-2).

25

Lee AG, ed. Curbside Consultation in Neuro-Ophthalmology: 49 Clinical Questions, Second Edition (pp 25-28). © 2015 SLACK Incorporated.

26  Question 7 Figure 7-1. Disc at risk. Note the small cupto-disc ratio and the crowded appearance of the nerve.

Figure 7-2. Inferior altitudinal visual field defect in a left eye with NAION.

In most patients with NAION, vision remains relatively stable. However, I caution patients that as long as the optic nerve is swollen, they may continue to lose vision. Progressive visual loss can occur over 6 to 8 weeks in approximately one third of patients. Some 40% of patients may show spontaneous visual improvement of 3 lines of vision or more 6 months from the onset of the visual loss. There are a few recommendations I would make regarding your patient’s diagnostic workup. Although my suspicion is high that he has NAION, given the pain and his relatively young age, MRI of the brain and orbits with contrast could be offered to evaluate for the possibility of demyelinating disease.

How Do You Evaluate Nonarteritic Anterior Ischemic Optic Neuropathy?  27 I do not routinely order an echocardiogram because embolic cardiac or aortic arch disease is rarely a cause of NAION, especially if there is no evidence of retinal emboli on the clinical examination. This patient is too young for temporal arteritis, but in a case involving an older person, I would perform a sedimentation rate, C-reactive protein, and complete blood count. Although the exact pathogenesis of NAION is not known, it is thought to be a vasculopathy of the paraoptic branches of the posterior ciliary arteries that feed the retrolaminar portion of the optic nerve head. Various risk factors have been associated with NAION, including optic disc morphology (disc at risk), diabetes mellitus, arterial hypertension, nocturnal hypotension, hyperlipidemia, medications (ie, amiodarone, sildenafil citrate), migraine, and optic nerve drusen. If the patient does not have one of the “big three” (diabetes mellitus, arterial hypertension, or hyperlipidemia), I would recommend a complete physical examination and blood workup by the primary physician or internist. There is some controversy as to whether patients with NAION are truly at an increased risk of cerebrovascular accidents or ischemic cardiac disease compared with the general population. If the patient has diabetes mellitus, arterial hypertension, hyperlipidemia, or a history of smoking cigarettes, then risk modification should be strongly encouraged. Patients with NAION in one eye should be counseled that there is an approximate 15% chance that the fellow eye will be involved within the next 5 years. Although it is controversial, you could start a daily aspirin if there is no contraindication to possibly lessen this risk. I find it disappointing to have to tell my patients that there is no proven therapy for NAION, but I try to stay optimistic by reminding them that we are trying to find a treatment. Surgical procedures such as transvitreal neurotomy and vitrectomy (for vitreous traction) have not been conclusively shown to be of benefit for patients with NAION. In the Ischemic Optic Neuropathy Decompression Trial, optic nerve sheath fenestration was found to be ineffective and in some cases harmful. A variety of medications—including topical brimonidine, intravitreal bevacizumab, anticoagulants, thrombolytics, levodopa, and diphenylhydantoin—have been tried to improve vision, all with poor or inconclusive results. Recently, a prospective nonrandomized evaluator-masked study of oral systemic corticosteroids found that approximately 70% of patients with visual acuity of 20/70 or worse who were treated within 2 weeks of onset of the visual loss experienced visual improvement at 6 months. These treated eyes were compared with a natural history cohort in which only 40.5% improved at 6 months. After 1 year, when all patients treated with corticosteroids regardless of initial visual acuity or time of initiation of treatment were considered, 25% of patients demonstrated improvement in visual acuity while approximately 60% were unchanged. However, the use of oral corticosteroids in the treatment of NAION remains hotly debated. There have been significant advancements in our understanding of the cellular and molecular pathogenesis of NAION based on the development of eloquent experimental animal models. These models show a significant inflammatory response with retinal ganglion cell death. Such insights will help us to develop novel therapies, such as the recently presented results of a phase I clinical trial that investigated the intravitreal safety of a small interfering RNA (siRNA) to protect retinal ganglion cells from apoptosis. In terms of follow-up, patients should be monitored until the optic nerve swelling resolves, which usually occurs within 4 to 6 weeks (Figure 7-3). Either diffuse or segmental optic nerve pallor becomes evident at this time and is associated with a fixed visual field defect. It is very rare (< 5%) for a second ischemic event to occur in an eye with a previous attack of NAION; if this happens, I often perform a complete hypercoagulable and inflammatory workup. I remind patients who have experienced NAION in one eye of the nearly 3.5-fold increased risk of NAION with cataract surgery in the fellow eye.

28  Question 7 Figure 7-3. (A) At the time of presentation, the inferior optic nerve was swollen, which is associated with hyperemia and peripapillary hemorrhage. (B) At 6 weeks, the optic nerve swelling was replaced by pallor.

Summary ●





Patients with NAION often present with painless loss of vision associated with a swollen optic nerve and a small cup-to-disc ratio in the fellow eye. NAION is an idiopathic condition often associated with diabetes mellitus, hyperlipidemia, and arterial hypertension. An erythrocyte sedimentation rate, C-reactive protein, and complete blood count should be performed for the evaluation of giant cell arteritis in elderly (> 50 years of age) patients with NAION.



Patients with NAION routinely do not require an echocardiogram or carotid ultrasound.



Identifiable risk factors should be referred for appropriate treatment and modification.



Currently there is no treatment for NAION. Daily aspirin may lower the risk of fellow eye involvement.

Bibliography Atkins EJ, Bruce BB, Newman NJ, Biousse V. Treatment of nonarteritic anterior ischemic optic neuropathy. Surv Ophthalmol. 2010;55:47-63. Beck RW, Hayreh SS. Role of aspirin in reducing the frequency of second eye involvement in patients with non-arteritic anterior ischaemic optic neuropathy. Eye (Lond). 2000;14(Pt 1):118. Bernstein SL, Johnson MA, Miller NR. Nonarteritic anterior ischemic optic neuropathy (NAION) and its experimental models. Prog Retin Eye Res. 2011;30:167-187. Hayreh SS. Ischemic optic neuropathy. Prog Retin Eye Res. 2009;28:34-62. Hayreh SS, Zimmerman MB. Non-arteritic anterior ischemic optic neuropathy: role of systemic corticosteroid therapy. Graefes Arch Clin Exp Ophthalmol. 2008;246(7):1029-1046. Katz BJ. A phase I open label, dose escalation tiral of QPI-1007 develivered by a single intravitreal (IVT) injection to subjects with low visual acuity and acute non-arteritic anterior ischemic optic neuropathy (NAION). Paper presented at NANOS annual meeting. Snowbird, Utah; 2013. Lam BL, Jabaly-Habib H, Al-Sheikh N, et al. Risk of non-arteritic anterior ischaemic optic neuropathy (NAION) after cataract extraction in the fellow eye of patients with prior unilateral NAION. Br J Ophthalmol. 2007;91:585-587. Lee AG, Biousse V. Should steroids be offered to patients with nonarteritic anterior ischemic optic neuropathy? J Neuroophthalmol. 2010;30:193-198. Newman NJ, Scherer R, Langenberg P, et al. The fellow eye in NAION: report from the ischemic optic neuropathy decompression trial follow-up study. Am J Ophthalmol. 2002;134:317-328. Optic nerve decompression surgery for nonarteritic anterior ischemic optic neuropathy (NAION) is not effective and may be harmful. The Ischemic Optic Neuropathy Decompression Trial Research Group. JAMA. 1995;273:625-632.

8

QUESTION

HOW DO YOU DIFFERENTIATE ARTERITIC FROM NONARTERITIC ANTERIOR ISCHEMIC OPTIC NEUROPATHY? Rod Foroozan, MD My patient, a 72-year-old man, noted the sudden onset of decreased vision in his right eye. He was found to have a right relative afferent pupillary defect and diffuse optic disc edema on the right. How should I evaluate him? Anterior ischemic optic neuropathy (AION) is the most common acute optic neuropathy in patients over 50 years of age. The classification of AION has been divided into that which is due to some type of vasculitis (arteritis), so-called arteritic AION (AAION), and the spontaneous idiopathic type, so-called nonarteritic AION (NAION). Although there is no proven treatment for typical NAION, the distinction is important because corticosteroids may preserve visual function in patients with AAION. In addition, patients with AAION may develop life-threatening sequelae from vasculitis, such as aortitis, which does not develop in patients with typical NAION. By definition, patients with AION must have optic disc edema accompanying the visual loss. Patients with AAION from giant cell arteritis (GCA) tend to be older and to have a more profound visual loss than those with NAION, who have visual acuity of 20/200 or better in 60% of cases; acuity is 20/200 or worse in 75% of patients with AAION. While NAION is typically unilateral, AAION may cause bilateral visual loss after days to weeks of the onset of symptoms, particularly if not treated with corticosteroids. Although AAION may be caused by several forms of vasculitis, by far the most common is that due to GCA. In patients over 50 years of age with optic disc edema, I will ask about systemic symptoms suggestive of GCA, including pain in the jaw with chewing, new-onset headaches, polymyalgia rheumatica, fevers, weight loss, scalp tenderness, and prior diplopia or transient visual loss. While vasculopathic risk factors—such as diabetes mellitus, hypertension, hyperlipidemia, and sleep apnea—may be important in patients with typical NAION, systemic symptoms should be coincidental or not present. Despite this distinction, some patients with AAION may not have systemic symptoms.

29

Lee AG, ed. Curbside Consultation in Neuro-Ophthalmology: 49 Clinical Questions, Second Edition (pp 29-32). © 2015 SLACK Incorporated.

30  Question 8

Figure 8-1. Arteritic anterior ischemic optic neuropathy from giant cell arteritis. The normal right optic disc has a sizable cup, and there is pallid swelling of the inferior half of the left optic disc. Figure 8-2. Nonarteritic anterior ischemic optic neuropathy. The swelling of the right optic disc (left) is hyperemic and sectoral, with involvement of the superior portion of the disc. In the asymptomatic eye the left optic disc is small, with a small optic cup.

While ancillary testing using elevations in the Westergren erythrocyte sedimentation rate, C-reactive protein, and platelet count may be helpful, I try to determine whether, based on my clinical suspicion, the patient should undergo a temporal artery biopsy. That suspicion may be increased by funduscopic findings in GCA, which may provide a clue to the presence of vasculitis, even in the absence of systemic symptoms. I like to think of AAION as being accompanied by findings suggestive of ischemia from more widespread involvement of the ocular circulation than would be expected in typical NAION (where theoretically only the short posterior ciliary arteries are involved). These findings include the following: More profound infarction of the optic disc, causing “chalky white” pale swelling (Figure 8-1). In NAION the optic disc edema is often hyperemic and sectoral (Figure 8-2). ●







The presence of a small optic disc and cup is important in NAION, whereas in AAION, the optic disc and cup may be small or large (see Figures 8-1 and 8-2). The presence of adjacent retinal infarction in a patient with AION. The presence of cotton wool spots away from the optic disc (Figure 8-3). While cotton wool spots may occur on the disc in NAION, they do not typically occur away from the optic disc.

How Do You Differentiate Arteritic From Nonarteritic AION?  31 Figure 8-3. Arteritic anterior ischemic optic neuropathy from giant cell arteritis. There is a cotton wool spot along the inferotemporal vascular arcade, away from the optic disc, in a patient with pale optic disc edema from arteritic anterior ischemic optic neuropathy.

Figure 8-4. Optic disc pallor after anterior ischemic optic neuropathy. (A) After resolution of optic disc edema from arteritic anterior ischemic optic neuropathy, the right optic disc is pale and cupped in a patient with giant cell arteritis. (B) After resolution of optic disc edema, the right optic disc is pale without cupping in a patient with nonarteritic anterior ischemic optic neuropathy.



Simultaneous involvement of both eyes or more than one circulation (choroidal, retinal, and optic disc), such as a combined retinal artery occlusion in one eye and AION in the fellow eye or a cilioretinal artery occlusion in the same eye in a patient with AION.

The presence of optic disc cupping after disc edema resolves (Figure 8-4). Optic disc pallor, often segmental, with arteriolar attenuation and without optic disc cupping, occurs in patients with NAION. Any of these findings in and of themselves may suggest the need for a temporal artery biopsy or further evaluation for vasculitis. A positive response to treatment with corticosteroids while awaiting the results of a temporal artery biopsy may also be helpful. This response may include an improvement in systemic symptoms, funduscopic findings, and ancillary blood test markers of inflammation. ●

32  Question 8

Summary ●





While the anatomic configuration of a small optic disc and cup is a risk factor for nonarteritic anterior ischemic optic neuropathy, this finding does not play a role in the pathophysiology of arteritic anterior ischemic optic neuropathy. Ocular findings during fundoscopy often play a key role in raising the suspicion for arteritic anterior ischemic optic neuropathy. Funduscopic findings, including more widespread involvement of the ocular circulation, suggest that anterior ischemic optic neuropathy may be arteritic and suggest the need for a temporal artery biopsy.

Bibliography Atkins AJ. Bruce BB, Newman NJ, Biousse V. Treatment of nonarteritic anterior ischemic optic neuropathy. Surv Ophthalmol. 2010;55:47-63. Danesh-Meyer H, Savino PJ, Spaeth GL, Gamble GD. Comparison of arteritis and nonarteritic anterior ischemic optic neuropathies with the Heidelberg retina tomograph. Ophthalmology. 2005;112:1104-1112. DelMonte DW, Bhatti MT. Ischemic optic neuropathy. Int Ophthalmol Clin. 2009;49:35-62. Hayreh SS, Podhajsky PA, Zimmerman B. Ocular manifestations of giant cell arteritis. Am J Ophthalmol. 1998; 125:509-520.

9

QUESTION

HOW SHOULD I TREAT GIANT CELL ARTERITIS? Jacqueline Leavitt, MD My patient is a 70-year-old woman with osteoporosis and a history of falls. Now she has developed no light perception in the right eye and a pale, swollen right optic nerve. A temporal artery biopsy was positive for giant cell arteritis. How should a patient with biopsy-proven giant cell arteritis be treated and how should the steroids be dosed? When vision is at risk, using intravenous steroids initially to treat giant cell arteritis (GCA) may lead to improved vision and/or decreased length of treatment. The best prospective study so far supports starting patients on 15 mg/kg per day of methylprednisolone for 3 days. This is followed by oral glucocorticoid maintenance. In patients with constitutional symptoms whose vision is not at risk, treatment with oral prednisone normally starts at 60 mg or more per day (1 to 1.5 mg/kg).

What About the Long-Term Treatment? There is no firm timeline for the long-term treatment of GCA with steroids. Treatment should be started even before the temporal artery biopsy is done, as long as the biopsy will be done within 1 week or so. After steroid therapy, most patients begin to feel better within a few days. It has been our experience at Mayo Clinic that the steroids can often be tapered slowly, beginning 1 to 2 months after onset of treatment, and then tapered down by about 10 mg/week, so that within a few months the dose is modest (10 to 20 mg/day). Slower tapering of the daily dosage by 1-mg increments and then continuing for a total treatment regimen of 1 to 2 years suffices for most patients.

How Do You Determine How to Reduce the Steroid Dose? I use symptoms and the blood tests to determine the rate of the taper. The absence of new or worsening clinical symptoms, as well as a normal/low erythrocyte sedimentation rate (ESR) and C-reactive protein (CRP), generally indicate that continued slow tapering or discontinuation of treatment is possible. If the patient experiences recurrences of the symptoms, develops new ischemic symptoms, or has a marked elevation of the ESR or CRP, increasing or restarting the steroid therapy is warranted. Lee AG, ed.

33

Curbside Consultation in Neuro-Ophthalmology: 49 Clinical Questions, Second Edition (pp 33-35). © 2015 SLACK Incorporated.

34  Question 9

What Side Effects Can the Patient Expect? Prolonged use of steroids can be associated with significant side effects (eg, weight gain, glucose intolerance, hypertension, osteoporosis, avascular necrosis, opportunistic infections). Side effects are much less with every-other-day steroids, but this regimen does not provide adequate therapy for GCA and is not recommended. Steroid-sparing agents are sometimes used to alleviate side effects. A recent meta-analysis of 3 studies using methotrexate as a steroid-sparing treatment of GCA found a benefit in preventing relapses; however, there was a latency of effect of 6 months.

What Are the Most Worrisome Complications of Giant Cell Arteritis? One of the infrequent complications from GCA is aortic aneurysm. Patients with GCA were found to be 17 times more likely to develop an aortic aneurysm and 2.4 times more likely to develop an isolated abdominal aortic aneurysm than age- and sex-matched controls in an Olmsted County incidence study. Aortic dissection is another complication associated with GCA.

Are There Any New Treatments for Giant Cell Arteritis? There are new immunopathological data about GCA that may help to determine the best therapy and ideal length of treatment. There are two different T-cell responses within the temporal arteries in GCA. The T-cell reaction is different between early (at time of diagnosis) and late GCA. Both Th1 and Th17 T-cells are present early in GCA; however, months after steroid therapy is initiated, only Th1 cells remain. Cytokines produced by Th1 cells are steroid-resistant. Perhaps in the future, therapies directed at Th1 T-cells will help treat cases of steroid-resistant GCA and allow for better determination of the appropriate length of treatment. Although there are no clinical trials in the literature to guide length of treatment in GCA, for now the patient’s clinical course and the clinician’s judgment are the best way to determine a treatment plan for each individual.

Summary ●

Start IV steroids if vision is at risk.



Start steroids prior to obtaining temporal artery biopsy.





Taper steroids slowly after a few months; use symptoms, ESR, and CRP to determine speed of taper. Steroid side effects can include weight gain, glucose intolerance, avascular necrosis, and opportunistic infections.



Aortic aneurysms occur more often in GCA.



Future therapies may target the different T-cell responses in GCA.

How Should I Treat Giant Cell Arteritis?  35

Bibliography Chan CC, Paine M, O’Day J. Steroid management in giant cell arteritis. Br J Ophthalmol. 2001;85:1061-1064. Chevalet P, Barrier JH, Pottier P, et al. A randomized, multicenter, controlled trial using intravenous pulses of methylprednisolone in the initial treatment of simple forms of giant cell arteritis: a one year follow study of 164 patients. J Rheumatol. 2000;27:1484-1491. Deng J, Younge BR, Olshen RA, et al. Th17 and Th1 T-cell responses in giant cell arteritis. Circulation. 2010;121:906-915. Evans JM, O’Fallon WM, Hunder GG. Increased incidence of aortic aneurysm and dissection in giant cell (temporal) arteritis: a population-based study. Ann Intern Med. 1975;122:502-507. Fraser JA, Weyand CM, Newman NJ, Biousse V. The treatment of giant cell arteritis. Rev Neurol Dis. 2008;5:140-152. Klein RG, Hunder GG, Stanson AW, Sheps SG. Large artery involvement in giant cell (temporal) arteritis. Ann Intern Med. 1975;83:806-812. Mahr AD, Jover JA, Spiera RD, et al. Adjunctive methotrexate for treatment of giant cell arteritis: an individual patient data meta-analysis. Arthritis Rheum. 2007;56:2789-2797. Mazlumzadeh M, Hunder GG, Easley KA, et al. Treatment of giant cell arteritis using induction therapy with high-dose corticosteroids. Arthritis Rheum. 2006;54:3310-3318. Weyand CM, Liao YJ, Goronzy JJ. The immunopathology of giant cell arteritis: diagnostic and therapeutic implications. J Neuroophthalmol. 2012;32:259-265.

10 QUESTION

WHAT IS THE EVALUATION OF TRAUMATIC OPTIC NEUROPATHY? Nicholas Volpe, MD and Jennifer K. Hall, MD

A 20-year-old man struck his right forehead after flipping over his bicycle. He did not lose consciousness and has no other neurological symptoms. He was seen in the emergency department and discharged. Now, however, he complains of visual loss OD. He has a right relative afferent pupillary defect (RAPD) but a normal-appearing optic disc OD. The left eye is normal. He is believed to have traumatic optic neuropathy (TON). What imaging, if any, should be performed? Steroids cannot hurt, right? Your patient’s story and presentation would indeed raise high suspicion for TON. Traumatic injury to the optic nerve can occur through either direct or indirect mechanisms. Direct injury is caused by penetrating trauma, while a blow to the brow or facial eminences can cause indirect injury. Indirect trauma to the optic nerve is further divided into posterior indirect TON and optic nerve avulsion (partial or complete severance of the optic nerve from the globe). The story above is most consistent with posterior indirect TON. The incidence of TON is highest in young men (as is any trauma), with bicycle and motor vehicle accidents providing the most common setting. Other causes include injuries from falls, falling objects, gunshots, assault, and skateboards. TON can occur after seemingly minor trauma. The incidence is 2% to 5% after facial trauma. The criteria for the diagnosis of posterior indirect TON include decreased acuity and color vision, a visual field defect, a RAPD in unilateral cases, a normal-appearing fundus, and a history of trauma. In most cases, because the injury is to the facial bones, there may be periorbital ecchymoses and/or brow laceration, but the globe is normal with no evidence of traumatic iritis, hyphema, vitreous hemorrhage, or commotio retinae. The visual field defect can take any form, although altitudinal defects and central scotomas are common. Visual acuity is usually 20/400 or worse; however, acuity can be as poor as no light perception or as good as 20/20. Vision loss from indirect posterior TON can be biphasic, with immediate compromise followed by delayed worsening. Direct TON is diagnosed based on history and clinical or radiological evidence of penetrating injury. In cases of optic nerve avulsion, a complete or partial ring of hemorrhage can be seen at the optic disc (Figure 10-1). In general, diagnoses to rule out when TON is being considered

37

Lee AG, ed. Curbside Consultation in Neuro-Ophthalmology: 49 Clinical Questions, Second Edition (pp 37-40). © 2015 SLACK Incorporated.

38  Question 10 Figure 10-1. Fundus photograph showing a partial ring of hemorrhage at the optic disc, typical of optic nerve avulsion.

include preexisting optic neuropathy, retinal compromise secondary to trauma, and functional (nonorganic) vision loss. We typically recommend that a computed tomography (CT) scan be done as part of the workup of a suspected TON. Direct coronal cuts (1.5-mm intervals) are desirable if the patient can be positioned safely, but newer CT coronal reconstructions can also be satisfactory. This allows superior evaluation of the optic canal. The canal should be carefully examined for any fractures and particularly for bony fragments that may be impinging on the nerve. Other findings that might be amenable to surgical treatment include an optic nerve sheath or subperiosteal hematoma or hemorrhage in the orbital apex, sphenoid sinus, or ethmoid sinus. Once a metallic foreign body or other contraindications are ruled out, magnetic resonance imaging (MRI) can be used to better evaluate soft tissue abnormalities but may not be necessary. CT can also play a role in surgical planning for optic canal decompression. In direct TON, a penetrating object causes direct injury to the nerve. Optic nerve avulsion can also occur in this setting. Optic nerve avulsion can also occur with indirect TON, secondary to twisting forces on the globe. Injury from indirect posterior TON is thought to occur within the optic canal. Following a frontal blow, sudden deceleration of the head with continued forward motion of the globe causes shearing forces along the intracanalicular nerve, where it has firm dural attachments. In addition, it has been demonstrated that the optic foramen is the major site of transmitted force from frontal blows, and sheering forces injure the intracanalicular optic nerve. Immediate vision loss from posterior indirect TON is thought to occur secondary to mechanical shearing of axons, as well as contusion necrosis secondary to ischemia from microvasculature compromise. A combination of apoptotic mechanisms, reperfusion injury, and edema (especially within the confines of the tight bony canal) is thought to be responsible for delayed vision loss. Currently, there is no evidence-based standard of care for the treatment of TON. Spontaneous improvement in acuity occurs in some patients. Treatment options have included steroids and/ or optic canal decompression. There have been no randomized clinical trials to support either of these modalities, and no clear consensus on the efficacy of these treatments has emerged from multiple retrospective or prospective descriptive studies. In addition, the National Acute Spinal Cord Injury Studies, which are often cited as a rationale for the treatment of TON with megadose steroids, do not demonstrate clearly beneficial outcomes and may not apply to TON. These studies

What Is the Evaluation of Traumatic Optic Neuropathy?  39 investigated the steroid treatment for acute brain or spinal cord injury, but not specifically TON. The most convincing benefit was seen in the group treated with megadoses of steroids within 8 hours of injury, although even this benefit may not be statistically significant. Importantly, there is some evidence that steroids may be detrimental. TON has been shown to worsen after megadoses of steroids in animal studies. In addition, results from the Corticosteroid Randomisation After Significant Head Injury (CRASH) study suggest that high-dose steroids are associated with increased mortality when given in the context of head injury. This large randomized placebocontrolled study investigated outcomes following megadose steroid treatment (2-g loading dose followed by 0.4 g per hour over 48 hours) versus placebo in 10,008 patients who had experienced head injury. The overall mortality rate 2 weeks following the injury was 21.1% in the steroid group and 17.9% in the placebo group (P = .0001). This refutes previous smaller studies that had suggested decreased mortality following steroid treatment for head injury. Based on an evaluation of the literature and interaction with colleagues, it appears that the trend in the treatment of TON was significantly affected by the CRASH study (published in 2004). Specifically, it appears that treatment with megadose steroids has diminished considerably since this study, particularly in the United States. Although the increase in mortality seen in the CRASH study with megadose steroid treatment pertained specifically to patients with head injury, this finding—in conjunction with the lack of rigorous evidence supporting the practice and other evidence suggestive of harm (animal studies)—causes me to agree with the trend away from aggressive steroid treatment. Relatively low-dose oral steroids (1 mg/kg for 1 to 2 weeks, followed by a short taper) or intravenous steroids (250 mg 4 times daily for 24 to 48 hours) in patients without associated traumatic brain injury are fairly low-risk options that may be beneficial. However, there is no rigorous evidence supporting this treatment. There is a theoretical rationale that steroids may lead to improvement in vision via reduction of edema. Optic canal decompression (even in the absence of obvious bony fragment impingement on the nerve) can also be considered but has not been proven to be superior to the natural history of this condition. In the case of radiologic evidence of a bony fragment or hematoma impinging on the optic nerve, surgery can be considered, particularly if there is progressive vision loss. However, the significant risks associated with such surgery need to be considered, and this has not been studied prospectively. In particular, additional injury to the optic nerve can occur, causing associated worsening of vision, and the proximity of the carotid artery to the surgical site adds to the risk of stroke or death. The experience of the surgeon should be taken into account in the decisionmaking process.

Summary In most cases, based on non–evidenced-based anecdotal experience, we treat some TON patients with observation or relatively low-dose steroids. You should image with CT and, if no contraindication, then possibly an MRI to evaluate for conditions that may be amenable to surgery. We have had some personal experience with improvement after optic canal decompression and recommend considering decompression surgery in certain circumstances, particularly if the patient can be examined in detail and there is evidence of continued deterioration on low-dose steroids. Ultimately, the clinician is left without clear evidence indicating that any treatment is better than the natural history in these patients. TON is a clinical diagnosis. ●



CT scan might show a canal fracture, orbital fracture, bony fragment, or sheath or orbital hematoma; these might be surgical considerations.

40  Question 10 ●



There are no randomized clinical trial data to provide evidence-based recommendations. Corticosteroids could be considered but are unproven, and risk-to-benefit analysis must be considered.

Bibliography Levin LA, Beck RW, Joseph MP, et al. The treatment of traumatic optic neuropathy: the International Optic Nerve Trauma Study. Ophthalmology. 1999;106:1268-1277. Roberts I, Yates D, Sandercock P, et al. Effect of intravenous corticosteroids on death within 14 days in 10008 adults with clinically significant head injury (MRC CRASH trial): randomized placebo-controlled trial. Lancet. 2004;364:13211328. Steinsapir KD, Goldberg RA. Traumatic optic neuropathy: a critical update. Compr Ophthalmol Update. 2005; 6(1):11-21. Steinsapir KD, Goldberg RA, Sinha S, Hovda DA. Methylprednisolone exacerbates axonal loss following optic nerve trauma in rats. Restor Neurol Neurosci. 2000;17:157-163. Volpe NJ. Comments on traumatic optic neuropathy: a critical update. Compr Ophthalmol Update. 2005;6(1):23.

11 QUESTION

WHAT IS THE EVALUATION AND MANAGEMENT FOR PAPILLEDEMA? Michael Wall, MD The patient is a 24-year-old obese woman with visual acuity 20/20 OU, perimetry showing only enlarged blind spots, and bilateral papilledema. How do I establish the diagnosis of idiopathic intracranial hypertension (IIH)? What treatments should be started, if any? How often should I see this patient? Should I refer her to a neurologist? I would recommend that the modified Dandy criteria should be met for the diagnosis of pseudotumor cerebri or IIH (Table 11-1). The patient should first demonstrate the signs and symptoms of increased intracranial pressure. Headache, transient visual obscurations, pulse-synchronous tinnitus, papilledema with associated visual loss, and diplopia due to sixth nerve palsies are the common presenting symptoms and signs. An especially important symptom is pulse-synchronous tinnitus. This bruit-like sound is present in two-thirds of patients and, unlike headache, is rare in the general population. Next, there should be an absence of localized findings on neurological examination except for the “false localizing” sixth nerve palsies. Neuroimaging should reveal the absence of deformity, displacement, or obstruction of the ventricular system. I like to use magnetic resonance imaging (MRI) and magnetic resonance venography (MRV) to look for additional support for the presence of intracranial hypertension. Findings characteristic of increased intracranial pressure on MRI are empty sella, smooth-walled venous stenoses of the lateral sinuses (on MRV), orbital findings related to the unfolding of the nerve sheath, and enhancement of the optic disc. The presence of venous sinus collapse (venous stenosis) is especially important, occurring in over 90% of cases when proper MRV is done; this can be confused with venous sinus thrombosis. Other neurodiagnostic studies should be normal except for increased cerebrospinal fluid pressure. The patient should be awake and alert and no other cause of increased intracranial pressure should be present. In the literature, many conditions are purportedly associated with IIH, but case-control studies show that many are simply chance associations because of their frequent occurrence in the population that is susceptible to IIH (young women in their childbearing years). For example, pregnancy, irregular menses, and oral contraceptive use are reported associations that have been shown to be due to chance alone. In case-control studies, no significant association is found between IIH and the use of multivitamins, oral contraceptives, or antibiotics. A case-control study has found strong

41

Lee AG, ed. Curbside Consultation in Neuro-Ophthalmology: 49 Clinical Questions, Second Edition (pp 41-43). © 2015 SLACK Incorporated.

42  Question 11

Table 11-1

Criteria for the Diagnosis of Idiopathic Intracranial Hypertensiona ●

Signs and symptoms of increased intracranial pressure



Absence of localizing findings on neurological examination



a

Absence of deformity, displacement, or obstruction of the ventricular system and otherwise normal neurodiagnostic studies except for increased cerebrospinal fluid pressure



Awake and alert patient



No other cause of increased intracranial pressure present

Modified Dandy criteria.

associations between IIH, obesity, and weight gain during the 12 months before diagnosis. The interested reader can review a critically evaluated list of conditions and associations of intracranial hypertension in Neurological Therapeutics: Principles and Practice by Noseworthy. Your patient sounds like the typical IIH phenotype and is of the characteristic age. We should proceed to see whether she fulfills the modified Dandy criteria for IIH. If she has typical symptoms of increased intracranial pressure, clear findings of obvious papilledema, MRI with characteristic abnormalities, normal cerebrospinal fluid pressure except for a high opening pressure, and no other cause of intracranial hypertension apparent, I would not go further, since there is little uncertainty. However, with the presence of diagnostic uncertainty (eg, if the patient is male), more testing (eg, MRV or an evaluation for sleep apnea) is performed. Let us presume that this is a typical presentation for IIH. I generally treat all patients with a low-sodium weight management program. Patients usually improve with weight loss of 5% to 10% body weight. Much more weight loss is usually not sustainable and does not appear to be necessary for IIH remission. Weight loss may be all our patient needs, since there is no visual loss with perimetric examination except for enlarged blind spots. However, if her symptoms of intracranial hypertension (eg, severe headache) were interfering with her activities of daily living, I would start acetazolamide in 2 divided doses, then gradually escalating doses to 1 to 2 g per day. Lasix can be used if the patient cannot tolerate acetazolamide. Topiramate does not appear to be any more efficacious than acetazolamide. Until recently, all reports of treatment of IIH have been anecdotal and uncontrolled. However, the results of the Idiopathic Intracranial Hypertension Treatment Trial, which compared acetazolamide-plus-diet with acetazolamide-plus-diet, showed the acetazolamide treated group had significant clinical improvement in visual field function, CSF pressure, papilledema grade, and quality of life measures. The frequency of follow-up depends mostly on the risk of visual loss. The 2 most important factors here are amount of visual loss present and the degree of optic disc edema. If the risk of further visual loss is low; the usual revisit time after diagnosis is 1 to 2 months. I would have this patient back in 2 months and, if she is doing well, again in 4 months. Since IIH can be a lifelong disease (like arterial hypertension), I follow patients who are in remission every 1 to 2 years. The key features in following a patient are the change in weight, change in symptoms, perimetry results, and papilledema grade (Figure 11-1). I also grade optic discs using the Frisén scale—a

What Is the Evaluation and Management for Papilledema?  43 Figure 11-1. An example of Frisén grades for optic disc edema. (A) Note the C-shaped halo with a temporal gap in Grade 1 disc edema. (B) There is Grade 2 (360 degrees) disc edema. (C) In Grade 3 disc edema, there is obscuration of the peripapillary major blood vessel as it crosses the disc margin. (D) In Grade 4 optic disc edema, there is obscuration of the major blood vessel in the center of the disc.

monocular-based graded staging scheme that is descriptive in nature—and take optic disc photos when changes occur. Diagnosis and treatment should be done either by an ophthalmologist and neurologist working together or ideally by a neuro-ophthalmologist. Complicated cases should be managed by a neuro-ophthalmologist.

Summary ●







IIH is a diagnosis of exclusion (modified Dandy criteria). The typical patient with IIH is an overweight young female; more aggressive evaluation for possible etiologies should be considered in thin patients, men, and the elderly. Medical treatment with weight loss and acetazolamide are the first lines of therapy. Surgical treatment (optic nerve sheath fenestration or shunting procedures) is reserved for patients with progressive disease who fail maximal medical therapy.

Bibliography Farb RI, Vanek I, Scott JN, et al. Idiopathic intracranial hypertension: the prevalence and morphology of sinovenous stenosis. Neurology. 2003;60:1418-1424. Giuseffi V, Wall M, Siegel PZ, Rojas PB. Symptoms and disease associations in idiopathic intracranial hypertension (pseudotumor cerebri): a case-control study. Neurology. 1991;41:239-244. Wall M. Papilledema and idiopathic intracranial hypertension (pseudotumor cerebri). In: Noseworthy JH, ed. Neurological Therapeutics: Principles and Practice. Abingdon, UK: Informa Healthcare; 2006:1955-1968. Wall M, McDermott MP, Kieburtz KD, et al, for the IIHTT Study Group. Effect of acetazolamide on visual function in patients with idiopathic intracranial hypertension and mild visual loss: the idiopathic intracranial hypertension treatment trial. JAMA. 2014;311:1641-1651. doi: 10.1001/jama.2014.3312 PMID: 24756514.

12 QUESTION

IS THERE A DIFFERENCE IN THE MANAGEMENT OF PSEUDOTUMOR CEREBRI IN PREGNANCY? Kathleen B. Digre, MD A 23-year-old obese woman was diagnosed with pseudotumor cerebri (PTC). She has been stable taking acetazolamide but she now calls you and says, “I found out I am 8 weeks pregnant.” What would you advise as to her treatment? PTC is a syndrome of increased intracranial pressure and can be either (1) primary, or idiopathic, intracranial hypertension (IIH) or (2) secondary intracranial hypertension due to a secondary cause. Because IIH is most common in obese women of childbearing age, it will be seen in pregnancy. Early myths regarding IIH included that pregnancy was a risk factor for developing IIH, miscarriages were more common with IIH, pregnancy termination improved IIH outcomes, and there was no safe treatment for the condition in pregnancy. All of these have been dismissed. That said, there are many differences in the diagnosis and management of IIH in pregnancy.

Evaluation and Diagnosis Although IIH is the most common cause of intracranial hypertension, secondary causes—especially venous sinus thrombosis—must be excluded. Pregnancy itself is a hypercoagulable state, and any genetic or predisposing condition to clot can manifest itself in pregnancy. It is thus especially important that imaging, including imaging of the cerebral veins, be done. We usually recommend magnetic resonance imaging (MRI) and magnetic resonance venography (MRV). If there is venous thrombosis, it should be treated with heparin and heparinoids (eg, Lovenox [enoxaparin sodium], FDA category B). What is not different in pregnancy is the requirement of a lumbar puncture to make the diagnosis. Pregnancy is not a contraindication to this procedure. The cerebrospinal fluid (CSF) pressure should be over 250 mm H 2O and the fluid must be normal (no elevated protein or cells). Importantly, an examination of the patient’s vision—including visual acuity, visual fields (formal automated perimetry), dilated examination, and photographs—should be performed.

45

Lee AG, ed. Curbside Consultation in Neuro-Ophthalmology: 49 Clinical Questions, Second Edition (pp 45-47). © 2015 SLACK Incorporated.

46  Question 12 Once the diagnosis has been made (normal imaging, elevated CSF pressure, and normal CSF constituents), follow-up should be much as in the nonpregnant state. If there are few symptoms and no visual field defects, then the patient may be followed for the development of changes. If, however, there are symptoms such as headache, transient visual obscurations, or diplopia, treatment options in pregnancy must be considered.

Treatment We always encourage weight loss in the nonpregnant state, but in pregnancy we recommend modest weight gain. Working with an obstetrician can be very helpful. The mainstay of therapy for PTC has been diuretics, especially acetazolamide. For many years, providers avoided this medication in pregnancy for fear of teratogenicity; it is classified in pregnancy FDA category C. Two studies have shown that it may be relatively safe for use in pregnancy even in the first trimester. We recommend a total dose of 1000 mg for most individuals in divided doses (500 mg twice daily). Sometimes this medication is not well tolerated and alternative medications may be sought, such as methazolamide (FDA category C), furosemide (FDA category C), and chlorthalidone (FDA category B). Some diuretics, such as thiazides (FDA category D), are best avoided. If vision is threatened and visual fields are deteriorating, we recommend fenestration of the optic nerve sheath. This procedure can be done under local anesthesia and is safe in pregnancy as long as the patient is positioned in lateral tilt. Shunting (ventriculoperitoneal and lumbar peritoneal), while done frequently in the nonpregnant state, poses obvious problems in the pregnant woman, but it can be done successfully. If vision is severely affected (eg, what we call aggressive or malignant IIH), a lumbar drain can be done as well.

Treatment of the Headache The headache associated with PTC can be difficult to manage in pregnancy, since we like to minimize medication exposures. Nonsteroidal anti-inflammatory drugs can be used in early pregnancy for symptomatic relief. These are generally FDA category B drugs; however, in late pregnancy, because of fetal premature closure of the ductus arteriosus and decreased amniotic fluid production, we tend to avoid them. Medications that can be used to treat headaches in pregnancy include most of the tricyclic antidepressants. Selective serotonin reuptake inhibitors are valuable when there is comorbid depression. We avoid anticonvulsants such as topiramate and valproate (FDA category D) early in pregnancy because these are associated with an increased risk of fetal malformations (neural tube defects).

Delivery Considerations Although CSF pressure is increased during the second stage of labor (pushing), most women who have normal visual function will do fine when experiencing a normal labor. If there is a concern for increasing pressure or if the woman has borderline visual function, consideration can be given to the use of low-outlet forceps. Spinal anesthesia has been used in PTC without complications.

Is There a Difference in the Management of Pseudotumor Cerebri in Pregnancy?  47

Visual Outcome If these patients are treated and followed appropriately, their visual outcomes should be the same as for nonpregnant women.

What to Tell the Patient “We will follow your condition in pregnancy as if you weren’t pregnant, but we know there are some differences. Future pregnancies will not necessarily be complicated by PTC.” Our advice is to partner with an obstetrician who has heard of IIH and work on the patient’s behalf for optimal care of her condition.

Summary ●





In pregnancy, it is important to make the correct diagnosis of IIH, including adequate imaging (venogram), lumbar puncture with fluid analysis, and opening pressure. Follow visual fields carefully, Use acetazolamide to treat IIH when possible. If vision is threatened, consider an optic nerve sheath fenestration.

Bibliography Digre KB. Headaches during pregnancy. Clin Obstet Gynecol. 2013;56(2):317-329. Digre KB, Varner MW, Corbett JJ. Pseudotumor cerebri and pregnancy. Neurology. 1984;34(6):721-729. Falardeau J, Lobb BM, Golden S, Maxfield SD, Tanne E. The use of acetazolamide during pregnancy in intracranial hypertension patients. J Neuroophthalmol. 2013;33(1):9-12. Golan S, Maslovitz S, Kupferminc MJ, Kesler A. Management and outcome of consecutive pregnancies complicated by idiopathic intracranial hypertension. Isr Med Assoc J. 2013;15(4):160-163. Huna-Baron R, Kupersmith MJ. Idiopathic intracranial hypertension in pregnancy. J Neurol. 2002;249(8):1078-1081. Karmaniolou I, Petropoulos G, Theodoraki K. Management of idiopathic intracranial hypertension in parturients: anesthetic considerations. Can J Anaesth. 2011;58(7):650-657. Lee AG, Pless M, Falardeau J, Capozzoli T, Wall M, Kardon RH. The use of acetazolamide in idiopathic intracranial hypertension during pregnancy. Am J Ophthalmol. 2005;139(5):855-859. Shapiro S, Yee R, Brown H. Surgical management of pseudotumor cerebri in pregnancy: case report. Neurosurgery. 1995;37(4):829-831.

13 QUESTION

WHAT IS THE EVALUATION AND MANAGEMENT OF LOW- AND HIGH-FLOW CAROTID CAVERNOUS FISTULAS? Victoria S. Pelak, MD; Emily M. Bratton, MD; and James A. Dixon, MD

A 21-year-old man sustained a closed head trauma in a motor vehicle accident. Over the ensuing week, he developed increasing right proptosis with chemosis, lid edema, limited movement of the right eye in all directions, and a “roaring” sound in his head. What is the most likely diagnosis and how should the patient be evaluated and treated? A 76-year-old man presented 1.5 months after syncopal episode with double vision and pain in the left neck and maxillary region. Examination revealed left eye sixth nerve palsy and mild proptosis of the left eye. CT scan of the brain and orbits revealed enlarged left superior ophthalmic vein. What is the most likely diagnosis and how should the patient be evaluated and treated? There are several features of this clinical presentation that should alert you to a possible direct carotid cavernous fistula (CCF). First, your patient is presenting with orbital signs, which—in conjunction with chemosis and periorbital soft tissue edema—are suggestive of orbital congestion. Second, your patient is also complaining of a “roaring” sound in his head, which is concerning for a high-flow vascular abnormality. Last, there is a history of recent head injury, which is a common risk factor for the formation of a direct CCF and, less frequently, an indirect CCF. Carotid cavernous sinus fistulas represent an abnormal anastomosis between the high-flow arterial system of the carotid arteries and the relatively low-flow venous system of the cavernous sinus. They are classified based on etiology (traumatic or spontaneous), rate of flow (high or low), and angiographic findings (direct or indirect). The Barrow classification, based on the architecture of the fistula, is most commonly used to describe CCFs (Table 13-1). A direct CCF, as its name suggests, is a direct connection between the intracavernous internal carotid artery and the cavernous sinus. The most common cause of direct CCF is a traumatic tear in the wall of the intracavernous carotid artery in the setting of head trauma (approximately 75% of all CCFs), typically occurring in young males (Figures 13-1 and 13-2). Rarely, direct CCFs can

49

Lee AG, ed. Curbside Consultation in Neuro-Ophthalmology: 49 Clinical Questions, Second Edition (pp 49-54). © 2015 SLACK Incorporated.

50  Question 13

Table 13-1

Barrow Classification of Carotid Cavernous Fistulas Type

Direct vs Indirect

Description

A

Direct

Communication between the ICA and the cavernous sinus

B

Indirect

Supply by dural branches of the ICA

C

Indirect

Supply by dural branches of the ECA

D

Indirect

Supply by dural branches of the ICA and ECA

ICA = internal carotid artery; ECA = external carotid artery.

Figure 13-1. Case 1. This 21-yearold man presented 3 days following a motor vehicle accident with near complete ophthalmoplegia and ptosis of the right eyelid due to a high-flow direct CCF. In all positions of gaze, forced duction testing revealed restriction (only right gaze is shown), implicating involvement of the third, fourth, and sixth nerves, in addition to congested extraocular muscles (also seen on MRI of the orbit but not shown here) as the cause of the ophthalmoplegia.

occur from spontaneous rupture of an aneurysm in the presence of underlying collagen vascular disease. Indirect CCFs are generally low-flow shunts formed between the cavernous sinus and dural branches of either the internal or external carotid arteries (Figure 13-3). The symptoms at presentation can vary considerably and depend upon the size of the fistula, duration of the lesion, and direction of venous drainage. Fistulas may drain either anteriorly into the ophthalmic veins or posteriorly into the petrosal sinuses. It is important to be aware that fistulas that drain posteriorly may not present with classic findings. Morbidity from CCFs is caused by both venous congestion and a lack of blood flow to ocular structures as a result of shunting.

Direct Carotid Cavernous Fistula Direct CCFs typically present acutely, usually within days to weeks of an inciting trauma. The classic triad at presentation consists of pulsatile exophthalmos, cephalic bruit, and conjunctival congestion. In addition to the classic triad, external examination may show eyelid edema, dilated conjunctival vessels extending to the limbus, exposure keratopathy, and limited extraocular

What Is the Evaluation and Management of Carotid Cavernous Fistulas?  51

A

B

Figure 13-2. (A) Lateral view of the right carotid angiogram of the patient shown in Figure 13-1. The high-flow CCF can be seen, with flow from the cavernous carotid directly into the superior ophthalmic vein (open black arrow) and inferior ophthalmic vein (solid black arrow). Flow during the arterial phase continues from the orbital veins to the facial vein (small arrow) and pterygoid plexus (bent arrow). (B) The same patient following embolization of the CCF with coiling (black arrow). No flow into the ophthalmic or facial veins or pterygoid plexus during arterial phase is seen, and the carotid artery remains patent with improved flow into the cerebral branches.

Figure 13-3. Left indirect cavernous carotid fistula precoiling (A) and postcoiling (B) coils present in the left cavernous sinus (arrow).

movements (see Figure 13-1). Rapidly progressive proptosis over the course of hours is unusual and may represent thrombosis of the orbital venous outflow system, which requires urgent evaluation and intervention. Dilated funduscopic examination can demonstrate intraretinal hemorrhages, vitreous hemorrhage, dilated retinal veins, optic disc swelling, proliferative retinopathy, or central retinal vein occlusion. Intraocular pressure can be elevated, as both neovascular glaucoma and angle-closure glaucoma (secondary to choroidal congestion and anterior displacement of the lensiris diaphragm) have been reported. Vision loss at presentation occurs in approximately 20% to 35% of patients and may result from any of the above processes, including ischemia of the optic nerve. One of the most devastating complications of direct CCFs is hemorrhage. Both intracranial

52  Question 13 hemorrhage (5% of patients) and severe epistaxis (1% to 2% of patients) have been reported in the literature, including cases resulting in death. For this reason, we always ask patients about neurological symptoms related to these lesions and perform a full neurological examination immediately.

Indirect Carotid Cavernous Fistula Indirect, or low-flow, fistulas present with less obvious signs progressing over weeks to months in comparison with high-flow fistulas. These fistulas typically occur spontaneously but have also been linked to pregnancy, sinusitis, trauma, and cavernous sinus thrombosis. Patients typically present with chronic red eyes and conjunctival injection and are less likely to have a cephalic bruit and noticeable exophthalmos on initial presentation. Indirect CCFs are more likely to resolve spontaneously than direct CCFs. Ophthalmic and neurologic manifestations are typically treated conservatively; however, interventional treatment is necessary in cases of vision loss and intractable symptoms. Conservative treatment includes prism correction for diplopia, topical pressurelowering agents for elevated intraocular pressure, aggressive lubrication for exposure keratopathy, and/or systemic steroids if needed. An example of intervention beyond conservative management is presented next. A 76-year-old man presented with three syncopal episodes accompanied by falling and loss of consciousness, requiring a pacemaker for bradycardia. Approximately 1.5 months following these episodes, he presented with pain in the left aspect of the neck and maxillary region as well as horizontal binocular diplopia. Upon examination, his visual acuity was 20/20 bilaterally with normal intraocular pressure, color vision, and pupillary examination. However, external examination and Hertel exophthalmometry revealed 2 mm of exophthalmos on the left as well as cranial nerve VI palsy of the left eye. CT scan of the brain and orbits without contrast revealed enlargement of the left superior ophthalmic vein (SOV) and a normal cavernous sinus. MRI could not be obtained because of the patient’s pacemaker. In addition, the patient had a history of chronic kidney disease. Therefore CT angiography was bypassed as the initial method of vascular imaging; instead, a transfemoral cerebral angiogram was obtained. Cerebral angiography revealed an indirect CCF, Barrow type D, of the left internal and external carotid artery branches on the left (Figure 13-3A), in which venous drainage was seen mainly through the SOV. He underwent coil embolization of the indirect CCF (Figure 13-3B) by the neurointerventional team via a transvenous approach and tolerated the procedure well. At his 1-month postoperative visit, he had improvement of extraocular motility and no longer had double vision. In this case of an indirect CCF, owing to persistent diplopia, treatment was elected and relieved his visual symptoms. In our opinion, both CT and MRI scans of the orbits are useful initial screening examinations. Both studies can demonstrate findings suggestive of a CCF or alternative causes for the patient’s presentation, although MRI is more useful in identifying the degree of associated cerebral parenchymal injury. Radiographic findings in the setting of CCF may include orbital edema, enlargement of the ipsilateral cavernous sinus, proptosis, and enlargement of the ipsilateral superior ophthalmic vein. As a general rule, an asymmetrical SOV or a vein larger than 4 mm probably represents pathological enlargement; MRI of the orbit is best to accurately measure vein size. It is important to note that an asymmetrical superior ophthalmic vein is not a specific finding and can be seen in other orbital pathologies, such as vascular malformations. However, with an appropriate history, an enlarged SOV is highly suggestive of an indirect or direct CCF. As mentioned previously, some fistulas primarily drain posteriorly into the petrosal sinuses; in these instances, the SOV may not be enlarged. If the diagnosis is still uncertain or the patient’s symptoms do not warrant intervention, we typically perform a CT angiogram (CTA) to help confirm the diagnosis

What Is the Evaluation and Management of Carotid Cavernous Fistulas?  53 and define the anatomy. Although CTA is less invasive than digital subtraction cerebral angiography (DSA) and may provide detail about the size and location of the fistula, DSA remains the gold standard for the diagnosis and confirmation of obliteration of the CCF. MR angiography (MRA) is another noninvasive technique for assessing CCF, although CTA is generally better at characterizing a fistula’s anatomy than MRA, especially segment 4 of the internal carotid artery, which is the location of approximately half of all direct CCFs. Therefore, if the patient’s symptoms warrant intervention, catheter angiography should be performed. Although CCFs may close spontaneously or after cerebral angiography, some fistulas require embolization. The goal of treatment is to close the fistula while maintaining patency of the internal carotid artery. Embolization is achieved utilizing detachable balloons and coils or liquid embolic agents. The reported success rate for embolization of direct fistulas ranges from 89% to 99%; for indirect fistulas, the range is from 70% to 78%. The surgeon typically takes either a transarterial or transvenous approach to the cavernous sinus. Transvenous approaches are typically via the inferior petrosal sinus; however, alternative routes include the pterygoid venous plexus, superior petrosal sinus, facial veins, or SOV. The advantage of catheterization of the SOV through an anterior orbitotomy approach has been highlighted as a safe and effective mode to gain access to the cavernous sinus. More recently, approaching the cavernous sinus through a thrombosed SOV has been proven to be safe and effective, limiting the morbidity of other less direct routes. In addition, Gamma Knife radiotherapy has shown promise as an alternative treatment for low-flow fistulas. If you suspect that your patient has a direct or indirect CCF, further imaging will lead to proper assessment and treatment of the condition. Early recognition is key to preventing ophthalmic and neurological morbidity and, in rare instances, mortality.

Summary ●

















CCFs typically occur after trauma in young males. CCFs are classified based on etiology (traumatic or spontaneous), rate of flow (high or low), and angiographic findings (direct or indirect). The classic triad of clinical signs at presentation of a direct CCF is pulsatile exophthalmos, cephalic bruit, and conjunctival congestion. Complete neurologic examination should be completed in all patients with direct CCFs as intracranial hemorrhage is a devastating complication. Indirect CCFs present with more subtle, chronic changes such as conjunctival injection and/ or diplopia. CT and MRI of the orbits are good screening tools and typically show enlargement of the superior ophthalmic vein. CT angiogram can confirm the diagnosis and define the anatomy of the fistula. Digital subtraction cerebral angiography is the gold standard to confirm diagnosis and obliteration of the CCF after embolization. When treatment is necessary, embolization of the fistula by various methods and routes has a high success rate.

54  Question 13

Bibliography Barrow DL, Spector RH, Braun IF, et al. Classification and treatment of spontaneous carotid-cavernous fistulas. J Neurosurg. 1985:62:248-256. Chen CC, Chang PC, Shy CG, Chen WS, Hung HC. CT angiography and MR angiography in the evaluation of carotid cavernous sinus fistula prior to embolization: a comparison of techniques. Am J Neuroradiol. 2005;26:2349-2356. Fleishman JA, Garfinkel RA, Beck RW. Advances in the treatment of carotid cavernous fistula. Int Ophthalmol Clin.1986;26:301-311. Gemmete JJ, Ansari SA, Gandhi DM. Endovascular techniques for treatment of carotid-cavernous fistula. J NeuroOphthalmol. 2009;29:62-71. Gupta AK, Purkayastha S, Krishnamoorthy T, et al. Endovascular treatment of direct carotid cavernous fistulae: a pictorial review. Neuroradiology. 2006;48:831-839. Harris GJ, Rice PR. Angle closure in carotid-cavernous fistula. Ophthalmology. 1979;86:1521-1529. Jung HH, Chang JH, Whang K, Pyen JS, Chang JW, Park YG. Gamma knife surgery for low-flow cavernous sinus dural arteriovenous fistulas. J Neurosurg. 2010;113(Suppl):21-27. Sugar HS. Neovascular glaucoma after carotid-cavernous fistula formation. Ann Ophthalmol. 1979;11:1667-1669. Yilmaz G, Yazici B, Cetinkaya A, Yagci A. Embolization of dural carotid-cavernous fistulas via the thrombosed superior ophthalmic vein. Ophthalmol Plast Reconstr Surg. 2013; 29: 272-276.

14 QUESTION

HOW DO I WORK UP AND MANAGE AN INTERNUCLEAR OPHTHALMOPLEGIA? Christopher C. Glisson, DO, MS and David I. Kaufman, DO

A 45-year-old patient has double vision. Examination reveals impairment of adduction in the left eye with nystagmus in the right eye on right lateral gaze. How is internuclear ophthalmoplegia (INO) proven? Can it be anything else? What is the workup for an INO? These examination findings describe the most common presentation of internuclear ophthalmoplegia (INO)—impaired adduction of one eye associated with monocular nystagmus in the contralateral (abducting) eye. However, accurate localization (and ultimately accurate diagnosis) requires careful consideration of the patient’s history, followed by a detailed examination to document the presence (or absence) of potential associated features. Using this systematic approach, the lesion can be exquisitely localized, alternate etiologies (or “mimics”) can be considered, and appropriate diagnostic testing can be obtained for confirmation of the underlying cause. Patients with an isolated INO may be asymptomatic. Alternatively, asymmetry in the speed of horizontal saccades between the abducting and adducting eye can give the brief sensation of disparate images that patients perceive as “having to catch up to each other.” Still other patients may report “double vision” in the horizontal plane. Overall, INO can precipitate a continuum of visual disturbances—ranging from blurring to overt diplopia—as a consequence of adduction weakness and ocular misalignment. Oscillopsia (the perception of movement of the visual world) of one of the images as a consequence of abduction nystagmus may also be described. On physical examination, the presence of an INO is suggested by unilateral adduction weakness in conjunction with abduction nystagmus in the fellow eye. These findings are best elicited by testing saccades rather than smooth pursuit movements, as the medial longitudinal fasciculus (MLF) mainly carries fibers relaying the “pulse” of the saccade rather than the “step” of smooth pursuit. Asking the patient to perform large-amplitude saccades, sometimes between random targets, can also increase the yield of testing for an INO by amplifying the adduction weakness of the ipsilateral eye.

55

Lee AG, ed. Curbside Consultation in Neuro-Ophthalmology: 49 Clinical Questions, Second Edition (pp 55-57). © 2015 SLACK Incorporated.

56  Question 14

Table 14-1

The Most Common Causes of Internuclear Ophthalmoplegia Older Than Age 45 Years

Younger Than Age 45 Years

Ischemia

Multiple sclerosis

Multiple sclerosis

Trauma

Tumor (primary or metastatic)

Tumor (primary or metastatic)

Myasthenia gravis ( pseudo-INO )

Ischemia Myasthenia gravis ( pseudo-INO )

Adapted from Lavin PJM, Donahue SP. Neuro-ophthalmology: ocular motor system. In: Bradley WG, Daroff RJ, Fenichel GM, Jankovic J, eds. Neurology in clinical practice: principles of diagnosis and management. 4th ed. Philadelphia: Elsevier; 2004:718; and Keane JR. Internuclear ophthalmoplegia: unusual cases in 114 of 410 patients. Arch Neurol. 2005;62:714-717.

The hallmark of INO is “adduction lag” of the eye ipsilateral to the site of the lesion. In addition to careful testing of saccades, identification of a subtle adduction lag can also be enhanced through the use of an optokinetic drum. This allows the amplitude and velocity of saccades in each eye to be compared, thus emphasizing the slowed adduction of the affected eye. Other examination features may include skew deviation (usually with hypertropia ipsilateral to the adduction lag), impairment of the vertical vestibulo-ocular reflex, and impaired vertical smooth pursuit. Neuroanatomically, INO results from a lesion involving the MLF, a paired fiber tract that transmits neural impulses from the sixth nerve nucleus to the contralateral third nerve nucleus. Disruption of the MLF (by any mechanism, see below) impairs the pathway, mediating conjugate eye movements within the horizontal plane by effectively limiting input to the medial rectus muscle ipsilateral to the lesion. The contralateral eye develops nystagmus in abduction. This nystagmus is likely a reflection of Hering’s law, with equal innervation to the yoked horizontal eye muscles in the setting of impaired adduction but full and unrestricted abduction. Numerous causes of INO have been reported in the literature. However, in practice, the most common causes are few in number (Table 14-1). If the INO is an isolated finding, the primary differential diagnosis should be formulated based on the patient’s age. In patients between age 18 and 45 years, demyelination (usually as a consequence of multiple sclerosis [MS]) is the most frequent cause; in patients older than age 45 years, the most likely causes are a small vessel stroke (although rarely vertebral artery dissection or other large artery lesion) followed by demyelination. INO may be unilateral or bilateral, reflecting involvement of one or both MLFs. A bilateral INO is most classically associated with MS in the appropriate age group. However, systemic processes such as metabolic and nutritional derangements can result in bilateral MLF involvement. Wall-eyed binocular INO (WEBINO) syndrome results from bilateral MLF lesions, with the associated finding of exotropia in primary position. However, unilaterality or bilaterality of INO is not consistent among etiologies and thus should not be used as a definitive diagnostic feature. Specific variations of INO, however, can assist with localization. Examples include the one-anda-half syndrome (a lesion of the caudal dorsal pontine tegmentum and ipsilateral MLF, with only

How Do I Work Up and Manage an Internuclear Ophthalmoplegia?  57 abduction of one eye remaining intact) and the eight-and-a-half syndrome (one-and-a-half plus cranial nerve VII palsy, often due to pontine ischemia). Patients with INO should be evaluated based on their clinical history and examination, and subsequent diagnostic investigations should be based on accurate clinical localization of the presumptive lesion causing the adduction deficit. In most instances, thin-slice (2 to 3 mm) magnetic resonance imaging (MRI) of the brain with gadolinium is required. Confirmatory findings at imaging should include involvement of the dorsal aspect of the caudal pons, ipsilateral to the impaired adduction. It should be noted that an abnormality is not always documented even on thin-slice MRI. Despite this occasional limitation, however, MRI allows the opportunity to investigate for MS, acute stroke, and other unexpected causes, such as an orbital or third nerve mass or thyroid eye disease. In the appropriate clinical setting, further studies directed toward alternative causes of INO (see Table 14-1) should be undertaken. If pseudo-INO from ocular myasthenia gravis is suspected, a rest test can be performed. If there is improvement in ocular motility after 10 to 15 minutes of rest, the observed motility deficit is likely due to myasthenia gravis. The use of ice in conjunction with the rest test can also assist in differentiating myasthenia gravis from the other more common causes of INO. Prostigmin and Tensilon (Valeant Pharmaceuticals International) could also be considered. Thyroid eye disease is often associated with proptosis or lid lag. A partial third nerve palsy due to a compressive lesion may be associated with aberrant regeneration involving the pupil or eyelid. Finally, the presence of an INO in an individual younger than age 45 years calls for the consideration of possible MS and the pursuit of that diagnosis with MRI and other appropriate studies to determine if immunomodulating agents are indicated. For those older than age 45 years, risk factors for stroke should be reviewed and evaluated, with appropriate therapy provided in confirmed cases.

Summary ●

INO results from a lesion of the ipsilateral medial longitudinal fasciculus.



Patients with INO may be asymptomatic or may describe blurred vision or diplopia.



INO is recognized clinically by adduction lag in one eye, which is classically associated with abduction nystagmus in the fellow eye.



In patients aged 18 to 45 years, demyelination is the most common cause of INO.



In patients older than age 45, stroke and MS are the most common causes of INO.



Myasthenia gravis can mimic INO (“pseudo-INO”).



MRI is recommended to evaluate for the causes (and other mimics) of INO, including MS, stroke, mass lesion, and thyroid eye disease.

Bibliography Eggenberger E. Eight-and-a-half-syndrome: one-and-a-half plus cranial nerve VII palsy. J Neuroophthalmol. 1998; 18(2):114-116. Keane JR. Internuclear ophthalmoplegia: unusual cases in 114 of 410 patients. Arch Neurol. 2005;62:714-717. Kubis KC, Danesh-Meyer HV, Savino PJ, Sergott RC. The ice test vs the rest test in myasthenia gravis. Ophthalmology. 2000;107(11):1995-1998. Leigh RJ, Zee DS. The Neurology of Eye Movements. 4th ed. New York: Oxford University Press, 2006. Wall M, Wray SH. The one-and-a-half syndrome: a unilateral disorder of the pontine tegmentum. Neurology. 1983;33:971.

15 QUESTION

WHAT IS THE WORKUP AND TREATMENT OF MYASTHENIA GRAVIS? Leah Levi, MBBS A 68-year-old woman presents with complaints of droopy upper lids over the past few weeks. Sometimes one lid or the other might even close completely. She has also noticed double vision, which comes and goes. Since the majority of patients with myasthenia gravis present with involvement of the lids and extraocular muscles, the ophthalmologist is often the first physician to make this diagnosis, usually based on key history and clinical findings in the office. These reveal a typically variable clinical picture that is modulated by rest and fatigue; they also help establish localization distal to the brainstem and cranial nerves. If I think that the patient has myasthenia gravis, an additional issue is the timing of referral to a neuromuscular specialist.

Key History There are some key questions that address the typical characteristics of myasthenia—namely improvement with rest, fatigability, and variability. The one question that I find most useful, and that addresses the effect of rest, is whether the ptosis or diplopia is present upon first awakening. Most patients with myasthenic ptosis report that the lids are virtually normal or much improved first thing after sleep. Similarly, a report by a patient with diplopia of little or no double vision upon awakening strongly suggests myasthenia and reflects improvement from a tropia to a phoria during sleep. This is in contrast to other causes of diplopia where overnight ocular dissociation leads to a tropia upon awakening with improvement afterward. Another question, which addresses fatigability, is to ask how long it takes for the ptosis or diplopia to recur after awakening. In most patients with myasthenia gravis, ptosis or diplopia will reappear soon after awakening as the effect of rest rapidly wears off. I do not find a history of

59

Lee AG, ed. Curbside Consultation in Neuro-Ophthalmology: 49 Clinical Questions, Second Edition (pp 59-63). © 2015 SLACK Incorporated.

60  Question 15 worsening ptosis or diplopia toward evening useful since this is not specific for myasthenia gravis. Any cause of ptosis or diplopia will worsen later in the day because the effort to overcome them becomes unsustainable over several hours. It is also useful to ask questions that capture variability, such as whether the ptosis or diplopia fluctuates over relatively short periods of time. Many patients with myasthenia will report that the ptosis changes from hour to hour or migrates from one lid to the other. The diplopia may change from horizontal to vertical over the course of days or weeks. In contrast, a history of gradually worsening ptosis over a long period of time without significant fluctuation or a pattern of diplopia that is unchanging, although not completely ruling out myasthenia gravis, strongly suggests another diagnosis. Likewise, a sudden onset of ptosis, diplopia, or both followed by stability is not characteristic of myasthenia gravis. If you suspect myasthenia, you need to ask questions that will help you time a referral to neuromuscular specialist appropriately. Asking about other weakness, particularly problems swallowing or breathing, will indicate that the patient may already have generalized myasthenia gravis and needs timely referral. If the ocular symptoms are isolated and have been present for at least 2 years, the risk of generalization is low; however, if they have been of more recent onset, timely referral is appropriate.

Key Physical Findings Pupils The pupils are not clinically affected by myasthenia gravis. There should be no pupillary dilation or Horner syndrome.

Ptosis There are several lid findings suggestive of myasthenia gravis. Bilateral ptosis is common in myasthenia but rare in brainstem or cranial nerve disorders. However, the bilateral nature of the ptosis may be masked by the effect of Hering’s law, in which the effort to open the more ptotic lid brings the less ptotic lid into a normal or even a retracted position. In apparently unilateral ptosis, lift the ptotic lid and observe whether the “normal” (or retracted) lid becomes ptotic (Figure 15-1). Observe the patient for 1 minute to see if the ptosis fluctuates—a sign typical of the variability of myasthenia. Look for levator weakness. While controlling for the frontalis by pressure with your thumb above the brow, measure the excursion of the upper lid margin from downgaze to upgaze. If lid excursion is normal (15 mm or more), measure the location of the lid crease to look for levator dehiscence as the cause of the ptosis. Look for the fatigability typical of myasthenia by asking the patient to look up and observe for worsening ptosis after about 30 seconds. Look for a Cogan lid twitch by asking the patient to look down for 30 seconds or so (briefly resting the levator) and then observing the lid as the patient saccades to primary position. Often, the myasthenic lid will overshoot and then come to rest in its ptotic position. Look for improvement of the ptosis with rest or ice. Most myasthenic lids will improve measurably, whereas nonmyasthenic lids will not (Figure 15-2). You can either have the patient close his or her eyes for 10 minutes (rest test) or use ice in a glove on closed lids for 2 to 3 minutes (ice test). The addition of ice improves the result. I find that the rest test alone usually suffices but use ice in equivocal cases. If you decide to do this testing while awaiting dilation, do not use phenylephrine

What Is the Workup and Treatment of Myasthenia Gravis?  61 Figure 15-1. (A) The patient appears to have ptosis of the left upper lid only. (B) Lifting of the ptotic left upper lid reveals that the contralateral lid is also ptotic. The right upper lid ptosis was disguised by the effect of Hering’s law.

A

B

A

B

Figure 15-2. (A) Patient before and (B) after the rest test, with clear improvement of the left-upper-lid ptosis, but also some improvement in the position of the right upper lid, revealing that there was bilateral asymmetrical ptosis.

drops since these improve ptosis independently and confuse the results. The rest or ice test is not as suitable for motility issues since the effect is so short-lived. In most cases the lid response makes the diagnosis.

Motility The presence of bilateral limitation of eye movement is very helpful since in myasthenia both eyes are commonly involved (Figure 15-3), whereas bilateral involvement would be most unlikely in brainstem or cranial nerve lesions. A motility pattern appearing to be an isolated unilateral cranial neuropathy or internuclear ophthalmoplegia may occur in myasthenia. However, it is uncommon for myasthenia gravis to mimic these patterns without other findings, such as ptosis or orbicularis weakness.

Orbicularis You should test orbicularis function in all patients with ptosis or motility problems. The orbicularis is weak in most myasthenic patients (Figure 15-4), often visibly fatiguing with continued testing. The presence of orbicularis weakness in a patient with ptosis and motility abnormalities indicates a localization that cannot be explained by a brainstem or cranial nerve lesion.

62  Question 15

Figure 15-3. The bilateral nature of the motility findings in myasthenia gravis, including an adduction deficit in the left eye and a supraduction deficit with a small infraduction deficit of the right eye. These findings cannot be explained by brainstem or cranial nerve dysfunction. In addition, this patient had bilateral asymmetrical ptosis and bilateral orbicularis weakness (see Figures 15-1 and 15-4), further supporting a more distal localization.

Figure 15-4. Bilateral orbicularis weakness in myasthenia gravis. This is the same patient as in Figures 15-1 and 15-3. All lid and motility findings normalized with pyridostigmine treatment.

Orbit You should look for signs of thyroid orbitopathy. This can produce bilateral motility dysfunction and can coexist with myasthenia gravis in about 5% of patients. You might need to consider forced duction testing if you suspect this.

Management If additional symptoms suggest generalized myasthenia gravis or if the isolated ocular symptoms are of recent onset, you should refer to a neuromuscular specialist in a timely manner. I warn the patient that if problems with breathing or swallowing develop in the meantime, an urgent visit to an emergency department would be in order. With this in mind, I give the patient my card with the diagnosis written on it.

What Is the Workup and Treatment of Myasthenia Gravis?  63 I usually order a chest CT scan and an anti-acetylcholine receptor antibody panel so that these will be available to the neuromuscular specialist. Bear in mind that about 50% of patients with isolated ocular myasthenia do not have anti-acetylcholine receptor antibodies and the diagnosis is made on clinical grounds. I usually defer to the neuromuscular specialist to begin a trial of pyridostigmine, then I see the patient again after a few weeks. If the patient has isolated ocular myasthenia gravis and pyridostigmine has not been fully effective, I add prednisone since this is usually more effective for the ocular signs and may decrease the rate of generalization. I usually start at 1 mg/kg per day for 2 weeks, followed by tapering. This results in remission for some patients, but others need a maintenance alternate-day dose. If a patient’s generalized myasthenia gravis improves, I defer to the neuromuscular specialist regarding further management, which may include immunosuppressive therapy, intravenous immunoglobulin, plasmapheresis, and, in some cases, thymectomy. If possible, I offer symptomatic treatment for the diplopia, aside from simply patching one eye. Even though the diplopia may be variable and incomitant, in some patients the degree and variability is within a range wherein the Fresnel prism can be helpful and provide single binocular vision in primary and reading positions. This is more likely to help if the strabismus is relatively small and without a large vertical component. Despite otherwise effective medical treatment, a few patients develop fixed strabismus and ptosis over time. If these remain stable for at least 1 year, strabismus surgery and ptosis surgery (usually a frontalis sling) can be offered.

Summary Clues that make myasthenia a consideration in a patient with ptosis, diplopia, or both include the following: Effect of rest ●



Symptoms absent upon awakening



Ptosis improves after office rest/ice test



Fatigable ptosis



Variable symptoms and findings



Bilateral findings not mapping to cranial nerves



Orbicularis weakness

Bibliography Kubis KC, Danesh-Meyer HV, Savino PJ, Sergott RC. The ice test versus the rest test in myasthenia gravis. Ophthalmology. 2000;107(11):1995-1998. Kupersmith MJ, Latkany R, Homel P. Development of generalized disease at 2 years in patients with ocular myasthenia gravis. Arch Neurol. 2003;60:243-248.

16 QUESTION

HOW DO YOU MANAGE VISUAL LOSS IN THYROID EYE DISEASE? James A. Garrity, MD A 50-year-old man with thyroid eye disease (TED), diplopia, and proptosis has recently noted progressive visual loss in one eye. The clinical examination suggests optic nerve dysfunction as the cause of diminished vision. How is the diagnosis of thyroid optic neuropathy established? What are the treatment options? How urgently should this patient be evaluated? When does the patient with TED require surgery? What surgeries are warranted and how urgently? TED usually arises in the setting of Graves’ disease with hyperthyroidism. From the ophthalmologist’s perspective, a cost-effective screen for TED can be determined with 3 blood tests: (1) total T4; (2) thyroid-stimulating hormone (TSH); and (3) thyroid-stimulating immunoglobulins (TSI) or TSH-receptor antibodies (TrAb). After discussing results of these tests with an endocrinologist, additional tests can be ordered if needed. I expect that up to 90% of my patients with TED will be hyperthyroid, but patients can be euthyroid or hypothyroid as well. The majority of TED patients in my practice, however, would be expected to have abnormal levels of TSI or TrAb, and a normal TSI or TrAb should be a red flag in patients with a presumed diagnosis of TED. There are many potential causes of visual loss for your patient in the setting of TED. From a practical standpoint, however, there are only 3, and they are related to diplopia (phoria breaking down), surface issues on the cornea, or an optic neuropathy. It is important to differentiate these causes since the treatment of each is vastly different. Patients with incipient diplopia almost always complain of “blurred vision” that improves when they close either eye. The examination typically shows a tropia, and a prism is usually curative. In most cases of cornea-related visual loss, one would expect to find lid retraction with or without lagophthalmos. A more insidious cause is related to stiff eyelids with poor blinks. The eyelids do not close completely with blinks, leaving an interpalpebral zone (which includes the visual axis) of punctate keratopathy. Rose Bengal or fluorescein dye will highlight the area of involvement and lead to the correct diagnosis. Treatment is directed at better lubrication or, in some instances, eyelid surgery.

65

Lee AG, ed. Curbside Consultation in Neuro-Ophthalmology: 49 Clinical Questions, Second Edition (pp 65-67). © 2015 SLACK Incorporated.

66  Question 16 Vision loss related to optic neuropathy is typically insidious; a “sudden” onset is distinctly uncommon and should prompt a search for a different diagnosis. Did your patient suddenly “become aware” of reduced vision in one eye? This occurs typically when the good eye is covered, which may account for some instances of sudden onset visual loss. In the setting of TED optic neuropathy (TEDON), the subjective perception is that the vision appears dim and colors may appear washed out. Objectively, visual acuity may be reduced, often to profound levels. On the other hand, visual acuity may remain normal, with disc edema, visual field loss, or both as the only findings. The examination should record visual acuity and a careful note of the pupillary responses should be made. An afferent pupillary defect may not be present if the optic neuropathy is symmetrical. Color vision is typically impaired. The external examination may be surprising in that, many times, the eyes appear relatively white and quiet. Ocular motility is usually impaired to some degree, which is reflective of the underlying extraocular myopathy ultimately responsible for the optic neuropathy. Exceptionally, an optic neuropathy may not be compressive but rather can occur on the basis of a stretched optic nerve related to excessive proptosis from an expanded orbital fat compartment. As a general rule, patients with optic neuropathy tend to have less proptosis than patients without optic neuropathy, which may reflect a relative physiological “autodecompression” with excess proptosis in the nonoptic neuropathy group. The fundus examination may be entirely normal, but approximately 20% to 30% of optic neuropathy patients have a swollen disc. Choroidal folds may also be present. Visual field examinations tend to show generalized depression, a central defect, or some type of an inferior defect. Any type of a superior defect should prompt a search for a different diagnosis. Imaging can be done with either magnetic resonance imaging (MRI) or computed tomography (CT); however, CT is my preference because bony details are shown. Appreciation of bony details, especially the roof of the ethmoid sinus, is important if orbital decompression is a consideration. I would not typically image the patient if I did not plan a decompression, but I would image if the TED were asymmetrical enough to question the diagnosis or if there were some other atypical feature. The expected finding is apical compression from enlarged extraocular muscles. Iodine contrast is not needed because orbital fat serves as an inherent contrast. Iodine in the contrast would also interfere with the endocrinologist’s treatment of hyperthyroidism for at least 6 weeks. I do not see a role for visual evoked potentials in establishing the diagnosis of TEDON. While this diagnosis was formerly considered an indication for emergent therapy, I am of the opinion that urgent therapy will suffice. When I reviewed all of the charts from our study of 215 patients with TEDON, it was apparent that many had had their disease for many months and, following therapy, their outcome was no different. Therefore, I have changed my approach in that therapy within the following few weeks is sufficient. With an established diagnosis of TEDON, the next question regards the natural history of the disease. Does treatment favorably influence its course? Data are limited, but Trobe et al did show that treatment was more effective than the natural history. Treatment is either nonsurgical or surgical. Nonsurgical therapy is centered on the use of corticosteroids. Steroids are effective, although relapse and side effects limit their use. Comorbid conditions, such as diabetes mellitus, also limit the effectiveness of steroids. Rapamycin and cyclosporine have been utilized, but treatment numbers are small. Radiation therapy also has limited experience. The reports of radiation therapy have the confounding variable of concomitant steroid use. Many patients with TEDON also have diabetes mellitus, which limits the use of radiation therapy because of the concern of radiation retinopathy. My treatment strategy has been to consider orbital decompression (transantral) as first-line therapy, with prolonged postoperative steroids as necessary. Decompression is associated with a more rapid return of visual function—a median of 2.5 weeks in some series. I have used transfrontal decompression as salvage therapy for persistent TEDON. The group from Amsterdam, in a randomized clinical trial, showed that intravenous steroids were a better form of treatment than decompression. However, this trial had small numbers.

How Do You Manage Visual Loss in Thyroid Eye Disease?  67

Summary ●





Patients with optic neuropathy make up a small but important segment of the TED practice. A noncontrast orbital CT scan might show the compressive optic neuropathy in TED and is potentially useful for atypical cases owing to alternative etiologies that might mimic TED (eg, sphenoid wing meningioma). There are many treatment options available for your patient, and individualizing them is appropriate.

Bibliography Fatourechi V, Bartley GB, Garrity JA, et al. Transfrontal orbital decompression after failure of transantral decompression in optic neuropathy of Graves’ disease. Mayo Clin Proc. 1993;68:552-555. Garrity JA, Fatourechi V, Bergstralh EJ, et al. Results of transantral orbital decompression in 428 patients with severe Graves’ ophthalmopathy. Am J Ophthalmol. 1993;116:533-547. Soares-Welch CV, Fatourechi V, Bartley GB, et al. Optic neuropathy of Graves’ disease: results of transantral orbital decompression and long-term follow-up in 215 patients. Am J Ophthalmol. 2003;136:433-441. Trobe JD, Glaser JS, Laflamme P. Dysthyroid optic neuropathy: clinical profile and rationale for management. Arch Ophthalmol. 1978;96:1199-1209. Wakelkamp IMMJ, Baldeschi L, Saeed P, et al. Surgical or medical decompression as a first-line treatment of optic neuropathy in Graves’ ophthalmopathy? A randomized controlled trial. Clin Endocrinol. 2005;63:323-328.

17 QUESTION

WHEN DO YOU USE RADIATION OR STEROIDS IN THYROID EYE DISEASE? Steven E. Feldon, MD, MBA A 65-year-old man with a history of Graves’ disease has vertical diplopia and visual acuity of 20/40 in the right eye and 20/80 in the left eye with a left afferent papillary defect. There is mild bilateral proptosis and moderate upper and lower lid swelling. Corneal exposure from upper lid retraction is present bilaterally. Oral steroids have been minimally effective with exacerbation when attempting to taper drug. What are the treatment alternatives? Thyroid eye disease (TED) is the most common disorder of the orbit. Unlike other causes of orbital inflammation, TED involves extensive remodeling of orbital tissues with enlargement of the extraocular muscles and an increase in the orbital fat compartment. These tissue alterations contribute to the clinical manifestations of TED, including exophthalmos, periorbital edema, chemosis, restricted motility, and optic neuropathy. The manifestations may overlap with signs typical of orbital inflammation, with mixed white cell infiltration and vasogenic interstitial edema. In addition, the closed compartment of the bony orbit may precipitate more inflammation due to ischemia caused by elevated tissue pressure with venous stasis (ie, compartment syndrome). Not surprisingly, with overlapping etiologies for the clinical symptoms and signs of TED and a finite time course for this self-limited disease (ie, Rundle curve), response to anti-inflammatory agents is incomplete and unpredictable (Figure 17-1). Corticosteroids and ionizing radiation work separately and in concert to initiate lympholysis and to reduce vascular permeability. These actions reduce orbital volume, thereby improving venous stasis and often leading to varying degrees of clinical improvement. Corticosteroids and radiation also may interrupt the activation of fibroblasts, which leads them to differentiate into scar-forming myofibroblasts and fat cells. Unfortunately anti-inflammatory treatment does not “undo” orbital tissue remodeling, nor does it remove the presumed antigenic stimulus underlying the disease process. Therefore exacerbation of disease following cessation or completion of treatment is frequent. Such treatment does not reduce the need for reconstructive and rehabilitative surgery to restore function and appearance. Another treatment option for TED is surgical intervention. Orbital decompression can be accomplished through several approaches designed to improve the ratio of orbital tissue volume to bony socket volume. The excision of orbital fat is useful for mild to moderate disease, but expanding the socket by removal of bone from one or more of the orbital walls is optimal for moderate

69

Lee AG, ed. Curbside Consultation in Neuro-Ophthalmology: 49 Clinical Questions, Second Edition (pp 69-71). © 2015 SLACK Incorporated.

70  Question 17 Figure 17-1. Rundle curve.

to severe disease. The mechanical consequences of decompression accomplish many of the goals of nonsurgical management, including a reduction in orbital tissue pressure, which lessens tissue ischemia and its contribution to the inflammatory process (Figure 17-2). Although there are increasing numbers of evidence-based studies evaluating various therapeutic approaches to TED, most are derived from a numerical Clinical Activity Score (CAS), a grading system based on the patient’s symptoms and signs. The components of CAS include retrobulbar pain, pain on attempted up- or downgaze, redness of the eyelids, redness of the conjunctiva, swelling of the eyelids, inflammation of the caruncle and/or plica, and conjunctival edema, with one point given for each component. However, these symptoms and signs may be multifactorial and not directly related to the autoimmune process. Further complicating interpretation of the results of various studies is the fact that follow-up is of limited duration and the natural history of TED is that of a self-limited inflammatory disease. With this background, the use of corticosteroids and radiation can be customized depending on whether the goal is prophylaxis, intervention for worsening disease, or rescue therapy. Low-dose prednisone definitely has a role in the treatment of Graves’ disease prior to I-131 radioablation of the thyroid gland. Bartalena et al have shown that the onset and progression of TED can be significantly reduced using prednisone 20 to 40 mg per day beginning just prior to I-131 treatment and continuing for 4 to 6 weeks. There are no corresponding data for orbital radiation as prophylaxis. In patients presenting with optic neuropathy, high-dose oral prednisone (100 mg per day) or perhaps pulse intravenous methylprednisolone 500 mg weekly for 6 weeks followed by 250 mg for another 6 weeks (total dose 4.5 g) is indicated as that dose minimized liver toxicity in a study by Le Moli. Failure to respond should trigger urgent orbital decompression, especially in diabetic or otherwise steroid-intolerant patients. Patients should be maintained on corticosteroids for 1 to 2 weeks postoperatively and then tapered. Recurrent optic neuropathy after decompression may be managed with further short courses of steroids with or without orbital radiation, which is thought to be ineffective unless there is concurrent steroid therapy. The most controversial use of corticosteroids and orbital radiation involves patients with relatively high CAS but without optic neuropathy. There is a substantial difference of opinion as to whether treatment shortens Rundle curve without effect on the final outcome or whether it flattens the curve, thus blunting the effects of the disease. There is also is a viewpoint suggesting that short-term interventions may prolong the time needed to achieve stability. Unfortunately, no clinical trial has yet been conducted to address this issue. In any case, the structural issues associated

When Do You Use Radiation or Steroids in Thyroid Eye Disease?  71 Figure 17-2. (A) A patient shown prior to bony orbital decompression with severe thyroid eye disease, including optic neuropathy, has several features of active disease, including red edematous lids, conjunctival injection, and redness of the plica and caruncle. (B) In the immediate postoperative period, there is marked reduction in all objective components of activity without additional orbital radiation or systemic corticosteroids.

with the disease may require surgical management. This begs two questions: (1) whether the risks of ionizing radiation to the orbit and systemic corticosteroids actually outweigh perceived benefits and (2) whether surgical orbital decompression should be considered as a treatment intervention in addition to being part of the rehabilitative process.

Summary Controversy remains in determining the appropriate therapy for advanced TED. Patients with TED should be managed following a careful assessment of visual and psychosocial needs. The decision to use steroids alone or in combination with radiotherapy requires close attention to the ratio of risk versus benefit of a self-limited disease, especially when there is a high likelihood that surgical management will be required with or without medical treatment.

Bibliography Bartalena L, Marcocci C, Bogazzi F, et al. Use of corticosteroids to prevent progression of Graves’ ophthalmopathy after radioiodine therapy for hyperthyroidism. N Engl J Med. 1989;321:1349-1352. Le Moli R, Baldeschi L, Saeed P, et al. Determinants of liver damage associated with intravenous methylprednisolone pulse therapy in Graves’ ophthalmopathy. Thyroid. 2007;17:357-362. Wakelkamp I, Baldeschi, L, Saeed P, et al. Surgical or medical decompression as a first-line treatment of optic neuropathy in Graves’ ophthalmopathy? A randomized controlled trial. Clin Endocrinol. 2005;63:323-328. Wiersinga WM, Perros P, Kahaly GJ, et al. Clinical assessment of patients with Graves’ orbitopathy: the European Group on Graves’ Orbitopathy recommendations to generalists, specialists and clinical researchers. Eur J Endocrinol. 2006;155:387-389. Zoumalan CI, Cockerham KP, Turbin RE, et al. Efficacy of corticosteroids and external beam radiation in the management of moderate to severe thyroid eye disease. J Neuroophthalmol. 2007;27:205-214.

18 QUESTION

HOW DO YOU MANAGE DIPLOPIA IN THYROID EYE DISEASE? Kimberly Cockerham, MD, FACS A frail White 65-year-old woman with a 15-year history of Graves’ disease and thyroid eye disease (TED) complains of worsening diplopia over the preceding several months. On examination, she has bilateral proptosis, upper and lower eyelid retraction, limitation of upgaze (right greater than left), and a 20 prism diopter exotropia in primary gaze. Before delving into an ophthalmological core history and physical examination, I am concerned that an exotropia is not typical for TED. In addition, while reactivation of thyroid eye disease has been described, it is unusual. The differential diagnosis of what might be occurring in the presence of known thyroid disease includes myasthenia gravis, lymphoma, giant cell arteritis, paraneoplastic syndrome, partial third nerve palsy, or infiltration/fibrosis of the lateral rectus muscle.

Optimize Your History Taking: Ask the Right Questions and Listen It is important to ask about applicable ocular and systemic issues; many patients do not volunteer this information because they do not know that it is relevant. Is the double vision worse in the morning, at midday, or in the evening? Thyroid eye disease causes double vision that is worse in the morning, whereas myasthenia gravis is characteristically associated with double vision that is worse at the end of the day. Both disorders can fluctuate from hour to hour, day to day, and week to week, and both can result in double vision that is exacerbated with visual activities, such as reading, working on the computer, and driving. Lymphoma and temporal arteritis, in contrast, present with a diplopia that is stable over the course of the day and from day to day. Important systemic historical points (noted in Tables 18-1 through 18-3) include fatigue, muscle weakness, change in voice or swallowing, weight loss, muscle ache, jaw claudication (ache occurring after prolonged chewing of meat or gum), headache, temporal tenderness, and transient blurring of vision.

73

Lee AG, ed. Curbside Consultation in Neuro-Ophthalmology: 49 Clinical Questions, Second Edition (pp 73-78). © 2015 SLACK Incorporated.

74  Question 18

Table 18-1

Features of Myasthenia Gravis ●







Affects all ages Double vision worse at the end of the day or with use (eg, computer work, reading, watching television) Ptosis of one or both eyelids that is worse at the end of the day or with use Afternoon tearing due to orbicularis weakness resulting in ectropion of puncta and/or eyelids



Snarl appearance to mouth due to weak perioral muscles



Weakness of proximal limbs, causing difficulty climbing stairs or brushing hair



Change in voice (nasal sounding)



Difficulty swallowing or choking with normal bites of food



Shortness of breath

Table 18-2

Features of Lymphoma ●

Affects all ages



Conjunctival infiltration (the classic salmon-patch appearance)



Orbital infiltration, including tendinous insertion of extraocular muscles



Enlargement of the lacrimal gland on examination and imaging



Enlargement of the superior rectus or lateral rectus while sparing the medial and inferior recti



Painless lumps in neck, armpit, or groin



Weight loss due to loss of appetite



Fever and night sweats



Pruritus of face and body



Diffuse weakness



Breathlessness with swelling of face and neck

How Do You Manage Diplopia in Thyroid Eye Disease?  75

Table 18-3

Features of Giant Cell Arteritis ●

Age older than 50 years (most commonly White patients older than age 70 years)



Transient visual loss



Acute anterior or posterior optic neuropathy



Cranial neuropathies



Orbital infiltration



Headache



Temporal tenderness and/or lack of temporal artery pulse



Jaw claudication



Fatigue and muscle aches (polymyalgia rheumatica)



Weight loss

Focus Your Examination: Remember to Look at the Patient With the Lights on Active thyroid eye disease is characterized by redness and swelling of the conjunctivae and eyelids. Hyperemia and chemosis of the plica and caruncle are particularly characteristic. The inflammatory signs are often accompanied by a vague ache and tenderness. Proptosis can progress insidiously or rapidly owing to intraconal fat expansion and/or enlargement of the extraocular muscles. The upper eyelids, and sometimes the lower eyelids, are typically retracted. Diplopia in the morning is common and may progress to constant diplopia. The inferior and medial recti are most often involved and present as esotropia with restriction of abduction and/or hypotropia with restriction of upgaze. In contrast, chronic thyroid eye disease is associated with a painless white eye and eyelid. The eyelid retraction, proptosis, and dysmotility do not fluctuate. Resistance to retropulsion is typically present (apply pressure with the fingers if standing behind the patient or thumbs from in front of the patient). TED typically causes an elevation of the upper eyelids that is most pronounced laterally because of the fibers of Müller muscle, which extend between the orbital and palpebral lobes of the lacrimal gland. Any patient with drooping of the upper eyelid has myasthenia gravis until proven otherwise. Patients with chronic TED often appear to have a lateral “fat pad” in the upper eyelid due to lacrimal gland prolapse. The lower eyelid may have fat herniation from intraconal fat expansion. Decreased visual acuity can occur in active or chronic disease and arises from tear instability, corneal alterations, or optic nerve dysfunction. The optic nerve may be stretched or compressed at the orbital apex by the extraocular muscles. Red desaturation is a helpful afferent function test for asymmetrical TED optic nerve compression. A relative afferent pupillary defect may be subtle or absent in bilateral cases.

76  Question 18

Table 18-4

Crucial Elements of the Ophthalmic Examination ●

Sustained upgaze and Cogan lid twitch (if abnormal, think myasthenia)



Saccades (if abnormal, it is not isolated TED)



Pursuits and pattern of deviation (if exotropia, it is not isolated TED)





Appearance of conjunctivae and lacrimal glands (if salmon pink infiltration, think lymphoid infiltration) Temporal tenderness or pulseless temporal artery (think temporal arteritis)

Remember that TED, myasthenia gravis, lymphoma, and giant cell arteritis can all mimic cranial nerve patterns of deviation, especially fourth and sixth nerve palsies. Of note, TED and lymphoid infiltration do not alter the velocity or accuracy of the saccade. In contrast, myasthenia gravis causes intrasaccadic delay, and cranial nerve palsies are characterized by slowed saccades. Do not be confused if the deviation maps out to a fourth nerve palsy with a specific head-tilt preference because this has been reported in TED. Isolated inferior oblique dysfunction has also been described. Temporal arteritis can result in orbital inflammation or cranial nerve ischemia with saccadic slowing. The ophthalmic examination should focus on quantifying the TED findings and categorizing the patient as active or chronic. However, care should be taken in atypical cases to also look for signs of myasthenia gravis and other conditions that can mimic TED (see Tables 18-1 through 18-4).

Are the Eyelid Findings Consistent With Thyroid Eye Disease? The upper eyelid assumes a staring appearance during the active phase of hyperthyroidism and less commonly in other autoimmune disorders of the thyroid. Lid lag is the dynamic reduction in descent velocity of the eyelid in attempted closure. This contrasts with lagophthalmos, which represents the separation in millimeters of the eyelids in passive downgaze. Test orbicularis strength by having the patient squeeze his or her eyes closed; then observe for upward deviation of the eye with forced closure (Bell’s phenomenon). Look for ptosis by measuring the vertical palpebral fissure, the levator function, and the distance of the upper and lower eyelids from the corneal reflex. If ptosis is present, proceed with ice/rest test, sustained upgaze assessment (inability to maintain upgaze for 5 seconds), and Cogan lid twitch testing (upward rebound of eyelid after sustained upgaze immediately followed by downgaze).

How Do You Manage Diplopia in Thyroid Eye Disease?  77 Figure 18-1. Esotropia, as pictured, due to medial rectus enlargement and fibrosis is common, but limitation of upgaze and a vertical diplopia is also prevalent. Exotropia is not characteristic of isolated thyroid eye disease.

Is the Dysmotility Consistent With Thyroid Eye Disease? The double vision of TED is typically due to inflammation and later restriction of the extraocular muscles. Vertical diplopia from restriction of upgaze is common, as is esotropia due to medial rectus involvement, as demonstrated in Figure 18-1. If dysmotility is present, proceed with an assessment of saccadic accuracy and velocity. Myasthenia gravis is characterized by an intrasaccadic delay; the saccade starts out fast and then slows. Cranial nerve dysfunction (III, IV, or VII) is characterized by slow saccades, while TED always has normal saccadic velocity and accuracy. Another feature that helps differentiate a neurological cause from a restrictive process is forced ductions. These are performed by using topical anesthetic and then grasping the conjunctiva and Tenon capsule overlying the medial rectus (or other muscle of concern) and moving the eye laterally. If it feels tight and cannot be fully abducted, this is a positive forced duction and indicative of thyroid eye disease. This can also be judged by using a Tono-Pen (Reichert Technologies) and measuring the intraocular pressure in primary gaze and then in abduction; an elevation of greater than 3 to 4 mm is also consistent with restriction.

If I Am Suspicious That Thyroid Eye Disease Is not Isolated, What Do I Do? Myasthenia Gravis Obtain acetylcholine receptor binding and modulating antibodies. Approximately half of the patients with ocular myasthenia do not demonstrate positive binding antibodies receptors. If negative, testing for anti-MusK antibodies may be positive and aid in diagnosis (of note blocking antibodies are rarely (1%) present without binding antibodies). Most clinicians no longer perform the Tensilon (edrophonium) testing; single-fiber electromyographic testing is most sensitive in systemic myasthenia. A CT scan of the chest should be performed; 10% of patient will demonstrate a thymoma. Surgical excision can be curative. Ocular myasthenia is often a clinical

78  Question 18 diagnosis based on clinical symptoms and characteristic eyelid signs including a positive ice test and characteristic saccades.

Non-Hodgkin Lymphoma A biopsy of the involved tissue is essential. Fresh tissue is sent for touch prep, immunohistochemistry, and flow cytometry. Referral to an oncologist is indicated for systemic workup. If the lymphoid infiltration is limited to one or both orbits, local external beam radiation is indicated.

Giant Cell Arteritis Laboratory assessment includes a complete blood count, sedimentation rate, and C-reactive protein. At least 20% of patients with biopsy-positive giant cell arteritis have normal labs, no systemic symptoms, or both. A biopsy of the temporal artery is essential; the larger the segment the better because skip lesions are possible. The tissue is sent for fixation and special stains. If the biopsy is positive, oral prednisone is initiated and involvement of a rheumatologist is recommended.

Summary ●









In TED, an exotropia is very unusual and should warrant careful examination for coexisting myasthenia gravis, lymphoma, temporal arteritis, neoplastic syndrome, a metastatic process, or cranial nerve palsy. Myasthenia gravis occurs in at least 5% of patients with Graves’ disease, so it should always be considered in those with double vision that is worse at the end of the day or in patients with exotropia or ptosis. Lymphoma and lymphoid hyperplasia occur more commonly in patients with Graves’ disease; the incidence depends on geographical location and genetics. Imaging studies—both CT and MRI—should be carefully evaluated for another process, such as lymphoma, that may occur concurrently with Graves’ disease. Thyroid imbalance is a common disorder; clinicians should be aware that patients with TED can have other coexistent diseases that should not be mistaken for the disease itself.

Bibliography Cockerham KP, Browne EE, Hong S. Orbital inflammation. Curr Neurol Neurosci Rep. 2003;3(5):401-409. Cockerham KP, Cockerham GC, Zwick O. Orbital inflammation. Focal Points. 2006. Cockerham KP, Chan SS. Thyroid eye disease. Neurol Clin. 2010;28:729-755. Cockerham KP, Olmos A, Gausas R. Ophthalmic and orbital manifestations of neurologic disease. In: M Aminoff, ed. Neurology and General Medicine. 4th ed. Philadelphia, PA: Churchill Livingstone; 2007. Evoli A, Tonali PA, Padua L, et al. Clinical correlates of anti-MuSK antibodies in generalized seronegative myasthenia gravis. Brain. 2003;126(pt 10):2304-2311. Krassas GE, Boboridis K. Recent developments in the medical treatment of thyroid eye disease. Orbit. 2006;25:117-122. Odel JG, Winterkorn JM, Behrens MM. The sleep test for myasthenia gravis. A safe alternative to Tension. J Clin Neuroophthalmol. 1991;11(4):288-292. Smith KH. Myasthenia gravis. Focal Points: Clinical Modules for Ophthalmologists. San Francisco: American Academy of Ophthalmology; 2003, module 4.

19 QUESTION

WHAT ARE THE EVALUATIONS AND TREATMENTS FOR ACQUIRED NYSTAGMUS? Janet C. Rucker, MD A 35-year-old woman with a history of hypertension and breast cancer is being evaluated because, over the past week, everything she looks at has appeared to be in motion. On examination, she is found to have nystagmus. What are the possible causes? What workup should be ordered? What if an MRI does not show a structural lesion in the brainstem or cerebellum? Nystagmus (Table 19-1) is a spontaneous repetitive movement of the eyes initiated by slow eye drifts away from desired eye position and initially categorized as jerk (repetitive slow drifts followed by corrective fast movements in the opposite direction) or pendular (slow oscillations to-and-fro without fast corrective movements). Both types may have horizontal, vertical, and/or torsional components. Not all nystagmus requires treatment. Pharmacological treatment is needed when nystagmus results in the symptom of oscillopsia, a subjective sense of visual motion, which is present in our patient. If our patient has downbeat nystagmus (DBN), you should immediately consider whether there is any relationship between her breast cancer history and the nystagmus. DBN is a jerk nystagmus initiated by slow upward drifts of the eyes with fast corrective downward phases. It is present in the primary position but may be of such low amplitude that it is not visible to the naked eye. It is typically much more prominent in lateral, downward gaze (“side-pocket nystagmus”) and often absent or converted to upbeat nystagmus in upgaze. A chin-down resting head position may be present to minimize oscillopsia. Strategies to optimize visualization of DBN in the central position include viewing the eye at the slit lamp with careful attention to scleral vessels for fastdownbeat motion and assessing for DBN by utilizing the advantage of a magnified view during dilated ophthalmoscopy. DBN most often represents cerebellar or cervicomedullary pathology; neurological evaluation is recommended to assess for signs of cerebellar ataxia and other signs of lower brainstem dysfunction. MRI of the brain with contrast is the initial study of choice to evaluate for a cerebellar or cervicomedullary structural process (eg, metastasis or leptomeningeal enhancement suggesting a relationship with her breast cancer history or other non-cancer-related common causes, such as stroke, atrophy suggestive of cerebellar degeneration, or Chiari I malformation). You should

79

Lee AG, ed. Curbside Consultation in Neuro-Ophthalmology: 49 Clinical Questions, Second Edition (pp 79-83). © 2015 SLACK Incorporated.

80  Question 19

Table 19-1

Acquired Types of Nystagmus Jerk Vestibular Peripheral Central Torsional Upbeat Localization: medulla, less often cerebellum or midbrain Downbeat Localization: cerebellum or craniocervical junction Substance-induced (lithium, anticonvulsants, alcohol) Periodic alternating nystagmus Localization: cerebellum (specifically flocculus and nodulus) Gaze-evoked nystagmus Physiological Pathological Localization: gaze-holding areas in brainstem, cerebellum Substance-induced (anticonvulsants, sedatives, alcohol)

Pendular Acquired pendular nystagmus Diseases of central myelin (multiple sclerosis, Pelizaeus-Merzbacher) Substance-induced (toluene) Oculopalatal tremor (vertical pendular) Whipple disease (convergent-divergent pendular)

Jerk or Pendular See-saw nystagmus Localization: suprasellar region or midbrain Nystagmus secondary to vision loss Optic nerve disease (usually demyelinating, pendular nystagmus typical) Retinal disease (jerk nystagmus typical)

What Are the Evaluations and Treatments for Acquired Nystagmus?  81 assess her medication and social history for exposure to lithium, anticonvulsant medications, or alcohol abuse. If the MRI is unrevealing, laboratory studies that may identify an underlying etiology of DBN include a neurological paraneoplastic antibody panel (of particular importance in our patient, given her history of breast cancer), vitamin E (to assess for deficiency), thiamine (although upbeat nystagmus and gaze-evoked nystagmus are more common than DBN with Wernicke encephalopathy), studies for celiac disease, and anti-glutamic acid decarboxylase antibodies. If she has rapidly progressive cerebellar dysfunction, lumbar puncture may be warranted to further rule out a meningeal infectious or neoplastic process. If she has slowly progressive cerebellar dysfunction and the above evaluation is unrevealing, a genetic degenerative spinocerebellar atrophy (SCA) may be the etiology. Genetic testing is available for some of these but is most revealing when there is a family history of cerebellar degeneration or when additional findings suggest a specific SCA (eg, slow horizontal saccades with SCA2, maculopathy with SCA7). No underlying cause for neurologically isolated DBN is found in up to 40% of patients. The best-studied treatments for DBN are the potassium channel blockers 4-aminopyridine (4-AP) and 3,4-diaminopyridine (3,4-DAP) (Table 19-2); 4-AP is better tolerated and has better penetration into the central nervous system. It is available in a long-acting form called dalfampridine that is FDA approved for gait dysfunction in patients with multiple sclerosis (MS). Contraindications to aminopyridine treatment include moderate or severe renal dysfunction or a history of seizure. An electrocardiogram (ECG) should be obtained before treatment initiation to exclude a prolonged QT interval and as a contraindication to treatment. The use of potassium channel blockers in DBN has a pathophysiological basis in that the medications block cerebellar Purkinje cell potassium channels, thereby restoring the baseline state in which the cerebellum inhibits upward vestibular eye movements. In the pathological state of impaired cerebellar function, such upward eye movements may be disinhibited, resulting in the slow upward drifts and corrective downward fast phases of DBN. Aminopyridines may be more effective in patients with cerebellar atrophy or idiopathic DBN than in those with structural cerebellar lesions. Other medications that might be tried for DBN include chlorzoxazone, gabapentin, and clonazepam (see Table 19-2). If our patient has pendular nystagmus, demyelination would be the most likely possibility; indeed, MRI of the brain in this young woman revealed T2-hyperintense periventricular lesions surrounding the ventricles. Acquired pendular nystagmus (APN) is most commonly encountered clinically in patients with an existing diagnosis of MS, although it may be the presenting feature of a first episode of demyelination, as in our patient. In this case, corticosteroid treatment may provide benefit. APN also occurs in clinical settings other than MS. For example, if it develops weeks or months following a posterior fossa stroke, it often represents oculopalatal tremor (OPT). In this setting, you should look in the patient’s mouth to assess for palatal movements that are synchronous with the nystagmus and obtain an MRI of the brain to look for confirmatory increased T2-weighted signal in and hypertrophy of the inferior olives in the medulla. In MS, APN may be accompanied by superimposed optic nerve demyelination, in which case the APN amplitude may be larger in the eye with the worse visual acuity. Thus, assessment of afferent visual function is also important in our patient. APN is one of the most visually disabling types of nystagmus. The two most effective treatments currently for this condition, particularly in the setting of MS, are gabapentin and memantine (Table 19-3). I prefer to start with gabapentin, given a published study reporting possible worsening of MS symptoms with memantine. The same medications may be tried for the APN of OPT, although it tends to be more treatment resistant.

82  Question 19

Table 19-2

Common Medication Treatments for Downbeat Nystagmusa

a

Medication

Typical Starting Dose (mg)

Dose Escalation (mg)

Contraindications

Common Side Effects

4-aminopyridine (4-AP)

5 tid

Maximum 10 tid

ECG needed prior to administration to rule out prolonged QT interval Moderate to severe renal dysfunction, seizure history

Insomnia, headache, dizziness, paresthesias

Dalfampridine (extendedrelease 4-AP)

10 daily

Maximum 10 bid

ECG needed prior to administration to rule out prolonged QT interval Moderate to severe renal dysfunction, seizure history

Insomnia, headache, dizziness, paresthesias

Chlorzoxazone 500 tid

None

Liver disease

Abdominal discomfort, dizziness; rare̶ gastrointestinal bleeding, hepatotoxicity

Gabapentin

100 tid

100 every 3 to 7 days to maximum dose 900 tid

Dose reduction Dizziness, somneeded with nolence, fatigue, renal dysfunction weight gain, unsteadiness

Clonazepam

0.25 daily

0.125 to 0.25 Significant liver mg every disease, narrow week to max angle glaucoma of 1.5 mg daily in divided doses

Drowsiness, ataxia, behavioral problems

All off-label non-FDA-approved recommendations based on available scientific evidence.

What Are the Evaluations and Treatments for Acquired Nystagmus?  83

Table 19-3

Common Medication Treatments for Acquired Pendular Nystagmusa Medication Typical Dose Starting Escalation Dose (mg) (mg)

a

Contraindications

Common Side Effects

Memantine

5 daily

5 every week Concomitant to maximum amantadine, 10 bidb ketamine, dextromethorphan

Fatigue, back pain, dizziness, headache, confusion, lethargy. In MS, may worsen other MS symptoms

Gabapentin

100 tid

100 every Dose reduction Dizziness, som3 to 7 days needed with nolence, fatigue, to maximum renal dysfunction weight gain, dose 900 tid unsteadiness

All off-label non-FDA-approved recommendations based on available scientific evidence.

b

Higher doses (40 mg daily, in divided doses) have been found efficacious in European trials.

Summary A systematic approach is required in the evaluation of nystagmus. Given that there are many types of acquired nystagmus (see Table 19-1), each with different localizations and etiologies and different pharmacological treatments based on these, you must first classify the nystagmus correctly to determine the proper diagnostic evaluation and medical therapeutic options. There are many types of nystagmus, each with different etiologies, localizations, and treatments. ●



Downbeat and acquired pendular nystagmus are two of the most common nystagmus types in adults.



Downbeat nystagmus is often due to pathology of the cerebellum or cervicomedullary junction.



Acquired pendular nystagmus is most commonly seen with MS or oculopalatal tremor.

Bibliography Claassen J, Spiegel R, Kalla R, et al. A randomized double-blind, cross-over trial of 4-aminopyridine for downbeat nystagmus—effects on slowphase eye velocity, postural stability, locomotion, and symptoms. J Neurol Neurosurg Psychiatry. 2013;84:1392-1399. Rucker JC. Pearls: nystagmus. Semin Neurol. 2010;30:51-53. Thurtell MJ, Joshi AC, Leone AC, et al. Crossover trial of gabapentin and memantine as treatment for acquired nystagmus. Ann Neurol. 2010;67(5):676-80. Thurtell MJ, Leigh RJ. Treatment of nystagmus. Curr Treat Options Neurol. 2012;14:60-72.

20 QUESTION

WHAT IS THE EVALUATION FOR ANISOCORIA? Sophia Chung, MD and Aaron Grant, MD

A 45-year-old woman is referred for anisocoria incidentally found on a routine eye examination. She was not aware of the problem. How should I approach her evaluation? In evaluating such a patient, we need to know whether the anisocoria is caused by a serious condition that will require urgent imaging and intervention or whether it is a less concerning entity. First, we must determine if there is a pathological basis for the anisocoria. Physiological anisocoria is the most common cause of pupillary asymmetry, occurring in 20% of the population. If the patient’s pupils are structurally normal, react normally to both light and near, and maintain the same degree of anisocoria in dark and bright light, the diagnosis is physiological anisocoria and no further workup is needed. Generally, the anisocoria is less than or equal to 1 mm. The pupil gauge is invaluable in this situation because 1 mm of anisocoria is fairly subtle in bright light with small pupils but obvious in large pupils. Notably, physiological anisocoria can vary in size and switch sides. We would ask the woman to bring in old photographs, which might reveal varying degrees of anisocoria. The history is important in evaluating a patient for anisocoria. Ask about prior eye disease such as iritis or herpetic infection, which can cause inflammatory changes such as posterior synechiae, iris atrophy, or atony. Trauma from direct eye injury or neck trauma might lead you to a diagnosis of traumatic mydriasis or Horner syndrome from a carotid dissection, respectively. Occupational history is equally important because health care or agricultural workers may have accidental (or purposeful) exposure to dilating or miotic agents. A history of cataract surgery, particularly if complicated, can cause pupillary asymmetry. A helpful way in which to document the duration of the anisocoria is to review old photographs. Anisocoria present for many years usually does not need a workup. Once you determine that the patient does not have physiological anisocoria, you then need to determine if her smaller or larger pupil is pathological. If her small pupil is problematic, the differential diagnosis is limited to Horner syndrome and pharmacological miosis. If her larger pupil is pathological, my differential is limited to Adie tonic pupil, third nerve palsy, pharmacological mydriasis, and benign episodic pupillary mydriasis.

85

Lee AG, ed. Curbside Consultation in Neuro-Ophthalmology: 49 Clinical Questions, Second Edition (pp 85-88). © 2015 SLACK Incorporated.

86  Question 20

Figure 20-1. Third-order Horner syndrome. (A) The anisocoria is increased in dim light. Note the right ptosis in the photo in bright light. (B) After cocaine 10% instillation, the left pupil dilates, more evident in the dim light, while the right pupil does not dilate. This confirms the Horner syndrome. (C) After 1% hydroxyamphetamine instillation, the right pupil does not dilate in response, localizing the lesion to the third-order neuron.

If the degree of anisocoria increases in dark; it is a problem of dilation of her smaller pupil. Causes include Horner syndrome or mechanical/pharmacological restriction to dilation. In Horner syndrome, I would expect dilation lag of the miotic pupil—that is, slower dilation due to sympathetic denervation when the light source is removed. Horner syndrome is often associated with ipsilateral ptosis owing to the loss of sympathetic innervation of Müller muscle with or without ipsilateral anhidrosis. Mechanical/pharmacological constriction of the pupil should be evident from a careful history and slit-lamp examination. If I suspect Horner syndrome, I typically employ pharmacological testing for confirmation. Traditionally, cocaine 10% is instilled in both eyes. After 30 to 60 minutes, the pupils are reexamined. If the anisocoria is greater than 1 mm, Horner syndrome is the likely diagnosis. However, the difficulty in obtaining cocaine and its relatively short shelf life have redirected many neuroophthalmologists to apraclonidine 1% as the confirmatory test. In using apraclonidine, the Horner pupil will dilate while the normal, larger pupil mildly constricts. A reversal of anisocoria confirms Horner syndrome. There is often retraction of the ptotic eyelid as well. Having confirmed Horner syndrome, the next step is to determine the site of interruption of the oculosympathetic pathway. The differential diagnosis depends on the location of the lesion. Traditionally, hydroxyamphetamine 1% is used to localize the lesion. If the third-order neuron is intact, the pupil will dilate in response to hydroxyamphetamine drops; in that case, the lesion involves the first- or second-order neuron. Neuroimaging from the head to the top of the chest and a thorough investigation are required for this localization of Horner syndrome. If the pupil does not dilate, the third-order neuron is damaged. If it exists in isolation (ie, there are no other cranial neuropathies), Horner syndrome can be benign (Figure 20-1). Of note, pain in the face or around the eye in association with Horner syndrome could signify dissection of the ipsilateral internal carotid artery and should be investigated with CT or MR angiography of the neck and skull base. Currently, hydroxyamphetamine drops can be difficult to obtain; therefore, many ophthalmologists will scan the patient once the diagnosis of Horner syndrome has been established without further pharmacological localization of the lesion. An important consideration in a pediatric patient with Horner syndrome is neuroblastoma arising from the sympathetic chain in

What Is the Evaluation for Anisocoria?  87

A

Figure 20-2. Adie tonic pupil. (A) In dim light, the pupils are nearly symmetrical. (B) In bright light, the right pupil does not constrict while the left pupil constricts normally. (C) Following administration of 0.1% pilocarpine, the pupils are nearly symmetrical in bright light.

B

C

the chest. Such a child should be tested for urine catecholamines and with imaging of the neck, chest, and abdomen. If the anisocoria is greater in bright light, the eye with the larger pupil is suspect. The differential diagnosis includes third nerve palsy, Adie tonic pupil, pharmacological dilation, traumatic/ mechanical mydriasis, and benign episodic pupillary mydriasis. A third nerve palsy that involves pupillary dilation requires urgent workup for an expanding aneurysm but is always associated with abnormal ocular motility and/or ptosis. Therefore, it is important to carefully examine the motility with alternate cover testing in multiple fields of gaze. If the motility is full and there is absence of ptosis, the next step is to examine the pupils at the slit lamp. A tonic pupil will segmentally constrict with vermiform (wormlike) movements, made obvious with magnification. The pupillary response to a near target is notably stronger to light in a tonic pupil, although slower than that of the fellow eye. The tonic pupil demonstrates supersensitivity to pilocarpine 0.1% eye drops (which should not affect a normal pupil), constricting the pupil within 1 hour of instillation (Figure 20-2). The tonic pupil is a benign condition typically affecting the parasympathetic fibers at the level of the ciliary ganglion. If the pupil does not constrict to pilocarpine 0.1%, pilocarpine 1% should then be instilled. A pharmacologically dilated pupil will not constrict to pilocarpine 1%. Benign episodic pupillary mydriasis, or “springing pupil,” is a benign condition that typically occurs in young healthy patients. Often, they have a history of migraine headaches. Since you may not witness anisocoria in the office, given its episodic nature, the diagnosis of springing pupil is often made by history and by photos taken while the pupil was dilated. Most commonly, the patient will be aware of mild blur, perhaps also photophobia and noticeable asymmetry of pupil size for minutes to hours. When this episodic dilation occurs in isolation, the patient can be reassured. Anisocoria ought not to instill fear in the heart of the clinician. The differential is relatively small, and a careful history and pupillary examination can guide you to the diagnosis. Pharmacological testing can help confirm the diagnosis. Although there are life-threatening conditions that present with anisocoria, the majority of patients will have a benign condition. An understanding of the pathophysiology and a few simple tests will allow you to easily differentiate between the two.

88  Question 20

Summary ●

Physiological anisocoria is common. The pupils react normally and maintain the same degree of anisocoria in light and dark.



Cocaine 10% or apraclonidine 1% can be used to confirm the diagnosis of Horner syndrome.



Anisocoria from a third nerve palsy is always accompanied by motility deficits and/or ptosis.



Adie tonic pupil will constrict with dilute (0.1%) pilocarpine.



Benign episodic unilateral mydriasis, or “springing pupil,” is a common cause of anisocoria in individuals who have migraines.

Bibliography Freedman KA, Brown SM. Topical apraclonidine in the diagnosis of suspected Horner syndrome. J Neuroophthalmol. 2005;25:83-95. Kardon, R. Anatomy and physiology of the autonomic nervous system. In: Miler NR, Newman NJ, Biousse V, Kererison JB, eds. Walsh and Hoyt’s Clinical Neuro-Ophthalmology. 6th ed. Philadelphia, PA: Lippincott Williams and Wilkins; 2006:649-714. Lam BL, Thompson HS, Corbett JJ. The prevalence of simple anisocoria. Am J Ophthalmol. 1987;104(1):69-73.

21 QUESTION

HOW AND WHEN SHOULD I WORK UP HORNER SYNDROME? Sachin Kedar, MBBS, MD and Valérie Biousse, MD

A 55-year-old man with hypertension complains of acute headache on the left and is found to have a left Horner syndrome. How should he be worked up and treated? This patient with an acute, isolated, painful Horner syndrome is considered to have a left internal carotid artery dissection until proven otherwise. He must be evaluated emergently with noninvasive cerebrovascular imaging studies. If a dissection is confirmed, he will have to be admitted and treated to prevent a cerebral infarction.

Presentation Horner syndrome is diagnosed clinically in a patient who presents with anisocoria, which is typically worse in the dark owing to paralysis of the pupillary dilator muscle. A narrowed ipsilateral palpebral fissure usually accompanies the anisocoria because the Müller muscle in the eyelids is denervated (Figure 21-1). Table 21-1 summarizes the clinical features of Horner syndrome.

Pathophysiology Horner syndrome is caused by a lesion anywhere along the sympathetic pathway that supplies the head, eye, and neck. The sympathetic pathway to the orbit comprises a 3-neuron relay to the end organ: first-, second-, and third-order neurons. This 3-neuron sympathetic pathway originates in the hypothalamus. The first-order neuron descends to the first synapse in the cervical spinal cord; the second-order neuron ascends to synapse in the superior cervical ganglion located close to the common carotid artery bifurcation; and the third-order (postganglionic) neuron provides the

89

Lee AG, ed. Curbside Consultation in Neuro-Ophthalmology: 49 Clinical Questions, Second Edition (pp 89-95). © 2015 SLACK Incorporated.

90  Question 21 Figure 21-1. Right Horner syndrome. There is a slight decrease of the palpebral fissure on the right and the right pupil is smaller than the left pupil.

Table 21-1

Clinical Signs in Horner Syndrome ●





Reduced palpebral fissure: mild ptosis involving both upper and lower lids due to paralysis of the smooth muscles (Müller) innervated by the sympathetic pathway Pseudoenophthalmos because of the reduced palpebral fissure Anisocoria̶miotic pupil is ipsilateral to eye with narrowed palpebral aperture



Dilation lag in the dark (slow dilation of the affected pupil)



Heterochromia in congenital Horner syndrome (lighter color on affected side)



Associated neurological symptoms and signs: Anhidrosis of ipsilateral face in preganglionic lesions (first or second order). Brainstem and spinal cord symptoms and signs suggest a first-order Horner syndrome. Arm pain, hand weakness, history of neck surgery or neck trauma suggests a second-order Horner syndrome. ○





sympathetic innervation to the head and neck regions. The common causes for Horner syndrome in adults and children are listed in Tables 21-2 and 21-3.

Pharmacological Testing Pharmacological testing is done to confirm the diagnosis of Horner syndrome in subtle cases and to localize the causative lesion. Diagnosis of Horner syndrome using cocaine or apraclonidine can be performed in adults and children older than age 1 year. Apraclonidine may cause acute dysautonomia in children younger than age 1 year.

How and When Should I Work Up Horner Syndrome?  91

Table 21-2

Most Common Causes of Horner Syndrome in Adults Based on Lesion Location Central (First Order)

Preganglionic (Second Order)

Postganglionic (Third Order)

Hypothalamus Stroke Tumor

Cervical spine disease Brachial plexus injury

Superior cervical ganglion Trauma Jugular venous ectasia Iatrogenic (surgical neck dissection)

Brainstem Stroke (lateral medullary infarction) Demyelination Tumor

Pulmonary apical lesions Apical lung tumor Mediastinal tumors Cervical rib Trauma Iatrogenic (jugular cannulation, chest tube, thoracic surgery) Subclavian artery aneurysm

Internal carotid artery Dissection Aneurysm Trauma Arteritis Tumor

Spinal cord (cervicothoracic) Trauma Syringomyelia Tumor (intramedullary) Demyelination Myelitis Arteriovenous malformation

Thyroid tumors

Skull base lesions (nasopharyngeal carcinoma, lymphoma) Cavernous sinus lesion Tumors Pituitary tumor Inflammation Thrombosis Carotid aneurysm Cluster headache

Cocaine Classically, cocaine drops have been used for pharmacologic confirmation, but they are difficult to obtain and store. Cocaine inhibits norepinephrine reuptake from the synaptic cleft and augments sympathetic activity in an intact neuron. Thus, Horner syndrome pupil should not

92  Question 21

Table 21-3

Most Common Causes of Horner Syndrome in Children Congenital*

Acquired

Birth trauma-related

Neuroblastoma

Cervical rib

Rhabdomyosarcoma

Congenital infections

Brainstem vascular malformations

Neuroblastoma

Brainstem tumors (glioma)

Congenital agenesis of the internal

Demyelination (brainstem)

Carotid artery

Carotid artery dissection

Idiopathic

Neck trauma Post surgical:  Jugular cannulation  Neck surgery  Thoracic surgery  Idiopathic

*Diagnosed within 4 weeks after birth

dilate as much as a normal pupil. After documenting the pupillary sizes, 2 drops of cocaine 10% are instilled in both eyes. The pupillary sizes are observed 45 to 60 minutes later. An increase of anisocoria greater than 1 mm indicates the presence of Horner syndrome.

Apraclonidine More recently, topical apraclonidine (used for glaucoma) has replaced cocaine for the diagnosis of Horner syndrome. Apraclonidine has a strong action as a presynaptic α2 agonist and a weak action as a postsynaptic α1 receptor agonist. Although normal pupils show no effect from apraclonidine, Horner syndrome pupils will dilate from the action of the weak α1 receptor agonist (denervation sensitivity). After documenting the pupillary sizes, 2 drops of apraclonidine 1% are instilled in both eyes. The pupillary sizes are observed 45 to 60 minutes later. A reversal of anisocoria indicates the presence of Horner syndrome. The size of the palpebral fissure may increase as well (Figure 21-2).

Hydroxyamphetamine Hydroxyamphetamine drops can be helpful to identify a postganglionic (third order) Horner syndrome but are infrequently used owing to the difficulties involved in obtaining, storing, and using controlled substances. This examination is usually not performed in young children, in whom the test is less reliable. Hydroxyamphetamine causes a release of the presynaptic

How and When Should I Work Up Horner Syndrome?  93 Figure 21-2. Diagnosis of Horner syndrome with apraclonidine drops (same patient as in Figure 21-1). The anisocoria is reversed, with the right pupil now slightly larger than the left pupil. Note the increase in palpebral fissures.

neurotransmitter into the synaptic cleft, thus augmenting the sympathetic effect. This effect is not seen if the postganglionic neuron (third order) is damaged. After documenting the pupillary sizes, 2 drops of hydroxyamphetamine 1% are instilled in both eyes. The pupillary sizes are observed 45 to 60 minutes later. Pupillary dilation is noted in the normal and the preganglionic (first- and second-order neuron) Horner syndrome, while postganglionic (third-order neuron) Horner syndrome will not dilate.

Evaluation of Adults With Horner Syndrome ●



The most classic cause of central (first-order neuron) Horner syndrome is lateral medullary infarction (Wallenberg syndrome); other causes include various thalamic, brainstem, and spinal cord lesions. Second-order Horner syndromes are most suggestive of neoplasm or trauma of the lower cervical spine, brachial plexus, or lung apex.

Third-order Horner syndromes point to lesions of the internal carotid artery, such as dissection or cavernous sinus aneurysms. A comprehensive history and physical examination usually provides clues to the localization and cause of Horner syndrome. A history of trauma or iatrogenic procedures involving the head, neck, and cervical spine should be sought. Physical examination should include detailed ocular, neurological, neck, supraclavicular, and chest examinations (sympathetic pathways to the head and neck). Lesions can be localized based on the accompanying signs and symptoms. Lesions of the brainstem and cervical cord are accompanied by neurological signs and symptoms, such as ataxia, nystagmus, sensory or motor deficits, and bowel or bladder symptoms. Lesions of the lung apex may be accompanied by arm pain and weakness from brachial plexus involvement. Lesions of the carotid artery may be accompanied by ipsilateral facial and neck pain. Lesions of the cavernous sinus may be accompanied by involvement of other oculomotor cranial nerves. Confirmatory tests can be ordered depending on the lesion location, presence of associated symptoms or signs, urgency of the workup, and the radiologist’s preference. Multiple imaging studies are often needed. For acute Horner syndrome with or without neurological deficits, urgent MRI of the head with an MR angiogram (MRA) of the head and neck should be obtained to evaluate cerebrovascular lesions, such as a carotid artery dissection. Patients with a neurological examination suggestive of cervical myelopathy should have a MRI of the cervical spine in addition to MRA of the head and neck. Patients with Horner syndrome accompanied by pain, numbness, or weakness of an arm should be evaluated by a computed tomography (CT) or MRI of the chest to evaluate for lesions at the pulmonary apex. Similar studies may be needed for patients with isolated or nonlocalizing Horner syndrome. We recommend a CT and CT angiogram of the head, neck, and chest, which ●

94  Question 21 will allow for examination of the brain and spine, of the head and neck soft tissues, and of large blood vessels of the head, neck, and chest; these views will also allow for examination of the pulmonary apex. Often, MRI and MRA studies of the brain, neck, and chest are obtained instead of a CT angiogram.

Evaluation of Children (up to Age 10 Years) With Horner Syndrome ●

The etiology of Horner syndrome in infants and children differs from that in the adult population.

The classic causes include birth trauma, neuroblastoma, vascular anomalies of the large arteries, and chest surgery. The evaluation of infants and children with Horner syndrome is mostly based on the age of discovery. A history of birth trauma or other iatrogenic procedures involving the head, neck, and cervical spine should be sought. Acute Horner syndrome in children younger than age 5 years should prompt investigations for a neuroblastoma. A comprehensive physical examination should be performed to evaluate for a supraclavicular or abdominal mass, which would be indicative of neuroblastoma. A contrasted MRI of the head, neck, chest, and abdomen should be obtained. A 24-hour urine sample should be sent to evaluate for urine catecholamines (vanillylmandelic acid, homovanillic acid). If these are unrevealing, an MRA of the neck and aortic arch is obtained. Since children have to be sedated for the MR examinations, MRI of the head, neck, and chest and MRA of the head and aortic arch can be performed in one test under sedation in a young child. MRI of the abdomen requires a separate test (and, therefore, another sedation) and is sometimes not performed in children with isolated Horner syndrome and a normal abdominal examination; however, this is decided on an individual basis. ●

Treatment of Horner Syndrome Most patients with Horner syndrome have no visual changes and tolerate a mild ptosis. Rarely, lid surgery is requested to correct a persistent ptosis. Topical apraclonidine corrects the ptosis associated with Horner syndrome and may be used intermittently for cosmetic reasons or when the ptosis reduces the superior visual field.

Summary ●







An acute, painful Horner syndrome should be presumed related to a dissection of the ipsilateral internal carotid artery unless proven otherwise. These patients are at risk for cerebral infarction and should be evaluated emergently. An isolated Horner syndrome in a young child should prompt a workup for neuroblastoma. A combination of an ipsilateral Horner syndrome (first order) and contralateral superior oblique palsy (fourth nerve palsy) suggests a lesion of the trochlear nucleus or its fascicle in the brainstem. A combination of an ipsilateral Horner syndrome (third order) and a sixth nerve palsy suggests a lesion in the cavernous sinus.

How and When Should I Work Up Horner Syndrome?  95

Bibliography Biousse V, Touboul PJ, D’Anglejan-Chatillon J, Levy C, Schaison M, Bousser MG. Ophthalmologic manifestations of internal carotid artery dissection. Am J Ophthalmol. 1998;126:565-577. Mahoney NR, Liu GT, Menacker SJ, Wilson MC, Hogarty MD, Maris JM. Pediatric Horner syndrome: etiologies and roles of imaging and urine studies to detect neuroblastoma and other responsible mass lesions. Am J Ophthalmol. 2006;142:651-659. Mughal M, Longmuir R. Current pharmacologic testing for Horner syndrome. Curr Neurol Neurosci Rep. 2009;9:384-389. Trobe JD. The evaluation of Horner syndrome. J Neuroophthalmol. 2010;30:1-2.

22 QUESTION

WHAT SHOULD I DO WITH A DILATED PUPIL? Randy Kardon, MD, PhD As a 44-year-old woman is being evaluated, the acute onset of a large pupil on the right side is noted. She is certain that it was not there 2 days earlier. She is being seen 3 days after onset of the problem. The right pupil is 6 mm and nonreactive to light and near and the left pupil is 4 mm and reactive to light and near. The motility exam is completely normal. What are the possible etiologies for this abnormality? Could this still be an early third nerve palsy? Can any eye drops help in the differential at this early time? In the case above, it is stated that the right pupil is nonreactive to light or near effort and the left pupil is normal in that it does react to both light and near. I usually use as bright a light as possible to confirm that the anisocoria increases in bright light, revealing that the problem is in the right dilated pupil in this case. Next, I determine whether the dilated nonreactive pupil is caused by iris sphincter palsy or excessive activation of the dilator muscle. Iris sphincter palsy is more common and overactivation of the dilator with sympathomimetic drugs is usually incomplete, with some pupillary reaction to light owing to the greater strength of the sphincter muscle. My first step is to establish that there is no associated motility disturbance—in particular, that this is an isolated mydriasis without extraocular motor signs of a third nerve palsy, which could be dangerous (eg, aneurysm of the posterior communicating artery or pituitary apoplexy). Sometimes, early involvement of the third nerve can be subtle and the patient may not be aware of diplopia or mild ptosis. Even if the patient has no symptoms of diplopia or shift on cross-cover testing, I usually take the time to show him or her a penlight in extreme fields of gaze with a red filter held over one eye to make sure that there is no very mild early third nerve dysfunction. Once you have established that there is no ocular misalignment even in extreme gaze, it is extremely unlikely that you are dealing with pupillary involvement from early third nerve involvement. It is possible to have pupillary involvement precede the extraocular involvement because of the superficial location of the pupillary fibers along the intracranial course of the third nerve, but this is rare. My next step is to determine if there is evidence of a segmental iris palsy. If the extraocular motility is normal, I then look carefully at the movement of the iris around the pupil border, using the magnification of a slit lamp and turning on and off the slit lamp beam to observe whether there is any evidence of segmental palsy. If there is a segmental palsy, it helps in localizing the lesion to the postganglionic territory. I have yet to observe an acute preganglionic efferent pupil

97

Lee AG, ed. Curbside Consultation in Neuro-Ophthalmology: 49 Clinical Questions, Second Edition (pp 97-99). © 2015 SLACK Incorporated.

98  Question 22

Figure 22-1. Left color panel of photos: Patient with unilateral left mydriasis that is more apparent in light (middle) compared with darkness (top). Addition of dilute 0.1% pilocarpine drops causes both pupils to constrict, but the left pupil constricts more (bottom), indicating cholinergic supersensitivity. Right monochrome panel of photos: infrared transillumination of the iris in the acute state of postganglionic denervation due to Adie pupil. In darkness, the entire sphincter is at rest and has no density except light gray in the pupillary border area (top left). With either light (middle left) or near effort (lower left), only the normal sphincter segment at the 7-o’clock meridian contracts, causing it to appear darker. The other segments are denervated and do not move to light or near and therefore do not change in density. After dilute pilocarpine 0.1% (top right), all of the nonworking segments that are supersensitive are contracted, even in darkness, causing them to now appear dark. Only the normal segment at the 7-o’clock meridian is not supersensitive and stays light gray in darkness. Aberrant regeneration with light-near dissociation has started to take place after 6 months, causing some pupil constriction even in darkness (middle right) and even more contraction with near effort (bottom right).

defect that affected some segments of the iris sphincter and not others. A segmental palsy with intervening, normally working sphincter segments is usually a sign of a postganglionic parasympathetic lesion. Yes, it is possible for all segments of the iris sphincter to be palsied, but this is very rare, and usually at least one small segment is spared. In some cases of segmental sphincter palsy, the cause is not denervation but rather direct damage to the sphincter muscle from ischemia (either previous high intraocular pressure from angle closure or pigmentary dispersion, ocular ischemia, or iris vasculitis from herpes zoster). My next step is to look for slit-lamp evidence of a retroillumination defect in the iris, which would be a sign of direct damage to the iris sphincter muscle and not denervation. The presence or absence of segmental palsy or sparing of some areas of the iris sphincter can be even better seen with infrared iris transillumination (Figure 22-1). Next, if the patient is still of an age where accommodative amplitude is measurable (middleaged or younger), I make an effort to quantify his or her accommodative amplitude in each eye separately by measuring the near point of accommodation while the patient is wearing glasses for distance correction. This tells you whether the ciliary body is also affected, either by denervation

What Should I Do With a Dilated Pupil?  99 or pharmacological influences. I also measure this before and after any pharmacological testing. Keep in mind that anticholinergic agents and parasympathetic nerve palsies will affect both the pupil and accommodation. I often ask the patient how he or she discovered the unequal pupils since the acute aspect is sometimes misleading. In other words, the patient may have acutely noticed it, but it may have been there for some time. Here, asking about light sensitivity or difficulty focusing at near may help to verify the timing of the event. Also, old pictures or even a magnified view of the patient’s driver’s license photo under the slit lamp can sometimes be revealing. If the cause is postganglionic parasympathetic denervation (eg, Adie pupil) and it has been present for more than 2 months, there should be, on careful examination, some aspect of aberrant regeneration with light near dissociation. Conversely, if the denervation is really acute, there should be no sign of light near dissociation. Pharmacological mydriasis (from either anticholinergic or sympathomimetic agents) may be associated with some pupillary reaction to light and near, especially if the contamination was weak or if the pharmacological mydriasis is starting to wear off. In these cases, the pupillary reaction is never segmental. The pupil may react some to direct-acting cholinergic stimulation (pilocarpine 0.5%), but not as much as the control opposite eye 30 minutes after having received the same drop. If pharmacological mydriasis is suspected, I inform the patient that it should begin go away within the next few days (unless he or she is intentionally causing the problem). Sometimes, I have the patient take a digital picture in bright light at home within a few days and email it to me for confirmation. Figure 22-1 demonstrates the clinical findings in a patient with an Adie pupil.

Summary ●





In a patient with unilateral mydriasis, first demonstrate that it is isolated, without any accompanying extraocular motility disturbance or ptosis. If these findings are absent, it is nearly certain that the mydriasis is not due to involvement of the third nerve. Establish whether segmental palsy of the iris sphincter is present. If it is segmental, the cause is either a postganglionic parasympathetic denervation (eg, acute Adie pupil; deep tendon reflexes may also be diminished, but not always) or, less commonly, direct damage to some segments of the iris sphincter from ischemia or trauma. When there is no segmental asymmetry of pupillary constriction, one should suspect either pharmacological mydriasis (anticholinergic or sympathomimetic) or sympathetic overactivation associated with migraine.

Bibliography Bremner F, Smith S. Pupil findings in a consecutive series of 150 patients with generalized autonomic neuropathy. J Neurol Neurosurg Psychiatry. 2006;77:1163-1168. Caglayan HZ, Colpak IA, Kansu T. A diagnostic challenge: dilated pupil. Curr Opin Ophthalmol. 2013;24:550-557. Falardeau J, Karon RH. Anisocoria. Focal Points. 2013;31:1-14. Jacobson DM. Benign episodic unilateral mydriasis: clinical characteristics. Ophthalmology. 1995;102:1623-1627. Kardon RH. Disturbances of the pupil. In: JH Noseworth, ed. Neurological Therapeutics Principles and Practice. 2nd ed. Oxford, UK: Informa Healthcare; 2006:1981-1997.

23 QUESTION

WHAT IS THE EVALUATION FOR EPISODIC ANISOCORIA? Aki Kawasaki, MD, PhD A 45-year-old man is noted by his wife to have an extremely large right pupil. He complains of some visual blurring in the right eye but denies any diplopia or toxic exposures. By the time he is evaluated 4 days later, the pupils are equal in size and reactivity, his vision is normal, and he has normal motility. What are the possible causes for his transient pupillary dilation? Patients with anisocoria nearly always point out the side of the larger pupil. That is not to say, however, that the larger pupil is the faulty one. In the setting of transient anisocoria, getting a descriptive account of accompanying symptoms or signs—such as visual blur, pain, ptosis, impaired eye movement, or conjunctival injection—will help to best guess the side of the abnormal pupil. In the patient described above, a history of visual blurring in his right eye suggests that his dilated right pupil was indeed the faulty one. A dilated pupil evokes anxiety on the part of both patient and clinician. The critical aspect for the clinician is to determine whether there are any associated neurological deficits, in particular ipsilateral ptosis or ophthalmoplegia, suggesting a partial third nerve palsy. If indeed the dilation has occurred as an isolated abnormality and has returned to its normal size and normal reactivity, this is certainly not a neurological emergency. Isolated and transient unilateral mydriasis is not a sign of a cerebral aneurysm, nor does it suggest a brain tumor that is in process of herniation. What, then, are possible causes of a transiently dilated pupil? Intermittent angle closure glaucoma must be excluded. A history of periorbital pain or headache and blurred vision—particularly seeing haloes around lights—accompanying pupillary dilation is highly suggestive. The patient may notice that the affected eye is red and tender to touch during the episode. Gonioscopy is diagnostic. Paroxysmal discharge of irritated cervical sympathetic nerves (Pourfour du Petit syndrome) is a rare cause of episodic unilateral mydriasis. It is suspected when other signs of sympathetic hyperactivity—notably lid retraction, conjunctival blanching, and facial hyperhidrosis—accompany the dilated pupil. This phenomenon has been described in occasional patients with disease of the cervical spine, upper cord, brachial plexus, or lung apex. The clue lies in ipsilateral arm pain and weakness or bilateral leg numbness. If such symptoms are present, imaging of the neck and chest is recommended.

101

Lee AG, ed. Curbside Consultation in Neuro-Ophthalmology: 49 Clinical Questions, Second Edition (pp 101-103). © 2015 SLACK Incorporated.

102  Question 23 A tadpole-shaped pupil is characteristic of unilateral mydriasis, which is distinctive for the pupil having a pointy peak on one side. It results from focal spasm of the iris dilator muscle. If the description is classic for a tadpole- or teardrop-shaped distortion of the pupil, pharmacological testing using either cocaine or iopidine eye drops is recommended to check for an underlying Horner syndrome. Pharmacological mydriasis usually occurs from inadvertent contact with plants containing belladonna alkaloids, such as jimsonweed (thornapple), angel’s trumpet, deadly nightshade, and black henbane. These plants may be cultivated in gardens or found growing wild along roadsides and in fields (eg, cornpicker’s mydriasis). The degree of pupillary dilation and loss of reactivity to light varies depending on the degree of plant alkaloid exposure. A similar degree of accommodative paresis accompanies the mydriasis. In the acute stage of intoxication, the diagnosis is made from a history of plant contact and the absence of pupillary constriction to topical pilocarpine 1%. In all cases, the mydriasis resolves in 1 to 7 days. Accidental ocular exposure to bronchodilating mists due to an ill-fitting aerosol mask is an occasional cause of a fixed and dilated pupil in a hospitalized patient. A correct diagnosis in this setting will spare everyone the stress, effort, and cost of an urgent neurological evaluation. Migraine can be associated with transient anisocoria, but the mechanism is not fully established and is likely multifactorial. In some patients, migrainous vasospasm causes reversible ischemia of the ciliary ganglion and leads to a dilated, poorly reactive pupil and accommodative palsy. Rarely, a permanently tonic pupil develops. In others, sympathetic dysregulation, occurring either as increased or decreased activity, has been described. Anisocoria during migraine may also be an exaggerated form of a physiological anisocoria. Benign episodic mydriasis is a syndrome of recurrent episodes of isolated unilateral mydriasis that typically last several hours. It occurs most commonly in young women with migraines. The mydriasis typically appears in the same eye but can alternate sides; it may occur during a migraine or independent of headache. When they are examined during an episode, some patients have a normal pupillary light reflex, whereas others show a poor light response with impaired accommodation. Some patients complain of visual blur; others have orbital pain or red eye. Benign episodic mydriasis probably represents a heterogeneous group of disorders, including migraine, having different mechanisms that result in transient unilateral mydriasis. It is called benign because it is not associated with any systemic or neurological condition. The patient described in this case seems to have benign episodic mydriasis. Given a normal neurologic and ophthalmologic examination, including gonioscopy, no further investigations are needed. A follow-up examination within the year can offer reassurance that signs of an underlying pathology have not surfaced in the meantime.

Summary ●





In the awake and alert patient, transient unilateral mydriasis, even if recurrent, is usually not a harbinger of serious intracranial pathology. The examination is focused on revealing ipsilateral ptosis or deficits in ocular motility suggesting a partial third nerve palsy. In the setting of a normal examination, the two most common causes of transient unilateral mydriasis are migraine and benign episodic mydriasis.

What Is the Evaluation for Episodic Anisocoria?  103

Bibliography Balaggan KS, Hugkulstone CE, Bremner FD. Episodic segmental iris dilator muscle spasm: the tadpole-shaped pupil. Arch Ophthalmol. 2003;121:744-745. Barriga FJ, Lopez de Silanes C, Gili P, Pareja JA. Ciliary ganglioplegic migraine: migraine-related prolonged mydriasis. Cephalalgia. 2011;31:291-295. Jacobson DM. Benign episodic unilateral mydriasis. Clinical characteristics. Ophthalmology. 1995;102:1623-1627. Miller NR. Intermittent papillary dilation in a young woman. Surv Ophthalmol. 1986;31:65-68. .

24 QUESTION

HOW DO YOU MANAGE TOXIC AND NUTRITIONAL OPTIC NEUROPATHIES? Alfredo A. Sadun, MD, PhD and Michelle Y. Wang, MD, PhD

A 35-year-old morbidly obese woman with a history of fibromyalgia, lupus, diabetes mellitus, hypertension, and gastric bypass surgery noted progressive vision loss in both eyes for 1 year. There was no history of smoking or alcohol abuse. Two years earlier, her visual acuity was 20/15 in both eyes (OU). On examination, visual acuities now are counting fingers OU. She can discern only 1 of 8 color plates OU. There is no relative afferent pupillary defect. Anterior segment and intraocular pressures are normal. Fundus examination demonstrates subtle temporal pallor OU. Visual field testing reveals dense central scotomas OU. What is the differential diagnosis and management? Toxic and nutritional optic neuropathies are typically characterized by slowly progressive, symmetrical, painless vision loss that is rarely worse than 20/200. Patients often describe the visual change as a dimming of central vision with a loss of color vision. There is no relative afferent pupillary defect due to symmetrical impairment. In the early stages, the optic discs may appear normal. Disc edema and hyperemia are occasionally seen in cases of acute intoxications of methanol or ethylene glycol. Temporal pallor followed by diffuse optic disc atrophy is often mild and late. These conditions share similar clinical features because of the underlying mitochondrial dysfunction. The papillomacular bundle (PMB) fibers are most susceptible to this type of injury because they are small, are unmyelinated, and have high energy demands. The most common nutritional optic neuropathy is due to deficiencies in vitamin B12 (cobalamin), folic acid, vitamin B1 (thiamine), vitamin B2 (riboflavin), and zinc. You often see those in conjunction with general malnutrition or malabsorption. Vitamin B12 deficiency is often secondary to pernicious anemia, partial or complete removal of the stomach or ileum, or a strict vegan diet. Severe eating disorders and gastric bypass surgery, as seen in the clinical case described here, are other risk factors for multiple nutritional deficiencies. Optic neuropathies due solely to nutritional deficiencies remain rare in the United States except in the setting of heavy tobacco and alcohol consumption.

105

Lee AG, ed. Curbside Consultation in Neuro-Ophthalmology: 49 Clinical Questions, Second Edition (pp 105-109). © 2015 SLACK Incorporated.

106  Question 24

Table 24-1

Agents Associated With Drug-Induced Optic Neuropathy MON (Confirmed) Non-MON (Purported) Antiretroviral drugs Chloramphenicol Erythromycin Ethambutol Linezolid Streptomycin

Amiodarone Chlorpropamide Cisplatin Clioquinol Cyclosporine Desferrioxamine Disulfiram Ethchlorvynol

Halogenated hydroxyquinolines Interferon-α Infliximab Iodoquinol Isoniazid Octamoxin Penicillamine

Pheniprazine Radiation Sildenafil Sulfonamides Tamoxifen Tobacco Vincristine

MON = mitochondrial optic neuropathy.

Many medications cause optic neuropathy by interfering with mitochondrial function (Table 24-1) but ethambutol is the most common cause. The regimen for treating mycobacterial disease usually has an initial phase of 2 months, with a recommended daily dose in adults of 15 to 20 mg/kg. This is followed by a continuation phase of 4 months including isoniazid and rifampin. Ethambutol is often continued until drug susceptibilities are available. Typically, we see toxic levels of ethambutol when the dosage is not adjusted according to the patients’ weight or renal function. Other drugs—such as chloramphenicol, linezolid, erythromycin, streptomycin, and antiretroviral drugs—have also been reported, especially in genetically predisposed individuals. Fortunately, drug toxicity is often dose- and duration-dependent and early discontinuation leads to significant visual recovery. There are other drugs with purported optic nerve toxicity that do not affect mitochondria (see Table 24-1). Toxic optic neuropathies include those due to methanol, ethylene glycol, and toluene, but many other agents have also been identified as potential causes (Table 24-2). Combined nutritional and toxic optic neuropathies such as the Cuban epidemic of optic neuropathy, tobacco-alcohol amblyopia, prisoners of war, and Strachan syndrome (nutritional optic neuropathy and peripheral neuropathy) have been reported. The differential diagnosis of toxic and nutritional optic neuropathy is extensive, including the following: Hereditary optic neuropathy ●



Compressive optic neuropathy



Infiltrative optic neuropathy



Inflammatory or demyelinating optic neuropathy



Vasculitic optic neuropathy



Radiation optic neuropathy



Maculopathy/macular dystrophy

How Do You Manage Toxic and Nutritional Optic Neuropathies?  107

Table 24-2

Nonpharmacologic Agents Associated With Toxic Optic Neuropathy ● ● ● ● ● ●

Arsenicals Aspidium (male fern) Carbon disulfide Carbon monoxide Carbon tetrachloride Ethylene glycola

● ● ● ● ●

Hexachlorophene Lead Methanola Thallium Toluenea

a

These are the most common.

Figure 24-1. Humphrey visual field of the same patient described in the clinical vignette demonstrates bilateral central scotomas even 10 months after correction for vitamin B12 deficiency secondary to gastric bypass surgery. However, visual acuities improved from counting fingers in both eyes to 20/20 in the right eye and 20/25 in the left eye.

Workup of a patient begins with a thorough history of medication use, toxic exposure, substance abuse, dietary deficiency, and family history. A history of chronic illness and surgery should also be explored. We should not overlook a detailed review of peripheral neurological symptoms. In addition to a comprehensive examination, visual field testing is essential (Figure 24-1). In some cases, magnetic resonance imaging (MRI) of the brain and orbits is needed to rule out compressive and ischemic lesions because loss of bilateral central vision can occur from bilateral anterior chiasmal or occipital lesions. Optical coherence tomography may assist in monitoring the thickness of the retinal nerve fiber layer (Figure 24-2).

108  Question 24

Figure 24-2. Optic coherence tomography retinal nerve fiber layer of the same patient after gastric bypass surgery demonstrates symmetric temporal pallor in both eyes. ONH, optic nerve head; RNFL, retinal nerve fiber layer; OD, right eye; OS, left eye; C/D Ratio, cup-to-disc ratio; S, superior; I, inferior; N, nasal; T, temporal.

For ancillary laboratory testing, we recommend testing levels of serum B12 and red blood cell folate in suspected cases. Vitamin B12 may be falsely normal in hepatic disorders or falsely low during pregnancy or folate deficiency. Elevated serum methylmalonate, homocysteine, or both may be used for confirmation. Antiparietal cell antibodies, anti-intrinsic factor antibodies, high gastrin, low pepsinogen I level, endoscopy, and peripheral blood smear demonstrating megaloblastic

How Do You Manage Toxic and Nutritional Optic Neuropathies?  109 anemia are useful tests. We should consider genetic testing for Leber hereditary optic neuropathy, as well as heavy metal screening, if the history is suggestive or the nutritional workup is negative. The key in managing drug/toxic optic neuropathy is to remove the offending agent. We may see a reversal of the process in some cases with early withdrawal. For methanol toxicity, either fomepizole or ethanol should be given along with hemodialysis, and bicarbonate should be administered to restore acid-base balance. For nutritional optic neuropathy, we need to correct all potential deficiencies. For vitamin B12 deficiency, we recommend a monthly intramuscular injection for the first 3 months. This is often accompanied by a single-dose intramuscular thiamine injection followed by oral thiamine. Oral maintenance of folic acid, B complex, and copper may be appropriate. Discontinuation of smoking and alcohol along with a well-balanced diet are helpful. Because of the systemic complexity of this condition, we highly recommend comanagement with an internist.

Summary ●





Toxic and nutritional optic neuropathies are typically characterized by fairly symmetric visual loss, early and profound loss of color vision, loss of high spatial frequency contrast sensitivity, central or centrocecal visual field defects, temporal pallor of the optic discs, and preferential loss of the papillomacular bundle. A thorough history of medication use, toxic exposure, substance abuse, dietary deficiency, family history, and peripheral neurologic symptoms should be documented. The workup for toxic and nutritional optic neuropathies includes visual field testing, optic coherence tomography, and ancillary laboratory testing tailored according to medical history and examination findings. In bilateral cases, magnetic resonance imaging of the brain may be necessary to rule out compressive and ischemic lesions.

Bibliography Carelli V, Ross-Cisneros FN, Sadun AA. Mitochondrial dysfunction as a cause of optic neuropathies. Prog Retin Eye Res. 2004;23:53-89. Luca CC, Lam BL, Moraes CT. Erythromycin as a potential precipitating agent in the onset of Leber’s hereditary optic neuropathy. Mitochondrion. 2004;4:31-26. Mackey DA, Fingert JH, Luzhansky JZ, et al. Leber’s hereditary optic neuropathy triggered by antiretroviral therapy for human immunodeficiency virus. Eye (Lond). 2003;17:312-317. Sadun AA, La Morgia C, Carelli V. Mitochondrial optic neuropathies: our travels from bench to bedside and back again. Clin Exp Ophthalmol. 2013;41:702-712. Wang MY, Sadun AA. Drug-related mitochondrial optic neuropathies. J Neuroophtjalmol. 2013;33:172-178.

25 QUESTION

WHAT ARE VISUAL PROCESSING DEFECTS AND HOW CAN I RECOGNIZE THEM? Swaraj Bose, MD A 70-year-old man who complains of a 5-year history of visual difficulty is being examined. He has been having trouble driving and has gotten into many accidents, although eye examinations have been said to be “normal.” Recently, he fell off of a fishing pier because he “ did not notice the ocean.” He also noted difficulty in cutting his food and in reaching for objects. On ophthalmological examination, his vision was intact with no significant visual field impairment. He missed all of the HHR color plates bilaterally, although he was able to name all of the colors on the plates correctly. He had difficulty describing the complete contents of a picture, although he could describe parts of the picture accurately. He had retained insight, a good sense of humor, and a relatively intact mental status otherwise. What is the likely cause of his visual impairment? My first reaction on hearing about the case would be to ask whether this man experiences organic or nonorganic (functional) vision loss. The next step should be to determine the possible anatomical location of his vision loss: (1) Is it optical in nature? Is something wrong with the focusing elements, such as the cornea, lens, or media transparency? (2) Is there a problem with the retinocortical component? (This includes problems in the retina, optic nerve, chiasm, optic tract, lateral geniculate body, optic radiations, and the primary visual cortex.) Or (3) Is this a problem with the visual integration or processing mechanism? (These include occipitoparietal or occipitotemporal pathway lesions or problems with visual attention and object recognition.) There are clues in the history suggesting that he is suffering from an organic cause rather than functional visual loss. Our patient has a history of repeated accidents, normal eye examination including visual acuity and fields, normal mental status, a reduced response to stimuli presented on either side of fixation (difficulty in cutting food, reaching for objects, missing color plates), a reduced ability to detect more than one visual object at the same time, and an inability to combine several viewed objects into a meaningful composite (difficulty in describing the complete contents of the picture but able to describe parts of the picture accurately). A combination of these findings suggests that there is a problem with visual processing as the cause of his visual disturbances. The diagnosis is bilateral visual inattention, visual disorientation, or simultagnosia.

111

Lee AG, ed. Curbside Consultation in Neuro-Ophthalmology: 49 Clinical Questions, Second Edition (pp 111-114). © 2015 SLACK Incorporated.

112  Question 25 Simultagnosia, a “spatial disorder of attention,” is an inability to report all of the items and relationships in a complex visual display despite unrestricted head and eye movements. It is caused by a defect in visual attention that results in an inability to sustain visuospatial processing across simultaneous elements in an array. This higher disorder of visual processing may result from bilateral superior strokes of the parieto-occipital lobe, occlusion of posterior cerebral arteries usually occurring after systemic hypotensive crisis or cardiac arrest, corticobasal syndrome, posterior cortical atrophy, or Alzheimer’s disease. A case has also been described following a lumboperitoneal shunt for pseudotumor cerebri. Experimental and clinical studies have confirmed that the parietooccipital areas are activated during visual attentional tasks. Simultagnosia occurs in 2 distinct settings: (1) hypoxic and hypotensive injuries in the distribution of the posterior cerebral circulation and (2) Alzheimer’s disease. In contrast to the typical case of Alzheimer’s disease, a visual variant has been described where patients present with predominantly visual symptoms, including homonymous visual field defects resulting from involvement of the visual association cortex. These patients have a normal eye examination but demonstrate atrophy of the parieto-occipital regions on magnetic resonance imaging (MRI), while those with normal brain MRIs show hypoperfusion in the parieto-occipital regions on functional neuroimaging (positron emission tomography [PET], functional MRI [fMRI]). The presenting complaints usually include tunnel vision, bumping into objects, trouble reading and interpreting pictures, loss of depth perception, and objects fading in and out of sight. Eye examinations—including visual acuity, pupils, anterior segment, and fundus—are normal. Visual fields are quite constricted. Visual stimuli may suddenly appear and disappear from fixation even as the eyes continue to stare at the stimulus. As in our case, observing the patient while performing an Ishihara or Hardy Rand and Rittler color test is a good and reliable test for simultagnosia, where the patient usually misses all color plates but is able to name the colors perfectly well. Another clue for diagnosis is the large discrepancy between relatively intact fields to cued, singlefinger confrontation without an interesting fixation target and depressed visual fields to uncued, standard multiple-finger confrontation or automated field testing. Also, these patients typically do not have problems reading individual letters, but they make numerous errors with words and sentences. Interpreting detailed pictures is also an issue; they can identify individual components but cannot identify their interactions. For example, a patient who was handed a dollar bill was able to identify George Washington but could not identify the bill as money. Other simple diagnostic tests include difficulty in counting arrays of objects without proprioceptive input, walking very cautiously and walking into large and previously viewed objects, and an inability to fixate on targets in extrapersonal visual space. Patients with simultagnosia are usually misdiagnosed with a functional visual loss; they often see several physicians before a diagnosis is established. Early recognition and a referral to a neuroophthalmologist is key to early management. The treating physician should obtain a thorough history, document normal anterior and posterior segment eye examinations, and perform screening tests, including visual field testing (Table 25-1). Other tests—such as neuropsychological testing (if you suspect Alzheimer’s disease), neuroimaging including brain MRI and MR angiography, and possibly functional neuroimaging (including PET and fMRI) in cases with negative MRI scans—may be needed to establish the correct diagnosis. I would like to emphasize that all patients do not need functional neuroimaging to establish a diagnosis and that cost-effectiveness should be kept in mind. Treatment strategies should be directed toward etiology and neurorehabilitation.

What Are Visual Processing Defects and How Can I Recognize Them?  113

Table 25-1

Screening Tests for Patients With Visual Processing Defects 1. Navigation

Patients with bilateral visual inattention walk hesitatingly in short, slow steps, with outstretched arms (like a blind person) and collide with objects previously noted to be there. It can be difficult to distinguish such a patient from the one with functional visual loss. Other tests should be used in conjunction.

2. Confrontation visual fields

Initially present single fingers; if the patient is able to identify single fingers, then present 2 fingers in the same quadrant. Continued failure to notice both fingers suggests visual inattention in the given quadrant.

3. Eliminating fixation target

While performing the confrontation visual field, instead of having the patient fixate on your eye, have him or her gaze ahead in space (eg, toward the wall) and repeat the test as above (test 2). The peripheral stimuli will be more easily identified when there is no competing fixation target, suggesting visual inattention.

4. Color plate tests

When the Ishihara or Hardy Rand and Rittler plates are used, the patient is usually unable to identify the numbers; however, he or she will be able to recognize the individual colors.

5. Counting arrays

After the patient correctly identifies an object like a pen, show 4 or 5 pens in an array and instruct the patient to count by sight. If this fails, ask the patient to touch and count the pens; if this tactile contact improves the accuracy of counting, visual inattention is a possibility.

6. Reading

Errors in reading̶such as compressing or omitting single words or parts, or jumbling the order of words̶suggest visual and spatial inattention.

7. Interpreting pictures

A failure to interpret the scene while correctly identifying the individual components in a magazine picture denotes visual inattention.

114  Question 25

Summary ●





Patients with lesions of the parietal vision-related cortex present with deficits related to the distribution of visual attention and the perception and manipulation of items in space. Those with unilateral lesions usually confine their attention to the ipsilateral hemisphere while those with bilateral occipitoparietal lesions keep their eyes fixed on a target, walk cautiously with outstretched arms, collide with previously seen objects, and behave as if they had searchlight vision. These patients should be distinguished from individuals with nonorganic or functional vision loss. Although they are usually not considered blind based on standard vision tests, functionally they are severely visually impaired.

Bibliography Lee AG, Martin CO. Neuro-ophthalmic findings in the visual variant of Alzheimer’s disease. Ophthalmology. 2004;111:376-381. Miller NR. Bilateral visual loss and simultagnosia after lumboperitoneal shunt for pseudotumor cerebri. J Neuroophthalmol. 1997;17:36-38. Poggel DA, Kasten E, Muller-Oehring EM, Buzenthal U, Sabel BA. Improving residual vision by attentional cueing in patients with brain lesions. Brain Res. 2006;1097:142-148. Rajgopal R, Bateman R, VanStavern GP. Visual Involvement in corticobasal syndrome. J Neuroophthalmol. 2012:32:338340. Trobe JR, ed. The Neurology of Vision. New York: Oxford University Press; 2001:334-336.

26 QUESTION

HOW DO I MANAGE PATIENTS WITH HEADACHE SYNDROMES WHO COME TO ME AS AN OPHTHALMOLOGIST? Deborah I. Friedman, MD, MPH A 46-year-old healthy woman experiences episodes of sharp pain in the left eye. She describes severe brief stabbing pains lasting 1 to 3 seconds that occur several times weekly. Her ophthalmic examination is normal. An ophthalmologist’s role in the management of headache syndromes falls into one of four major categories. Your responsibility is to determine whether the patient’s problem is (1) life threatening, (2) sight threatening but not ophthalmic in origin, (3) sight threatening and ophthalmic in origin, or (4) not ophthalmic in origin and nonemergent. In other words, should the patient be sent to the emergency department (ED), can he or she be referred to a neurologist, or is the condition ophthalmic? Life-threatening causes of headache with ophthalmic manifestations include pituitary apoplexy, cerebral aneurysm, venous sinus thrombosis, cavernous sinus thrombosis, carotid or vertebral artery dissection, and high-flow carotid-cavernous fistula. Most patients with these conditions present to the ED rather than the ophthalmologist’s office. However, a patient with an unruptured aneurysm of the internal carotid-posterior communicating artery may experience painful third nerve palsy. The pupil is generally involved with aneurysmal compression and a complete, painful third nerve palsy with a fully dilated pupil (about 6 mm in diameter) is highly suspicious for a compressive lesion requiring emergent evaluation. Any patient with painful ophthalmoplegia needs an urgent evaluation. While the Tolosa-Hunt syndrome of idiopathic inflammation within the cavernous sinus is a well-known cause, other more sinister conditions, such as lymphoma or fungal infection, first must be excluded. The differential diagnosis of headache with a Horner syndrome includes carotid dissection, vertebral artery dissection, and brainstem stroke or hemorrhage. Paresis of cranial nerves III, IV, and VI with ipsilateral frontal pain and numbness in the upper face (V1) occurs with cavernous sinus thrombosis. Complete ophthalmoplegia with conjunctival injection, chemosis, proptosis, and bruit suggests a high-flow cavernous-carotid fistula.

115

Lee AG, ed. Curbside Consultation in Neuro-Ophthalmology: 49 Clinical Questions, Second Edition (pp 115-117). © 2015 SLACK Incorporated.

116  Question 26 Sight-threatening, nonocular etiologies include optic neuritis, pseudotumor cerebri syndrome, and stroke. Patients with optic neuritis should be referred urgently for intravenous methylprednisolone treatment and evaluation for possible multiple sclerosis, neuromyelitis optica, and other etiologies. The typical patient with the pseudotumor cerebri syndrome is an obese female of childbearing age who develops symptoms de novo (idiopathic intracranial hypertension). However, the syndrome has many etiologies, including medications and conditions that impair venous outflow from the brain. Patients will generally have headache, transient obscurations of vision, and pulsatile tinnitus. Decreased visual function and diplopia suggest more advanced disease. After a careful examination—including acuity, pupil testing, perimetry, ocular motility, and documentation of papilledema—send the patient to the ED with a copy of your office notes, results of visual field testing, and fundus photographs, if available. The documentation from a well-done examination from the ophthalmologist’s office is an invaluable addition to the relatively crude assessment of vision that can be performed in the ED and inpatient settings. In the absence of other neurological symptoms or signs, ischemic or hemorrhagic strokes affecting the occipital lobe may produce headaches associated with a homonymous hemianopia. Pituitary tumors are often associated with headache. The ophthalmic manifestations depend on the direction of suprasellar extension of the tumor in relation to the optic chiasm. The classic bitemporal hemianopia is a late finding in many cases, but a temporal defect respecting the vertical meridian is characteristic. Junctional scotomas and homonymous field defects may also occur. If visual manifestations or periocular pain is present, some patients with a primary headache disorder will seek care from an ophthalmologist first. If the ophthalmic examination is normal, such a patient may be referred to a neurologist or a headache specialist. The visual aura of migraine, which may occur with or without a headache, is often thought by the patient to be an ocular problem. The symptoms of visual aura may be positive or negative. The headache may begin during or after the aura symptoms, and some patients have little or no headache. Cluster headache is one of the most severe forms of pain imaginable; it is located in the area of V1, localized near or behind one eye. The pain peaks within minutes and is generally described as boring, crushing, pushing, and excruciating. Ipsilateral autonomic features of cluster headache also involve the eyes: lacrimation, conjunctival injection, ptosis, Horner syndrome, eyelid edema, and rhinorrhea or nasal congestion. Cluster headache is twice as common in men as in women, and the headaches may begin at any age. The first episode may suggest carotid dissection. While not emergent in the sense that they represent a sinister underlying problem, cluster headaches are so severe that an urgent referral to a neurologist or headache specialist is recommended to initiate treatment. Two “cluster headache cousins” are trigeminal autonomic cephalgias—paroxysmal hemicranias and SUNCT syndrome—which have features similar to cluster headaches but are briefer and occur more frequently. The paroxysmal hemicranias last up to 30 minutes but are otherwise similar to cluster headaches. SUNCT syndrome (short-lasting unilateral neuralgiform headaches with conjunctival injections and tearing) produces attacks lasting minutes; they are characterized by intense pain around the eye as well as conjunctival injection and tearing, often with eyelid edema and erythema. They may occur 80 to 100 times daily, so it is possible that an attack may be witnessed in the office. There are two other types of primary headache that are frequently unrecognized, but patients often see an ophthalmologist initially because their pain involves the eye area. With hemicrania continua, the unilateral, side-locked headache need not encompass the entire side of the head and is often located in the front of the head, around the eye or “in the eye.” The pain is always present, although it is generally mild (“in the background”) to moderate in intensity. At times, the severity of the pain increases, with migrainous characteristics. Many patients have symptoms for years before receiving the correct diagnosis. Primary stabbing headache is characterized by brief

How Do I Manage Patients With Headache Syndromes?  117 stabbing pains, described as feeling like an ice pick, a jolt, or a jab of pain. The pain may occur anywhere on the head, often varying in location from episode to episode, but it may also occur in the eye. It is harmless but disconcerting; patients with primary stabbing headache are concerned that the cause may be a brain tumor or aneurysm. Both syndromes are indomethacin responsive, with the response to indomethacin being both diagnostic and therapeutic. Tension-type headaches are usually bilateral and often frontal or behind the eyes. The pain is aching or pressure-like, mild to moderate in intensity, and not worsened by routine physical activity. A severe tension-type headache may throb with some mild photophobia. The symptoms of tension-type headache and “eye strain” headache are similar. If no significant refractive error or ophthalmic abnormality is found on the examination, referral to a primary care physician or neurologist is warranted.

Summary ●





Hemicrania continua and idiopathic stabbing headache are primary headache disorders that frequently affect the periocular area; both are treated with indomethacin. Providing documentation of the ophthalmologic examination—including best correct visual acuity, pupillary testing, perimetry, and fundus photography—is extremely helpful when referring a patient with papilledema for further evaluation. Potentially life-threatening disorders producing headaches are often accompanied by other neuro-ophthalmic signs, including Horner syndrome, cranial nerve palsies/ophthalmoparesis, proptosis, pupillary dilation, and papilledema.

Bibliography Friedman DI. Pseudotumor cerebri presenting as headache. Exp Rev Neurother. 2008;8(3):397-407. Friedman DI, Frishberg B. Neuro-ophthalmology and its contribution to headaches: a case-based approach. Exp Rev Neurother. 2010;10(9):1467-1478. Mathew PG, Garza I. Headache. Semin Neurol. 2011;31(1):5-17. May A. Diagnosis and clinical features of trigemino-autonomic headaches. Headache. 2013;53(9):1470-1478.

27 QUESTION

WHAT IS THE EVALUATION OF OPTIC ATROPHY? Julie Falardeau, MD A 47-year-old patient with impaired vision and optic atrophy bilaterally was seen for a routine eye examination. What etiologies should be considered? What is the likelihood of discovering a treatable etiology of the optic atrophy? A detailed history is essential in the evaluation of optic atrophy. The clinician must pay attention to the age of the patient, the onset of vision loss (sudden versus gradual), and the rate of visual decline (static versus progressive). An acute or subacute onset favors an underlying ischemic (older patients) or inflammatory/demyelinating process (typically but not exclusively younger patients). Leber hereditary optic neuropathy should be strongly considered in a younger patient reporting bilateral, simultaneous or sequential, painless, severe vision loss. Gradual monocular visual decline over the course of weeks to months should raise a concern for a compressive etiology. Toxic exposure, nutritional deficiency, or a compressive etiology should be considered with progressive binocular visual impairment. The presence of associated symptoms may also be helpful. A history of pain that is worse with eye movement at the onset of visual loss strongly suggests optic neuritis. The recent onset of headache, jaw claudication, or scalp tenderness in an elderly patient with optic atrophy should raise a concern for temporal arteritis. A detailed review of systems may reveal the presence of progressive peripheral sensory symptoms and gait disturbance, potentially reflecting a nutritional/toxic peripheral neuropathy. Associated skin lesions could suggest sarcoidosis or an infectious process, such as syphilis or Lyme disease in endemic areas. Medical history can provide potential etiologic clues: the presence of vasculopathic risk factors, multiple sclerosis, prior trauma, history of intracranial hypertension, systemic inflammatory disorders such as sarcoidosis, and so on. A thorough social history may reveal heavy tobacco and alcohol use, poor diet, or toxic exposure. A review of the medication list might reveal agents potentially associated with toxic optic neuropathy (eg, ethambutol, linezolid). An investigation of the visual status of other family members, especially in patients with bilateral optic atrophy, may uncover a hereditary optic neuropathy such as dominant optic atrophy or Leber hereditary optic neuropathy. Along with a thorough ophthalmic examination, formal visual field testing is essential in the evaluation of optic atrophy. Toxic, nutritional, and hereditary optic neuropathy should be considered in the presence of central or cecocentral scotoma. The combination of optic disc pallor and hemianopic visual field loss typically result from an optic chiasmal or retrochiasmal pregeniculate lesion.

119

Lee AG, ed. Curbside Consultation in Neuro-Ophthalmology: 49 Clinical Questions, Second Edition (pp 119-121). © 2015 SLACK Incorporated.

120  Question 27

Investigation Magnetic resonance imaging (MRI) of the brain and orbits with gadolinium (or a computed tomography scan if MRI is contraindicated) should be obtained in a patient with unexplained optic atrophy. I also recommend obtaining a neuroimaging study in patients with suspected toxic, nutritional, or hereditary optic neuropathy (unless the patient reports a strong family history of a similar disorder) since a compressive or infiltrative process can occasionally have a similar presentation. In the presence of a suggestive history or examination, targeted laboratory tests should include a complete blood count, serum B12 and folate, thiamine, sedimentation rate, C-reactive protein, angiotensin converting enzyme, syphilis serology, Lyme titer, Leber mitochondrial DNA mutation testing, and heavy metal screening. If the etiology of optic atrophy remains unknown after the initial evaluation/investigation, I would then recommend close monitoring to document the stability of the visual loss. In patients with a reportedly normal neuroimaging study but who are showing signs of progressive optic atrophy or progressive visual decline, the clinician should strongly consider obtaining a second opinion from neuroradiology. Alternatively, for progressive cases, the clinician may consider referring the patient to a neuro-ophthalmologist. To go back to our middle-aged patient with impaired vision and optic atrophy bilaterally, I would consider a compressive, toxic, nutritional, or hereditary etiology such as dominant optic atrophy. I would emphasize diet, tobacco and alcohol use, and drug and toxin exposure. I would investigate the patient’s prior visual status (has the patient had visual impairment since childhood?) and investigate the visual status of other family members. I would obtain visual field testing because it may reveal clues as to the causes of the optic atrophy. A bitemporal visual field deficit would strongly favor a suprasellar compressive process, whereas bilateral central or cecocentral scotoma would suggest toxic, nutritional, or hereditary optic neuropathy. MRI of the brain and orbits with and without contrast should be performed. In the presence of a reassuring neuroimaging study and based on the information obtained while interviewing the patient, I would consider laboratory studies such as a complete blood count, vitamin B12 , vitamin B1, folate, syphilis serology, and heavy metal screening. If that investigation were unrevealing, I would then simply observe given the very low probability at this point of missing a treatable condition.

Summary ●







A systematic approach—including a detailed history and thorough examination with formal visual field testing—will aid the clinician in determining the etiology of optic atrophy in the majority of cases. A neuroimaging study with gadolinium-enhanced MRI of the brain and orbits should be performed for cases of unexplained optic atrophy. In the absence of a suggestive history or examination findings, nontargeted broad-spectrum laboratory testing should be avoided, given its relatively low yield. If the optic atrophy is explained and nonprogressive, monitoring is appropriate. However, a referral to neuro-ophthalmology is recommended if the optic atrophy is unexplained and progressive.

What Is the Evaluation of Optic Atrophy?  121

Bibliography Lee AG, Chau FY, Golnik KC, et al. The diagnostic yield of the evaluation for isolated unexplained optic atrophy. Ophthalmology. 2005;112:757-759. Newman NJ. Treatment of hereditary optic neuropathies. Nat Rev Neurol. 2012;8:545-556. Orsaaud C, Roche O, Dufier JL. Nutritional optic neuropathies. J Neurol Sci. 2007;262(1-2):158-164. Wang MY, Sadun AA. Drug-related mitochondrial optic neuropathies. J Neuroophthalmol. 2013;33:172-178.

28 QUESTION

HOW DO I TREAT A CHILD WITH ASYMMETRIC NYSTAGMUS? Madhura A. Tamhankar, MD and Grant T. Liu, MD

A 2 year old who was noted by his mother to have “ jumping” eyes is being examined. He is found to have a pendular intermittent unilateral nystagmus of high frequency and small amplitude with a “shimmering” quality. The nystagmus is mainly horizontal with a torsional component. It is associated with irregular head nodding, and the child often also demonstrates a head tilt. What is the likely cause of these abnormalities? Should neuroimaging be performed? The clinical scenario involving this 2-year-old child is that of a spasmus nutans‒like condition. True spasmus nutans is a benign nystagmus that is composed of the triad of nystagmus, head nodding, and torticollis. It typically begins in the first year of life, usually after 6 months, and spontaneously remits in 1 to 2 years. It consists of pendular oscillations of low amplitude and high frequency (above 7 Hz) that may be unilateral or bilateral but asymmetrical. It is usually intermittent but is described as shimmering. It is mostly a horizontal nystagmus, but vertical or rotary components can be present. True isolated and idiopathic spasmus nutans is not accompanied by any neurological abnormalities, although strabismus or amblyopia may coexist. The diagnosis of spasmus nutans is one of exclusion. Groups of spasmus nutans‒like conditions exist that mimic the clinically observed signs of spasmus nutans yet are secondary to sensory defects, either intracranial tumors or retinal degenerative conditions. Several reports in the literature have documented a spasmus nutans‒like nystagmus with hypothalamic and parasellar tumors, such as optic pathway gliomas. Other signs of afferent visual pathway dysfunction—including loss of visual acuity, afferent pupillary defect, optic atrophy, endocrinological disturbances, and signs and symptoms of raised intracranial pressure, such as papilledema—may be present in these conditions. One large retrospective series of 67 children presenting with spasmus nutans found no child with intracranial abnormalities, although not all children underwent neuroimaging. Given the reported literature on the presence of hypothalamic and chiasmal gliomas in children presenting with spasmus nutans-like nystagmus, we would obtain magnetic resonance imaging (MRI) of the brain in this or any other patient presenting with this condition to exclude intracranial structural abnormalities.

123

Lee AG, ed. Curbside Consultation in Neuro-Ophthalmology: 49 Clinical Questions, Second Edition (pp 123-124). © 2015 SLACK Incorporated.

124  Question 28 Spasmus nutans‒like nystagmus and head movements have also been described in association with retinal dystrophies, such as congenital stationary night blindness and in spinocerebellar degenerations. Some authors have suggested that children presenting with a spasmus nutans–like condition should have an electroretinogram (ERG) routinely to rule out sensory causes for the nystagmus. Smith et al found an abnormal ERG in 3 of 8 children who presented with spasmus nutans‒like nystagmus that was consistent with cone-rod dystrophy in 2 patients and rod dystrophy in 1 patient. As reported by recent studies, the advent of handheld high-definition spectral domain optical coherence tomography (OCT) may, in the future, offer a useful screening method to identify foveal hypoplasia in infantile nystagmus. A normal OCT may obviate the need for electrodiagnostic testing, while if foveal hypoplasia is identified, further evaluation in the form of ERG and/or genetic testing may be undertaken to identify the relevant etiologies. We currently perform an ERG in patients presenting with asymmetrical acquired nystagmus only if a clinical suspicion for retinal dystrophy exists, as with subnormal visual acuity and an abnormal ophthalmic examination. It is important to note that although true spasmus nutans is thought to be self-limited, the visual outcome of patients with this condition is unclear. Although good visual acuity can be expected in patients with spasmus nutans, abnormal stereo acuity, latent nystagmus, strabismus, and amblyopia may develop in the long term.

Summary ●





Horizontal asymmetrical nystagmus usually occurs in 1 of 3 situations: (1) secondary to an intracranial lesion, (2) with monocular vision loss, or (3) as part of a triad that constitutes the diagnosis of spasmus nutans (asymmetrical nystagmus, abnormal head posture, head shaking). In addition to detailed history taking and clinical examination for systemic abnormalities, neuroimaging is suggested in all children presenting with this type of nystagmus to exclude intracranial space-occupying lesions. Electroretinography is reserved for those children in whom clinical suspicion for a retinal dystrophy exists based on history and ophthalmic examination.

Bibliography Arnoldi KA, Tychsen L. Prevalence of intracranial lesions in children initially diagnosed with disconjugate nystagmus (spasmus nutans). J Pediatr Ophthalmol Strabismus. 1995;32(5):296-301. Erratum in: J Pediatr Ophthalmol Strabismus. 1995;32(6):347. Farmer J, Hoyt CS. Monocular nystagmus in infancy and early childhood. Am J Ophthalmol. 1984;98(4):504-509. Gottlob I, Wizov SS, Reinecke RD. Spasmus nutans. A long-term follow-up. Invest Ophthalmol Vis Sci. 1995;36(13):27682771. Lavery MA, O’Neill JF, Chu FC, Martyn LJ. Acquired nystagmus in early childhood: a presenting sign of intracranial tumor. Ophthalmology. 1984;91(5):425-453. Lee H, Sheth V, Bibi M, et al. Potential of handheld optical coherence tomography to determine cause of infantile nystagmus in children by using foveal morphology. Ophthalmology. 2013;120(12):2714-1724. Smith DE, Fitzgerald K, Stass-Isern M, Cibis GW. Electroretinography is necessary for spasmus nutans diagnosis. Pediatr Neurol. 2000;23(1):33-36.

29 QUESTION

WHAT IS OPSOCLONUS AND HOW DO I MANAGE IT? Steve Newman, MD The patient presented is a 56-year-old man who noted frequent conjugate, involuntary, largeamplitude saccades in all directions. What is the likely cause and what evaluation is warranted? It is likely that this patient would also complain of oscillopsia. This perception of the world jumping is often the initiating complaint in patients that are found to have the condition described here. Opsoclonus, also known as saccadomania, is basically due to a free run of the saccadic burst cell generator, producing conjugate eye movements in all directions. Characteristic of these conjugate eye movements is the lack of normal intrasaccadic latency, which usually measures between 200 and 250 ms. Opsoclonus is distinguished from ocular flutter in that opsoclonus may occur in all directions, whereas ocular flutter is only horizontal. Both are presumed to be due to the same mechanism, which is an abnormality in the pause cells located within the nucleus raphe interpositus, which normally damps down the burst cells located in the parapontine reticular formation. The clinical findings are often accompanied by myoclonus and ataxia, indicating additional pathology affecting the cerebellum (particularly the fastigial nucleus or Purkinje cells). Opsoclonus usually occurs in three settings. Approximately 50% of these cases in children are due to the presence of a neuroblastoma potentially mediated through B-cell dysregulation. In young adults, the most common etiology is demyelinating disease (rarely neuromyelitis optica). In older patients, it is most commonly due to paraneoplastic syndromes. Anything that affects the pause cells may also produce this finding. Other inciting pathology has included infectious (varicella, HIV, hepatitis, neuroborreliosis, scrub typhus, Japanese encephalitis), parainfectious (post-viral, following influenza), and metabolic diseases (Hashimoto thyroiditis), which tend to have a better prognosis. Less commonly, vertebrobasilar insufficiency with or without MRI-visible brainstem infarcts or mass lesions may produce opsoclonus. In a 56-year-old man, the most likely etiology would be paraneoplastic, although we cannot exclude the possibility of other brainstem pathology. The key here would be obtaining a history of previous cancer, particularly small cell lung carcinoma, breast carcinoma (somewhat less likely in this patient), or other malignancies (thymoma; obviously endometrial and ovarian malignancies are not an issue in this patient). Autoantibodies, including anti-Ri, may be positive, usually associated with cancer of the breast

125

Lee AG, ed. Curbside Consultation in Neuro-Ophthalmology: 49 Clinical Questions, Second Edition (pp 125-126). © 2015 SLACK Incorporated.

126  Question 29 or pelvic organs and less commonly with small cell lung cancer or bladder cancer. Anti-Hu has also been reported, although this would be less common. Anti-Yo and other anti‒Purkinje cell antibodies may be found in women with paraneoplastic opsoclonus and cerebellar degeneration associated with cancer of the breast or gynecological organs. The absence of identifiable antibodies is not uncommon and does not exclude paraneoplastic syndrome. MRI scanning may be appropriate, and a lumbar puncture may be done to exclude the possibility of an inflammatory (ie, oligoclonal bands may be present) or infectious etiology such as viral encephalitis. Rare cases of opsoclonus have been associated with trauma, usually after hypoxia, although the history here would be diagnostic. Toxic causes of opsoclonus have also been reported (amitriptyline, lithium, phenytoin, cocaine). If a tumor is identified, treatment of the tumor itself sometimes results in improvement. Unfortunately, opsoclonus often fails to resolve after the neoplasm is resected. Other treatments have included the use of adrenocorticotropic hormone, systemic corticosteroids, and immunemodulating drugs such as cyclophosphamide (Cytoxan). Clonazepam, propranolol, and gabapentin may improve symptoms even if they do not abolish the eye movements. Plasmapheresis does not appear to be effective. Although intravenous immunoglobulin has been tried, it is unclear exactly how much benefit might be expected. The presence of B-cell activating factor in patients with opsoclonus has led to the use of B-cell depletion therapy with rituximab or ofatumumab. Spontaneous improvement in oscillopsia has also been reported.

Summary While many patients have nystagmus (and some have other spontaneous non-nystagmus movements), the sensation of the world moving (oscillopsia) is a most unusual symptom. The most common settings are in children related to the presence of a neuroblastoma, in young adults related to demyelinating disease, and in older patients related to paraneoplastic syndrome. In all cases, antibodies target the pause cell centers in the brainstem. When encountering the findings of opsoclonus, thinking about paraneoplastic syndromes and demyelinating disease often leads to the diagnosis of a systemic condition.

Bibliography Bataller L, Graus F, Saiz A, Vilchez J. Clinical outcome in adult onset idiopathic or paraneoplastic opsoclonus-myoclonus. Brain. 2001;124:437-443. Jen JC, Lopez I, Baloh RW. Opsoclonus: clinical and immunological features. J Neurol Sci. 2012;320:61-65. Klaas JP, Ahlskog JE, Pittock SJ, et al. Adult-onset opsoclonus-myoclonus syndrome. Arch Neurol. 2012;69:1598-1607. Lemos J, Eggenberger E. Saccadic intrusions: review and update. Curr Opin Neurol. 2013;26:59-66. Panzer J, Dalmau J. Movement disorders in paraneoplastic and autoimmune disease. Curr Opin Neurol. 2011;24:346-353.

30 QUESTION

WHAT IS WERNICKE ENCEPHALOPATHY AND HOW DOES IT AFFECT THE EYE? Lina Nagia, DO and James Corbett, MD

A 40-year-old man was seen in the emergency department and reported to have had slowly progressive confusion and imbalance for several days. He is disoriented and confused, with an unsteady gait. He has a full range of eye motion and no ocular misalignment. However, he has primary position upbeat nystagmus that becomes downbeat nystagmus with convergence, as well as mild horizontal end gaze nystagmus. What is the likely etiology of his confusion and eye findings? In this patient, our major concern would be Wernicke encephalopathy (WE). This is an acute neurological condition caused by thiamine deficiency and is historically associated with chronic alcoholism. The classic clinical triad of WE includes ataxia, confusion, and ophthalmoplegia. It is important to note that this triad of presenting symptoms occurs in only about 16% of patients. Stupor and coma are rarely seen in the initial phase of WE, but if untreated, WE can rapidly progress in a matter of days and lead to death, with an estimated mortality rate of 17%. A newer operational definition by Caine et al added nutritional deficiency to the triad and advised that only 2 of 4 criteria need to be present for diagnosing WE. Despite the importance of the oculomotor findings in both the classic and revised criteria for diagnosis, the incidence of these clinical findings is relatively low, occurring in only about one-quarter to one-third of patients studied at necropsy. The visual problems associated with WE include bilateral, frequently asymmetrical ocular motility disturbances and, rarely, acute nutritional optic neuropathy (Table 30-1). Although fourth nerve palsy has not been reported, sixth and third nerve palsies can occur. Asymmetry of pupil size and impaired light response have also been reported in 19% of patients with WE; however, this is the same prevalence as physiological anisocoria, which occurs in 19% of the population at any given time. Although classically described in malnourished alcoholics, other conditions have also been associated with WE (Table 30-2).

127

Lee AG, ed. Curbside Consultation in Neuro-Ophthalmology: 49 Clinical Questions, Second Edition (pp 127-130). © 2015 SLACK Incorporated.

128  Question 30

Table 30-1

Ophthalmological Involvement With Wernicke Encephalopathy 1. Horizontal Disturbances ○ ○ ○ ○ ○

Gaze palsy Abduction paresis Vestibular paresis Internuclear ophthalmoplegia Horizontal nystagmus

2. Vertical Disturbances ○ ○ ○ ○

Upbeat nystagmus (gaze-evoked) Downbeat nystagmus (gaze-evoked) Skew deviation Mild ptosis (rare)

3. Nutritional Optic Neuropathy ○ ○

Optic disc edema Retinal hemorrhages

The cause of WE is a deficiency of vitamin B1 (thiamine), which is a key coenzyme at 3 different points in intermediate carbohydrate metabolism. Depletion of thiamine and the subsequent stress of a glucose load may result in the relatively abrupt onset of WE. Blood levels of thiamine may be normal even in deficiency states, but blood transketolase levels reflect low thiamine levels more accurately. In your patient, however, you should not await the results of blood tests to initiate treatment. In addition, you should obtain serum magnesium levels (magnesium is a cofactor in glycolysis, which utilizes thiamine). If magnesium is low, it should be replaced when patients with WE are being treated. The repletion of thiamine when the magnesium level is low will fail to reverse the clinical features of WE. Regarding neuroimaging, computed tomography (CT) of the brain is of no help. Magnetic resonance imaging (MRI) in patients with WE reveals several lesions that are best seen on fluidattenuated inversion recovery, T2-weighted, and diffusion-weighted imaging sequences. High signals are found in the medial thalamus, hypothalamus, mammillary bodies, periaqueductal region, and surrounding the fourth ventricle (Figure 30-1). Other areas include the basal ganglia, cerebral cortex, pons, and medulla. Alterations are often symmetrical, and it is theorized that the specific areas targeted might be those involving intense thiamine metabolism. Imaging after treatment frequently shows generalized atrophy and volume loss throughout the brain. Regarding treatment, immediate administration of thiamine supplementation is critical. There is no consensus regarding the optimal dosing for treatment. Acutely, a minimum of 500 mg of thiamine hydrochloride in 100 mL of normal saline should be administered intravenously 3 times daily for the first 2 to 3 days, and 250 mg daily for the next 3 to 5 days, given over 30 minutes. Multiple studies found parenteral thiamine administration to be generally safe. Follow with thiamine 100 mg by mouth 3 times daily for the rest of hospital stay and outpatient treatment. It is

What Is Wernicke Encephalopathy and How Does It Affect the Eye?  129

Table 30-2

Conditions Other Than Alcoholism Associated With Wernicke Encephalopathy ● ● ● ● ● ● ● ● ● ● ●

Any condition associated with persistent vomiting or gastrointestinal disease Hyperemesis gravidarum Pyloric stenosis Esophageal stenosis Bariatric surgery Any condition associated with starvation Tumors causing anorexia Total parenteral nutrition without B-complex vitamins High-glucose infusions in patients with nutritional deficiency AIDS Neuropsychiatric conditions Anorexia nervosa Schizophrenia Dementia Miscellaneous conditions Neonatal illness Hunger strike Prisoners of war Complication of medications ○ ○ ○

● ● ● ● ●

Figure 30-1. Wernicke encephalopathy. Axial fluid-attenuated inversion recovery (FLAIR) MRI shows signal abnormalities in the (A) periaqueductal region and (B) medial thalami. (Reprinted with permission of Michael Vaphiades, DO.)

130  Question 30 important to correct other nutritional deficiencies, including replacing magnesium. Patients with anorexia, severe weight loss of any cause, persistent vomiting, starvation of any cause, and who are desperately ill in intensive care settings are all at risk for development of WE.

Summary ●





The diagnosis of WE should be considered in any individual who is confused, ataxic, or unable to walk and who has oculomotor or visual problems in the setting of nutritional deficiency of any cause. Patients with possible WE should have immediate IV administration of thiamine that should be given in advance of any IV glucose solutions. Ocular motility should start to improve within hours. Close monitoring and appropriate subsequent supplementation of thiamine is essential. Supplementary magnesium, if levels are low, is also critical. The clinical recovery from WE is uneven. Oculomotor signs improve first and most rapidly. Gait problems may persist due to concomitant nutritional neuropathy and/or cerebellar degeneration. The full-blown amnestic syndrome recovers fully in less than 20% of patients.

Bibliography Foster D, Falah M, Kadom N, Mandler R. Wernicke encephalopathy after bariatric surgery: losing more than just weight. Neurology. 2005;65:1987. Sechi G, Serra A. Wernicke’s encephalopathy: new clinical settings and recent advances in diagnosis and management. Lancet Neurol. 2007;6:442-455. Singh S, Kumar A. Wernicke encephalopathy after obesity surgery: a systematic review. Neurology. 2007; 68:807-811. White ML, Zhang Y, Lee AG, et al. MRI imagining with diffusion weighted imaging in acute and chronic Wernicke encephalopathy. AJNR. 2005;26:2306-2310. Zuccoli G, Pipitone N. Neuroimaging findings in acute Wernicke’s encephalopathy: review of the literature. AJR. 2009;192:501-508.

31 QUESTION

HOW DO I MANAGE POSTOPERATIVE VISUAL LOSS AFTER NONOCULAR SURGERY? Wayne Cornblath, MD After a lumbar laminectomy, a 45-year-old man complains of visual loss upon awakening. What etiologies should be considered? Should I give steroids? Should this patient receive a transfusion? Although the case presented is that of a patient who had lumbar spine surgery, postoperative visual loss (POVL) can occur with virtually any type of surgery. Among others, POVL has been seen with coronary artery bypass grafting, as well as with gastrointestinal, genitourinary, and lower extremity joint replacement surgeries. In the early 1990s, several case reports of POVL with spine surgery, as in this case, appeared, leading to further study and recommendations by the American Society of Anesthesiologists. In these patients, the discovery of the visual loss can be delayed by postoperative medications and anesthesia effects, producing confusion. Once the patient complains of visual loss, the examination is more difficult because of the limiting conditions under which it is performed (ie, in a hospital room, intensive care unit, or recovery room and the effect of medications on the examination, including miotic pupils and limited cooperation from the patient). Finally, medicolegal concerns always seem to arise in the case of a patient who has a potentially devastating complication far from the operative site. In evaluating POVL, the critical decision points are localization, etiology, and treatment. The five major possible causes of POVL are (1) external orbital compression with central retinal artery occlusion (CRAO), (2) ischemic optic neuropathy (anterior or posterior), (3) compression of the optic nerve or chiasm, (4) cortical visual loss, and (5) nonorganic visual loss. The visual loss can be unilateral or bilateral, mild or severe. Ischemic optic neuropathy is the most common etiology. This is a rare setting in which the history of the visual loss from the patient is not helpful in determining the etiology. The patient complains of visual loss, either in the recovery room or in the next 1 to 7 days. The onset of the visual loss can be delayed or the discovery of the visual loss can be delayed by confusion or drowsiness caused by postoperative pain medications. The examination will be critical, but it can be difficult. Visual acuity—usually done at near with appropriate presbyopic correction, pupillary examination, confrontation visual fields, external examination, ocular motility examination, and dilated fundus examination—requires all of the preceding.

131

Lee AG, ed. Curbside Consultation in Neuro-Ophthalmology: 49 Clinical Questions, Second Edition (pp 131-134). © 2015 SLACK Incorporated.

132  Question 31 Direct orbital compression occurs with surgery in the prone position and is usually monocular and is associated with ocular motility abnormalities, external edema, chemosis, and fundus findings of CRAO. Visual acuity and visual field function vary greatly. Ischemic optic neuropathy (ION) will show a relative afferent pupillary defect if the condition is unilateral or bilateral and asymmetrical. If it is bilateral and symmetrical, sluggish, poorly reactive pupils are seen. Unfortunately, medications can produce small, poorly reactive pupils and limit the pupillary examination. The optic nerves are swollen in approximately half of these patients (anterior ION) and normal in the other half (posterior ION). Occasionally, in patients with bilateral involvement, one nerve will be swollen and one will be normal. As in patients with nonarteritic anterior ION not associated with surgery, acuity can range from normal to no light perception and the typical visual field defect is altitudinal in nature. Compressive optic neuropathy can be unilateral or bilateral. Again, a relative afferent pupillary defect will be seen if the condition is unilateral or bilateral and asymmetrical. Visual acuity can be variable and the typical visual field defect is altitudinal. Typically, there is no optic nerve swelling and compressive optic neuropathy cannot be distinguished from posterior ION without imaging. Compression of the chiasm will produce variable acuity loss. The visual field findings are key to this diagnosis, with either bitemporal or junctional visual field loss. Typically, the optic nerves are not swollen. When the visual loss is severe, a bitemporal or junctional visual field defect may be absent; imaging is then needed to make the diagnosis of chiasmal compression. Cortical visual loss will have two patterns. If there is unilateral involvement of the postchiasmal optic pathway, the patient will have a homonymous hemianopic visual field defect with normal acuity. If there is bilateral involvement of the postchiasmal optic pathways, bilateral homonymous hemianopic visual field defects with variable loss of acuity will be seen. Macular sparing can leave markedly restricted visual fields with normal visual acuity or there can be variable loss of visual acuity. However, if visual acuity is affected with cortical visual loss, the visual acuity will be identical in each eye. In cortical visual loss, even if the patient is completely blind, the pupils will be normally reactive and the funduscopic appearance will be normal. Occasionally, these patients may even deny being blind (Anton syndrome). Patients with nonorganic visual loss will have inconsistent findings on examination, normal imaging, and, at times, an indifference to their level of visual loss. Neuroimaging is critical to confirm a diagnosis of cortical visual loss, particularly in patients with retrobulbar optic neuropathy or a chiasmal pattern of visual loss to rule out an acute compressive process, typically pituitary apoplexy. MRI is the preferred imaging modality because of the early detection of infarction, ability to distinguish posterior reversible encephalopathy syndrome (PRES), and exquisite optic nerve and chiasmal imaging (Figure 31-1). Laboratory evaluation should include hemoglobin. Checking the erythrocyte sedimentation rate is not helpful because it is routinely elevated postoperatively. Cortisol and TSH should be checked if pituitary apoplexy is a consideration. Treatment of POVL is controversial, with no clear evidence to guide us. In patients with optic neuropathy, I recommend transfusion to a hemoglobin of 10 g/dL if the patient is below this level, maintenance of normal blood pressure, and supplemental oxygen. There is no proven role for corticosteroid treatment, either high-dose intravenous or oral medications; intraocular pressure– lowering agents; or antiplatelet medications. In a patient with PRES, blood pressure, medications, and any other possible factors must be addressed. Recovery over time has occurred but is quite variable and impossible to predict. There have been several interesting studies of POVL in recent years. Two studies have used the Nationwide Inpatient Sample (NIS) to look at the incidence of POVL. The NIS allows diagnostic codes from millions of surgeries to be reviewed (with obvious limitations on details available and concerns about the accuracy of coding). In one study, the overall risk of POVL in spine surgery was 0.094%, with cortical visual loss being 0.087%, ION being 0.006%, and CRAO being 0.001%

How Do I Manage Postoperative Visual Loss After Nonocular Surgery?  133 Figure 31-1. (A) Normal T1-weighted MRI in a patient with postoperative visual loss. (B) Diffusionweighted imaging shows bilateral occipital infarctions.

in a total of 4,728,815 cases. Identified risk factors for ION were hypotension, peripheral vascular disease, and anemia. In the second study, a different time period of NIS data was used and multiple surgery types were evaluated. The highest rate of POVL, 0.087%, was found with cardiac surgery and the lowest rate, 0.0012%, involved appendectomy. Spinal surgery had a risk of POVL of 0.031%, with cortical visual loss being 0.015%, ION being 0.009%, and CRAO being 0.007% in a total of 465,345 cases. Identified risk factors for POVL were male gender, blood transfusion, anemia, and a Charlson risk factor greater than zero. Interestingly, over the 10 years of data reviewed by the study, the incidence of POVL decreased. Two studies used individual cases to determine risk factors for ION. The first study used cases with ION submitted to the American Society of Anesthesiologists Postoperative Visual Loss Registry and case controls from 17 academic medical centers. Identified risk factors were male sex, obesity, greater estimated blood loss, Wilson frame use, longer anesthetic duration, and lower percent colloid administration. Of note, the last four risk factors were not available in the NIS studies. The second study reviewed 360 articles from the literature and abstracted 119 cases in which enough data were available. Prone position surgery lasting longer than 5 hours and blood loss of more than 1 L were associated with ION. The American Society of Anesthesiologists reviewed the topic and published a practice advisory concerning POVL associated with spine surgery. They note that (1) preoperative anemia, (2) procedures longer than 6.5 hours, (3) blood loss of 45% or more, and (4) factors 2 and 3 combined all led to an increased risk of POVL. Preoperative ophthalmic examination is not believed to be useful. In terms of intraoperative management, the advisory notes that intraoperative hypotension should be used on a case-by-case basis; there is no recommendation for colloids versus crystalloids as intraoperative fluids, for a lower limit of hemoglobin, or for the use of vasopressors. Consideration of staged surgical procedures is recommended for procedures anticipated to be high risk (factors 2 and 3 above).

Summary ●

Localization of visual loss ○

CRAO +/- ophthalmoplegia, edema, chemosis from external orbital compression



Anterior ischemic optic neuropathy



Posterior ischemic optic neuropathy

134  Question 31







Cortical visual loss



Nonorganic visual loss



Unilateral or bilateral

Evaluation ○

Neuroimaging (MRI preferred) if no abnormality is seen in the eye



Check hemoglobin

Treatment ○

Consider transfusion if hemoglobin is below 10



Maintain blood pressure



Consider supplemental oxygen

Bibliography American Society of Anesthesiologists Task Force on Perioperative Visual Loss. Practice advisory for perioperative visual loss associated with spine surgery: an updated report. Anesthesiology. 2012;116:274-285. Lee LA, Newman NJ, Wagner TA, Dettori JR, Dettori NJ. Postoperative ischemic optic neuropathy. Spine. 2010; 35:S105-S116. Patil CG, Lad EM, Lad SP, Ho C, Boakye M. Visual loss after spine surgery: a population-based study. Spine. 2008;33:14911496. Postoperative Visual Loss Study Group. Risk factors associated with ischemic optic neuropathy after spinal fusion surgery. Anesthesiology. 2012;116:15-24. Shen Y, Drum M, Roth S. The prevalence of perioperative visual loss in the United States: a 10-year study from 1996 to 2005 of spinal, orthopedic, cardiac, and general surgery. Anesth Analg. 2009;109:1534-1545.

32 QUESTION

HOW DO I MANAGE TRANSIENT MONOCULAR VISUAL LOSS IN A YOUNG, OTHERWISE HEALTHY PATIENT? Rosa Ana Tang, MD, MPH and Roberto A. Cruz, MD

A 27-year-old woman reports having had 3 episodes of transient “ darkening” of vision in her right eye. Each episode lasted 3 to 5 minutes without any associated symptoms. Her general health is excellent; her only medication is an oral contraceptive. She has no history of migraine. Her examination, including visual fields, is entirely normal. What evaluation and therapy should be undertaken? Transient monocular vision loss (TMVL) of abrupt onset typically represents a focal retinal, choroidal, or optic nerve functional deficit due to ischemia. Historical clues are important, such as asking your patient what area of the visual field was lost; the duration of the loss; and the presence of any neurological symptoms, such as headache, numbness, or weakness. In your patient, the absence of scintillations and headaches and the fact that the vision loss is monocular makes a diagnosis of migraine less likely. A careful history, asking for associated systemic symptoms, is important, although none are reported by your patient. The differential diagnosis of TMVL in young individuals includes several entities. Vasospasm in the eye is thought to be due to spasm involving the retinal circulation and often affects the same eye every time. These patients have a stereotypical pattern of “patchy darkening” of vision for approximately 1 minute, followed by poor acuity for 1 to 5 minutes, followed by a return to baseline. There are no scintillations and no neurological abnormalities on examination. In my opinion, the best “investigation” is a thorough history and neuro-ophthalmic examination. Vasospasm is a benign condition that rarely leads to any permanent visual damage. I have found that the frequency of the TMVL events can sometimes be reduced by the use of calcium channel blockers. In the past, TMVL in young patients had been termed retinal migraine. This designation appears no longer correct for at least 2 reasons. First, most of the reported patients diagnosed with retinal migraine have not met strict International Headache Society criteria. Second, there are

135

Lee AG, ed. Curbside Consultation in Neuro-Ophthalmology: 49 Clinical Questions, Second Edition (pp 135-137). © 2015 SLACK Incorporated.

136  Question 32 no studies in humans to suggest that the retina is subject to “spreading depression,” the process believed to underlie the binocular visual aura emanating from the visual cortex in migraine. Transient monocular visual loss can be caused by reversible (by anticoagulation) retinal artery thrombi associated with heritable thrombophilia. These ocular vascular thrombotic events have opened a diagnostic window for screening and treatment of a variety of disorders, including factor V Leiden, prothrombin gene mutations, and hypofibrinolysis. Hypercoagulable/hyperviscosity syndromes are a consideration, especially in patients with a normal MRI of the brain who also lack significant ipsilateral internal carotid artery stenosis, atrial fibrillation, or cardiac thrombus. The patient in question is taking birth control pills; if she is a smoker and has history of migraine, she is at risk for both ocular and brain stroke. In other cases involving changes in intermittent systemic blood pressure (too high or too low), TMVL may be associated with changes in posture, bright light exposure, heavy exercise, or eating a large meal. If paroxysmal hypertension related to renal artery stenosis or pheochromocytoma is the cause, the patient may also experience episodic dizziness, as well as palpitations, sweating, and pallor. Cardiac emboli or arrhythmia related to valvular disease, such as patent foramen ovale or endocarditis in drug addicts, is characterized by erratic pulse, heart murmur, or both on physical examination. Less likely is the presence of premature atherosclerosis, carotid dissection, or fibromuscular dysplasia. Carotid artery dissection (CAD) is a common cause of stroke in previously healthy young individuals. About 75% of CADs lead to ischemic events, making it imperative that they be recognized and treated as soon as possible. Rare causes of TMVL are orbital tumors and posterior vitreous detachment. The latter gives rise to a typical “one-time event” of monocular TMVL in the majority of patients. Ocular coherence tomography is the instrument of choice to study the vitreoretinal interface and will image the presence of vitreoretinal traction in these patients. A standard algorithm for the assessment of young patients (below 50 years of age) with TMVL does not exist; diagnostic evaluation is determined by the history and specific examination findings, such as ocular inflammation, retinal venous or arterial changes, presence of a carotid bruit, level of blood pressure, pulse, heart murmurs, and any neurological deficits. A thorough ophthalmic examination is essential, especially dilated fundoscopy and visual field testing. In a healthy young patient with an isolated single episode of TMVL and no other associated symptoms and signs, I would conclude that the condition is due to retinal vasospasm and not do any further workup. Both visual and systemic prognoses are excellent. However, our patient has had 3 occurrences of TMVL; therefore, studies to exclude an embolic source (eg, heart disease), carotid dissection (look for ipsilateral Horner syndrome) vasculitis, or a hypercoagulable state need to be considered (Table 32-1). If all studies are normal, I would reassure the patient of the benign nature of the disorder and perhaps offer treatment with a calcium channel blocker. If the patient experiences a change in these stereotypical visual events or new signs and symptoms develop, reevaluation is indicated.

Summary ●





Many otherwise healthy young patients with an isolated single episode of TMVL and no other associated symptoms and signs have retinal vasospasm. Selected patients might require testing to exclude heart disease, vasculitis, and possibly a hypercoagulable state. A dramatic change in prior stable and stereotypical visual events or the development of new neurological signs or symptoms should prompt reevaluation as indicated.

How Do I Manage Transient Monocular Visual Loss in a Young Patient?  137

Table 32-1

Hypercoagulable State Testing ● ● ● ● ● ● ● ● ● ● ● ● ● ● ● ● ● ● ● ● ●

Anticardiolipin (aCL) (IgG or IgM) in serum or plasma Phospholipid units (GPL)/mL or IgM phospholipid units (MPL)/mL Lupus anticoagulant levels Anti‒beta-2 glycoprotein I antibodies (IgG or IgM) in serum or plasma Homocysteine levels Prothrombin fragment 1 and 2 D-dimer test Antithrombin III Fibrin and fibrinogen degradation products Fibrinopeptide A Platelet count Fibrinogen Prothrombin time Activated partial thromboplastin time Thrombin time International normalized ratio Protamine test Factor V Leiden Proteins C and S Prothrombin gene heterozygosity Lupus anticoagulant in plasma

Bibliography Bernard GA, Bennett JL. Vasospastic amaurosis fugax. Arch Ophthalmol. 1999;117:1568-1569. Donders RCJM, Kappelle LJ, Derksen RHWM, et al. Transient monocular blindness and antiphospholipid antibodies in systemic lupus erythematosus. Neurology. 1998;51:535-540. Glueck CJ, Goldenberg N, Bell H, Golnik K, Wang P. Amaurosis fugax: associations with heritable thrombophilia. Clin Appl Thromb Hemost. 2005;11(3):235-41. Glueck CJ, Wang P. Ocular vascular thrombotic events: a diagnostic window to familial thrombophilia (compound factor V Leiden and prothrombin gene heterozygosity) and thrombosis. Clin Appl Thromb Hemost. 2009;15(1):12-8. Hill DL, Daroff RB, Ducros A, Newman NJ, Biousse V. Most cases labeled as "retinal migraine" are not migraine. J Neuroophthalmol. 2007;27(1):3-8. Saarela M, Sundararajan S, Strbian D. Young patient with headache and amaurosis fugax. Stroke. 2014;45:e3-4.

33 QUESTION

WHAT IS THE EVALUATION FOR TRANSIENT MONOCULAR VISUAL LOSS IN AN OLDER ADULT? Byron L. Lam, MD A 72-year-old man who, 2 days previously, experienced loss of vision lasting approximately 1 minute in his left eye is being examined. He has no accompanying symptoms. His review of systems documented a 20-year history of hypertension, and he had coronary artery surgery 2 years prior. Medications include antihypertensive agents and aspirin. Ophthalmological evaluation is unremarkable. What should be done next? Ischemia is the main concern in transient monocular visual loss (TMVL). In older patients with TMVL, carotid artery stenosis, carotid or cardiac embolic phenomena, and ischemia associated with giant cell arteritis are serious conditions that could potentially cause permanent visual loss from ocular ischemia and increase the risk of morbidity and mortality. Workup in an older patient with TMVL is mandatory. Assessment begins with asking the patient whether the TMVL is complete or partial, as well as the duration and frequency of the episodes. Ischemic events typically last from seconds to several minutes and resolve spontaneously. Patients with partial visual loss related to ischemia are likely to report loss of the upper or lower half of the visual field with demarcation between the seeing and nonseeing areas. Increase in the frequency of TMVL is worrisome and suggests an increased risk of a permanent ischemic event. Elderly patients with TMVL should be asked about the presence of giant cell arteritis symptoms. This should be followed by a prompt workup and treatment for giant cell arteritis, if indicated. For all patients with monocular visual loss, a careful medical history should be obtained with a focus on vascular risk factors, including hypertension, diabetes mellitus, hyperlipidemia, coronary arterial disease, cardiac arrhythmia, and carotid artery disease. Other factors including tobacco use and oral contraceptives may contribute to the risk of ischemic events. Blood pressure check will assist in detecting poorly controlled hypertension. Patients with significant hypertension (eg, diastolic > 100 mm Hg) should be sent promptly to their internist or urgent care for treatment. A careful funduscopic examination after pupillary dilation should be performed, with an emphasis on detecting retinal intra-arterial plaques, an important clinical sign of emboli. The three common types of retinal emboli are cholesterol, platelet-fibrin, and calcium. Cholesterol emboli, also called Hollenhorst plaques, are beige, refractile emboli often found at retinal arteriolar bifurcations. Cholesterol emboli are related to atheromatous carotid

139

Lee AG, ed. Curbside Consultation in Neuro-Ophthalmology: 49 Clinical Questions, Second Edition (pp 139-141). © 2015 SLACK Incorporated.

140  Question 33 artery plaques, while platelet-fibrin and calcific emboli are associated with thrombus in the carotid artery or from cardiac valves. Given that retinal intra-arterial plaques may be fleeting, the absence of plaques does not rule out an embolic cause. In addition to consideration of giant cell arteritis, the workup will be expedited by sending the patient for a carotid duplex ultrasonogram, an important test to assess the presence of carotid artery stenosis and atheromatous plaques. Other diagnostic considerations include an echocardiogram to assess any source of cardiac emboli and an electrocardiogram to determine the presence of previous myocardial infarction or cardiac arrhythmia. Brain MRI with and without gadolinium will detect whether there are any cerebral infarcts; the extent of white matter changes on fluidattenuated inversion recovery (FLAIR) MRI is an indicator of the degree of arteriosclerosis. If new hemorrhagic infarcts are found, prompt hospital admission by an internist or a stroke specialist should be considered. The North American Symptomatic Carotid Endarterectomy Trial found carotid endarterectomy to be beneficial in symptomatic patients with 70% or greater stenosis, but carotid endarterectomy is contraindicated if the carotid artery is completely occluded. Ipsilateral stroke within 2 years was 9% in those who had surgery versus 26% in those treated medically. The benefit was significantly reduced for symptomatic patients with less than 50% stenosis. For patients who experienced TMVL, 6 factors were found to increase the risk of stroke: (1) age greater than 75 years, (2) male gender, (3) history of hemispheric transient ischemic attack or stroke, (4) intermittent claudication, (5) 80% to 94% internal carotid artery stenosis, and (6) no collateral circulation on cerebral angiography. In the Asymptomatic Carotid Atherosclerosis Study, the 5-year risk for ipsilateral stroke was 5% in those who had surgery compared with 11% for those treated medically. Taken together, consideration for carotid endarterectomy is warranted in symptomatic patients with greater than 50% stenosis (especially in those with 70% or greater stenosis) and healthy asymptomatic patients with greater than 60% stenosis. Also taken into consideration are the facts that the perioperative mortality of carotid endarterectomy is in the range of 1% to 2% and postoperative strokes occur in 1% to 5% of patients. Newer surgical alternatives to carotid endarterectomy include angioplasty and stenting; several clinical trials are continuing to define their clinical applications. Among patients with symptomatic or asymptomatic carotid stenosis, the risk of the composite primary outcome of stroke, myocardial infarction, or death does not differ significantly for carotid artery stenting versus endarterectomy. During the periprocedural period, a higher risk of stroke occurs with stenting and a higher risk of myocardial infarction occurs with endarterectomy. New ischemic lesions on diffusion-weighted imaging are found more frequently in patients after stenting. Medical treatments in patients with carotid stenosis and emboli include antiplatelet agents such as aspirin and clopidogrel (Plavix). Ticlopidine (Ticlid) is used less often because of the risk of neutropenia. In this patient, who is already taking aspirin, the addition of clopidogrel can be considered. Anticoagulation using warfarin in patients with noncardiac emboli is controversial. The visual prognosis of patients experiencing TMVL is generally favorable and related to the extent of carotid stenosis and the frequency of recurrent emboli. Mortality is increased in patients with TMVL, and death has been found to be more likely to result from myocardial infarction than from stroke. When pooled data from two population studies, the Beaver Dam Eye Study and the Blue Mountains Eye Study, were analyzed, the presence of retinal arteriolar emboli increased all-cause mortality by approximately 30% over 10 to 12 years.

Summary Initiating further workup in older patients with TMVL is critical. In my experience, ophthalmologists tend to refer this type of patient to neuro-ophthalmologists, internists, or stroke

What Is the Evaluation for Transient Monocular Visual Loss in an Older Adult?  141 specialists for further evaluation. However, the importance of performing a careful history and examination by the comprehensive ophthalmologist cannot be overstated. In older patients with TMVL, common workup considerations include giant cell arteritis and carotid artery stenosis/embolus. ●





Workup is mandatory when acute symptomatic retinal embolus is present. Referral to a neuro-ophthalmologist, internist, or stroke specialist for further workup may be indicated.

Bibliography Barnett HJ, Taylor DW, Eliasziw M, et al. Benefit of carotid endarterectomy in patients with symptomatic moderate or severe stenosis. North American Symptomatic Carotid Endarterectomy Trial Collaborators. N Engl J Med. 1998;339:1415-1425. Brott TG, Hobson RW, Howard G. Stenting versus endarterectomy for treatment of carotid-artery stenosis. N Engl J Med. 2010;363:11-23. Ederle J, Davagnanam I, van der Worp HB, et al.; ICSS investigators. Effect of white-matter lesions on the risk of periprocedural stroke after carotid artery stenting versus endarterectomy in the International Carotid Stenting Study (ICSS): a prespecified analysis of data from a randomised trial. Lancet Neurol. 2013;12:866-872. Executive Committee for the Asymptomatic Carotid Atherosclerosis Study. Endarterectomy for asymptomatic carotid artery stenosis. JAMA. 1995;273:1421-1428. Wang JJ, Cugati S, Knudtson MD, et al. Retinal arteriolar emboli and long-term mortality: pooled data analysis from two older populations. Stroke. 2006;37:1833-1836.

34 QUESTION

WHAT IS THE EVALUATION FOR A HOMONYMOUS HEMIANOPIA? Jonathan C. Horton, MD, PhD A 67-year-old man developed visual loss; his visual field shows a right homonymous hemianopia. Are there characteristics of the field defect or any associated symptoms that might provide a clue to the localization of the lesion? What type of imaging study should be performed and how urgently should it be done? Is there any proven treatment for homonymous visual field defects? The term homonymous hemianopia refers to a field defect caused by a lesion of the visual system posterior to the optic chiasm. When discovered as an acute finding, it is a sign of serious intracranial disease that requires emergent evaluation. The decussation of nasal retinal fibers at the optic chiasm unites in the visual pathway each eye’s representation of the contralateral hemifield of vision (Figure 34-1). Consequently, damage to the optic tract, lateral geniculate nucleus, optic radiations, or visual cortex produces an overlapping pattern of visual field loss in each eye, which usually respects the vertical meridian. Patients are sometimes unaware of a fresh hemianopia or erroneously attribute their symptoms to monocular visual loss. Careful testing of the visual fields is the crux of accurate diagnosis. Perimetry should be obtained in any patient complaining of unexplained visual loss. My preference is the Humphrey 24-2 SITA program because it provides threshold data in only a few minutes per eye. Although it does not test the peripheral field, the risk of missing a hemianopia is small because the central 24 degrees occupy most of the postchiasmal visual pathway. If computed perimetry is not available immediately, the visual fields should be tested manually, at least by finger confrontation. Hemianopia is sometimes incomplete; the degree of similarity between the pattern of field loss in each eye is referred to as the congruity. A high degree of congruity suggests a lesion in the primary (striate) visual cortex. However, this criterion is not always reliable, and many lesions involve both the optic radiations and visual cortex. In any case, localization based on the examination findings is moot because all patients with hemianopia require neuroimaging. I would refer this patient to the hospital for urgent brain scanning and inpatient neurological evaluation. Any process that can injure the brain can give rise to a homonymous hemianopia (Table 34-1). The most common etiology is stroke, which can be caused by hemorrhage, embolus, thrombosis, dissection, or vasculitis. In patients with stroke involving the middle cerebral territory, there are

143

Lee AG, ed. Curbside Consultation in Neuro-Ophthalmology: 49 Clinical Questions, Second Edition (pp 143-147). © 2015 SLACK Incorporated.

144  Question 34 Figure 34-1. Diagram of the visual pathway, with highlighting in red to show the postchiasmal regions where a lesion will produce a contralateral homonymous hemianopia.

often accompanying signs such as hemiplegia, hemianesthesia, dysphasia, or stupor. In contrast, patients with stroke from involvement of the posterior cerebral artery may have no findings except for hemianopia. In this setting, the diagnosis can be missed by the ophthalmologist who neglects visual field testing, especially given that some patients describe homonymous hemianopia as monocular vision loss and others never even notice their hemianopia. Sometimes a patient will report an episode of acute vertigo, numbness, or diplopia, suggesting an embolus that has become lodged in a posterior cerebral artery after traveling up the basilar artery. Patients who are evaluated within 4.5 hours of the onset of symptoms may be candidates for treatment with intravenous tissue plasminogen activator. In practice, few patients with isolated hemianopia receive medical care within this time frame because they do not realize the seriousness of their predicament. Nonetheless, urgent neurological evaluation is still advisable to reduce the likelihood of stroke progression or the occurrence of a second event. In fact, even if hemianopia is not present on an office examination, a reliable history of a transient ischemic attack that produced temporary hemianopic visual loss should trigger prompt referral. What type of imaging study should be obtained? Noncontrast computed tomography (CT) is low in cost, rapid, and available in any emergency department. It is excellent for excluding hemorrhagic stroke and especially suitable in the setting of head trauma. However, magnetic resonance imaging (MRI) is superior for the detection of nonhemorrhagic stroke, as well as the identification of other lesions, such as demyelinating plaques, tumors, and infections. Figure 34-2 shows the visual fields from a patient who reported a 10-day history of a shadow in her vision on the left side. CT revealed no abnormality. Regular T2-weighted MRI sequences were nearly normal (Figure 34-3A). However, fluid-attenuated inversion recovery (FLAIR) MRI sequences showed an infarct involving the lower right calcarine visual cortex in the territory of the lingual artery (Figure 34-3B). MR angiography showed partial occlusion of the proximal right posterior cerebral artery (Figure 34-3C). Diffusion-weighted MRI is the most sensitive technique for the detection of fresh cerebral ischemia. Figure 34-4 shows the visual fields from a man who reported a 1-day history of

What Is the Evaluation for a Homonymous Hemianopia?  145

Table 34-1

Causes of Homonymous Hemianopia ● ● ● ● ● ● ● ● ● ● ● ● ● ● ● ●

● ● ● ● ●

Stroke, tumor, and trauma account for more than 95% of cases Stroke (embolic, thrombotic, hemorrhagic, vasculitic, dissection) Tumor (primary or metastatic) Trauma Infection (bacterial, fungal, viral, parasitic) Arteriovenous malformation Nonorganic visual loss Demyelination Migraine Congenital malformation Perinatal hypoxic or hemorrhagic injury Progressive multifocal leukoencephalopathy Posterior reversible leukoencephalopathy Neurosurgery Occipital lobe retraction syndrome Eclampsia, syndrome of hemolysis, liver enzymes, low platelet count (HELLP syndrome) Dementia Epilepsy Mitochondrial myopathy, encephalopathy, lactic acidosis, and stroke (MELAS) Drug toxicity (cyclosporine, tacrolimus, sirolimus, cisplatin) Nonketotic hyperglycemia

Figure 34-2. Left homonymous hemianopia, mapped with a Humphrey threshold program, showing macular sparing, as well as partial sparing of the lower visual field.

146  Question 34

A

B

C

Figure 34-3. (A) Coronal T2-weighted MRI from the patient with the partial left homonymous hemianopia illustrated in Figure 34-2. The study appears virtually normal. (B) FLAIR MRI showing an ischemic stroke (arrow) involving the lower bank of the calcarine fissure, where the upper quadrant of the visual field is represented in the primary (striate) visual cortex. (C) MRA showing partial occlusion (arrow) of the right posterior cerebral artery, presumably from an embolus.

A

B

C

D

Figure 34-4. (A) Left upper quadrant scotoma subtending 10 degrees. It appears deceptively incongruous because it merges in the left eye with the blind spot. (B) Axial FLAIR MRI showing a large infarct in the right striate cortex. (C) Corresponding diffusion-weighted MRI highlighting the lesion. (D) Parasagittal T1-weighted MRI showing the infarct along the lower calcarine sulcus at the posterior pole. The line indicates the approximate plane of the axial images.

What Is the Evaluation for a Homonymous Hemianopia?  147 blurred vision in the left eye. There was a perfectly congruous parafoveal scotoma in the upper left quadrant of vision. MRI performed 2 days after the onset of symptoms showed an infarct at the right occipital pole. Although the field defect is small, the infarct is large because the representation of the central visual field is highly magnified in striate cortex. The stroke is detectable on T1-weighted and FLAIR sequences but is more obvious on the diffusion-weighted image. Another advantage of diffusion-weighted MRI is that cytotoxic edema can be detected within 30 minutes of stroke onset. It can take hours or days before any abnormality becomes visible on CT or standard T1- and T2-weighted MRI sequences. If noncontrast studies show no evidence of stroke, gadolinium should be administered. It increases the sensitivity of MRI for the detection of tumors and infections. Once the etiology of the hemianopia is established—through a review of the patient’s history, findings, and neuroimaging studies—a focused laboratory evaluation is appropriate. A stroke workup usually includes echocardiography, vascular imaging, outpatient electrocardiographic monitoring, and selected hematological studies. For tumor, a biopsy is usually performed. Removal of a solitary metastasis may be advisable, depending on the patient’s overall prognosis. Naturally, the prognosis for homonymous hemianopia depends on the etiology. For a few months after trauma or a stroke, improvement can occur spontaneously, especially in patients with an incomplete hemianopia. Surgery for a tumor, arteriovenous malformation, or infectious lesion sometimes worsens a hemianopia by damaging the surrounding brain tissue. Occasional patients with hemianopia have macular sparing, usually from collateral blood flow via the middle cerebral artery to the occipital pole, where central vision is represented. These individuals are able to read normally and adapt better to their handicap than patients with macular splitting. Unfortunately rehabilitation strategies for patients with hemianopia—based on prisms, visual exercises, or computer training—are not effective. Patients with a complete hemianopia must be told explicitly by the ophthalmologist that it is not safe or legal for them to drive. Patients with a partial hemianopia should be retested by the state motor vehicle division to assess their ability to drive.

Summary ●







Onset of homonymous hemianopia signifies the development of a serious neurological condition, which requires urgent evaluation. There are many causes of homonymous hemianopia, but stroke is the most common. Neuroimaging should be performed to identify the cause of homonymous hemianopia; diffusion-weighted MRI can detect ischemia within 1 hour of stroke onset. Exercises, prism glasses, and computer programs designed to stimulate blind regions within the visual fields are not effective for the treatment of homonymous hemianopia.

Bibliography Fraser JA, Newman NJ, Biousse V. Disorders of the optic tract, radiation, and occipital lobe. Handb Clin Neurol. 2011;102:205-221. Pambakian A, Currie J, Kennard C. Rehabilitation strategies for patients with homonymous visual field defects. J Neuroophthalmol. 2005;25:136-142. Wechsler LR. Intravenous thrombolytic therapy for acute ischemic stroke. N Engl J Med. 2011;364:2138-2146. Zhang X, Kedar S, Lynn MJ, et al. Homonymous hemianopias: clinical-anatomic correlations in 904 cases. Neurology. 2006;66:906-910.

35 QUESTION

WHAT IS THE EVALUATION FOR A PAINFUL THIRD NERVE PALSY WITHOUT A FIXED AND DILATED PUPIL BUT WITH ANISOCORIA (PARTIAL PUPILLARY INVOLVEMENT)? Michael M ich haell S S. V Vaphiades, aph hiad des DO DO A 67-year-old woman with diabetes mellitus presents with pain and a third nerve palsy OD. Her pupil is slightly bigger on the right side but is still reactive. How should this patient be managed? The management of your patient requires a review of the “rule of the pupil.” Put simply, this states that a compressive lesion of the third nerve (particularly an aneurysm) will cause a fixed, dilated pupil. In contrast, third cranial nerve palsy from an ischemic mononeuropathy (diabetes mellitus, hypertension) demonstrates no anisocoria and the pupil is briskly reactive. There are important caveats associated with these guidelines: A third cranial nerve palsy due to aneurysmal compression at or near the junction of the internal carotid and posterior communicating arteries may initially demonstrate normal pupillary size and reactivity in 14% of patients, but pupillary involvement may develop in the ensuing 7 to 10 days. ●





To be judged “pupil-sparing,” the isocoric and reactive pupil of the third cranial nerve palsy must be seen in a setting of complete ophthalmoplegia. It has been reported that an ischemic third cranial nerve palsy due to diabetes mellitus may have anisocoria up to 2.5 mm.

Third nerve palsies due to lesions in the cavernous sinus tend to be partial, and “all bets are off ” in terms of the pupillary involvement in that the pupil may be normal or minimally involved. Where does this leave us in evaluating your 67-year-old woman with diabetes mellitus with a painful third cranial nerve palsy and an enlarged yet reactive pupil? Pain may be present with both compressive and ischemic causes, so this symptom is of limited diagnostic help. One possibility is that your patient may have aneurysmal compression and, with careful follow-up, she might be found to have developed a fixed, dilated pupil. She may also have a diabetic ophthalmoplegia with a slightly dilated pupil that remains reactive. Because an intracranial aneurysm is a life-threatening condition, I would emergently order a computed tomographic angiogram (CTA) because I believe that the maximum-intensity projection images on the CTA are often better than those obtained with conventional CT scanning. You can ●

149

Lee AG, ed. Curbside Consultation in Neuro-Ophthalmology: 49 Clinical Questions, Second Edition (pp 149-150). © 2015 SLACK Incorporated.

150  Question 35 get all of the information that you need in a single study. In either case, if the conventional CT scan were normal, I would obtain a CTA; if the conventional CT were abnormal (subarachnoid hemorrhage), I would still order a CTA. I prefer the CTA because it is a low-risk test with good resolution in detecting an aneurysm. On the way to the scanner, since she may also have a vasculopathic process, I would obtain a complete blood count, metabolic panel, cholesterol, and triglycerides to survey for leukopenia, anemia, thrombocytopenia, diabetes mellitus, renal failure, and hyperlipidemia—all of which may potentially impact diagnosis and treatment. The creatinine should be done stat because the patient must have normal renal function to have a CTA. I would ask about symptoms of giant cell arteritis (jaw claudication, scalp tenderness, weight loss, fever/chills, headache, or myalgias) and consider obtaining an erythrocyte sedimentation rate and C-reactive protein. The CTA must be of high quality, done at an institution skilled with this technology and with radiologists familiar with the software. High-quality CTA may detect intracranial aneurysms as small as 3 mm in size. If the CTA is normal, a vasculopathic process is assumed. Vasculopathic risk factors must be controlled and any laboratory abnormalities addressed. If platelets are normal and there is no history of gastric ulcers, prescribe one baby (coated) aspirin daily with food. By 3 months, there should be complete or near complete resolution of the third cranial nerve palsy (rarely longer, but always less than 6 months). If, at this point, there is persistent ophthalmoplegia, other cranial neuropathies develop, or signs of aberrant regeneration are found, further imaging studies are warranted. This would include a contrasted fat-suppressed magnetic resonance imaging (MRI) scan. Some authors might start with a combination of MRI and magnetic resonance angiography (MRA); this would depend on your local institutional expertise with the various imaging modalities. If in doubt, you should consult with your local neuroradiologist on the best imaging studies at your institution. If an aneurysm is still suspected, intra-arterial catheter angiography may still be indicated. I think that the most likely cause of your patient’s third cranial nerve palsy is microvascular ischemia and that there will be complete resolution of the cranial neuropathy. However, for any patient presenting with a third cranial palsy and any anisocoria, aneurysmal compression must be considered. Such patients require meticulous clinical evaluation and judicious use of neuroimaging studies.

Summary The evaluation of patients with a third nerve palsy must be individualized. If an aneurysm is highly suspected, intra-arterial catheter angiography may still be indicated despite previous negative neuroimaging. Microvascular ischemic palsies resolve over time.

Bibliography Burde RM, Savino PJ, Trobe JD. Clinical Decisions in Neuro-Ophthalmology. 3rd ed. St Louis: Mosby; 2002. El Khaldi M, Pernter P, Ferro F, et al. Detection of cerebral aneurysms in nontraumatic subarachnoid hemorrhage: role of multislice CT angiography in 130 consecutive patients. Radiol Med. 2007;112:123-137. Glaser JS. Neuro-ophthalmology. 3rd ed. Philadelphia: Lippincott Williams & Wilkins; 1999. Jacobson DM. Pupillary involvement in patients with diabetes-associated oculomotor nerve palsy. Arch Ophthalmol. 1998;116:723-727. Jacobson DM. Relative pupil-sparing third nerve palsy: etiology and clinical variables predictive of a mass. Neurology. 2001;56:797-798. Kissel JT, Burde RM, Klingele TG, Zeiger HE. Pupil-sparing oculomotor palsies with internal carotid-posterior communicating artery aneurysms. Ann Neurol. 1983;13:149-154. Trobe JD. Isolated pupil-sparing third nerve palsy. Ophthalmology. 1985;92:58-61. Trobe JD. Managing oculomotor nerve palsy. Arch Ophthalmol. 1998;116:798. Trobe JD. Third nerve palsy and the pupil: footnotes to the rule. Arch Ophthalmol. 1988;106:601-602. Vaphiades MS, Horton JA. MRA or CTA, that’s the question. Surv Ophthalmol. 2005;50:406.

36 QUESTION

HOW DO YOU MANAGE AN ISOLATED AND PRESUMED VASCULOPATHIC, PUPIL-SPARING THIRD NERVE PALSY? Tom Carlow, MD An 80-year-old woman presented with a complete left third nerve palsy. Pupils measured 5 mm bilaterally and both reacted normally to light. She had been treated for hypertensive vascular disease for the past 25 years. The patient felt well, although she did report increasing headaches recently and some decrease in her overall level of energy.

How Would I Evaluate This Patient? I would first determine if the rest of her general ophthalmological and neurological examinations were normal. I will assume that this patient’s examination was otherwise unremarkable and that she had no history of trauma. Her long-standing hypertension would tend to support a vasculopathic or ischemic etiology for her third nerve palsy. A vasculoplastic process is the most frequent cause of an isolated pupil-sparing oculomotor palsy and is commonly associated with hypertension, diabetes mellitus, atherosclerosis, vasculitis, smoking, and an elderly age. A complete pupil-sparing, not partial, ophthalmoparetic third nerve palsy in an 80-year-old woman would also be consistent with a vasculoplastic process. Maximal external ophthalmoplegia in an ischemic oculomotor palsy is typically complete within 1 week of onset and can be preceded by periorbital pain. This patient is not described as having severe pain; however, periorbital discomfort cannot be used to separate a vasculopathic process from an aneurysm since pain can be present in both disorders. A fasting blood sugar, lipid profile, antinuclear antibody, and hemoglobin A1C should be considered. If she developed a subtle anisocoria greater than 0.5 mm while being monitored, I would have a low threshold for requesting magnetic resonance imaging (MRI) and a MR angiogram (MRA) or CT angiogram (CTA).

151

Lee AG, ed. Curbside Consultation in Neuro-Ophthalmology: 49 Clinical Questions, Second Edition (pp 151-153). © 2015 SLACK Incorporated.

152  Question 36 The cavernous sinus and subarachnoid portions of the third nerve have demonstrated ischemia and demyelination with a vasculopathic oculomotor palsy. Complete isolated third nerve palsies have been rarely documented with small midbrain infarcts. Since the patient complained of generalized fatigue, it would be extremely important to determine whether her headache was localized to her temples and whether that region was tender to the touch, suggesting a diagnosis of temporal or giant cell arteritis (GCA). Jaw, tongue, and swallowing claudication; weight loss; fever; transient visual loss; muscles aches; and any history of a transient cerebral ischemic attack would make GCA a definite diagnostic possibility. Oculomotor nerve palsy with the pupil spared or dilated can occur with GCA and may be the presenting sign. Diffuse ischemia of the extraocular muscles in GCA may result in an apparent third nerve palsy. GCA is far more common in women and increases with age. I believe that anyone over age 50, certainly this woman, with unexplained neuro-ophthalmic signs or symptoms should have an erythrocyte sedimentation rate (ESR) and a C-reactive protein (CRP). The combination of both a ESR and CRP will identify almost all patients with GCA with a higher sensitivity (ie, 99.2%) than will either test run separately. If her history was consistent with the diagnosis of GCA, corticosteroids should be started immediately and bilateral temporal artery biopsies obtained emergently. It must be remembered that not all patients with an elevated ESR, CRP, or both have GCA. An increased ESR or CRP can be seen in many disorders, including lymphoma, carcinoma, myeloma, renal disease, infection, collagen vascular disease, and after surgery. Myasthenia gravis and thyroid eye disease can mimic a pupil-sparing third nerve paresis. An ice test on the patient’s ptotic lid and an acetylcholine receptor antibody panel (binding, blocking, and modulating), as well as a thyroid screen, should be requested depending on her history. Systemic lupus erythematosus, lymphoma, leukemia, monoclonal gammopathy, syphilis, and viral inflammation may cause a pupil-sparing third nerve palsy. Complete blood count (CBC), Venereal Disease Research Laboratory test, fluorescent treponemal antibody absorption, and antinuclear antibody test should also be considered. The history of when the pupil-sparing oculomotor palsy became manifest is extremely important. If the pupil was spared only a day from the onset, especially if the paresis was not complete, the patient should be closely monitored, particularly within the first week and at weekly intervals for the first few weeks to be assured that the pupil remains spared and that an aneurysm was not responsible for the palsy. Total oculomotor extraocular motor paralysis, with the pupil spared, present for 3 months without signs of resolution should have an MRI, with and without contrast, and a MRA, or preferably a CTA. This woman’s external oculomotor palsy was complete; however, if the palsy was incomplete and present for several weeks even with the pupil spared, I would be concerned about a subarachnoid compressive lesion and give strong consideration for a MRI and a CTA. Should signs of third nerve aberrant regeneration develop (constriction of the pupil or elevation of the ptotic lid on attempted elevation, depression, or adduction on the involved side), a compressive lesion must be considered and a MRI and CTA would be in order.

How Would I Treat This Patient? I would obtain a CBC, fasting blood sugar, hemoglobin A1C, ESR, and CRP and treat if indicated. She should be instructed to call if her pupil dilates and if she develops new neurological signs or symptoms. Any atypical features or findings would lower my threshold for obtaining a MRI and a CTA. I would see her back in 1 week and then again briefly in 1 month. An MRI and, if needed, a CTA would be ordered if there were no signs of resolution at 3 months. As she improves, an eye patch and possibly a Fresnel prism could help to control her double vision.

How Do You Manage Isolated and Presumed Vasculopathic Third Nerve Palsy?  153 Controversy, as with any difficult diagnostic decision, does exist, with some authors in agreement with this paradigm, while others would suggest that a neuroimaging study might have a role in the initial evaluation of an acute complete oculomotor mononeuropathy even with the pupil spared.

Summary ●





Consider checking blood pressure and ordering laboratory studies (eg, CBC, fasting blood sugar, hemoglobin A1C, ESR, and CRP) in patients with a pupil-sparing third nerve palsy. The patient should be instructed to call immediately if the initially uninvolved pupil dilates or if there are new signs or symptoms suggestive of an aneurysm. Atypical features for ischemic palsy should prompt consideration for obtaining an MRI with MRA or CTA.

Bibliography Capo H, Warren F, Kupersmith M. Evolution of oculomotor nerve palsies. J Clin Neuro-Ophthalmol. 1992;12:21-25. Chou L, Galetta S, Liu G, et al. Acute ocular motor mononeuropathies: prospective study of the roles of neuroimaging and clinical assessment. J Neurol Sci. 2004;2219:35-39. Jacobson D. Relative pupil-sparing third nerve palsy: etiology and clinical variables predictive of a mass. Neurology. 2001;56:797-798. Kupersmith M, Heller G, Cox T. Magnetic resonance angiography and clinical evaluation of third nerve palsies and posterior communicating artery aneurysms. J Neurosurg. 2006;105(2):228-234. Parikh M, Miller N, Lee A, et al. Prevalence of normal C reactive protein with an elevated erythrocyte sedimentation rate in biopsy-proven giant cell arteritis. Ophthalmology. 2006;113(10):1842-1845. Schultz K, Lee A. Diagnostic yield of the evaluation of isolated third nerve palsy in adults. Can J Ophthalmol. 2007;42:110-115.

37 QUESTION

WHAT IS THE APPROPRIATE EVALUATION OF A FOURTH NERVE PALSY? Robert L. Tomsak, MD, PhD and Matthew J. Thurtell, MBBS, FRACP

A 50-year-old woman presents with binocular vertical diplopia. She has a leftward head tilt and a right hypertropia worse on leftward gaze. How should I evaluate this patient? Fourth nerve palsy results in weakness of the superior oblique muscle, which normally depresses and intorts the eye, and thus it can produce both vertical and torsional diplopia. Torsional diplopia, where one image is tilted relative to the other, is especially suggestive of acquired, rather than congenital, fourth nerve palsy. Asking for a detailed description of the diplopia is, therefore, the first step in the evaluation of this patient. As patients with fourth nerve palsy often report that their diplopia is increased with downward gaze (eg, while reading) and gaze to the opposite side, you should ask what exacerbates the diplopia with these points in mind. It also helps to ask about the tempo of onset of the diplopia, as this varies depending on the etiology. For example, sudden onset of vertical-torsional diplopia, with or without pain, suggests a microvascular fourth nerve palsy, while gradual onset or intermittent vertical diplopia could be due to decompensation of a long-standing fourth nerve palsy. Always enquire about a history of head trauma, as this is the most frequently identified cause of fourth nerve palsy. Ask about comorbidities, such as hypertension, diabetes mellitus, and other vascular risk factors, as their presence suggests a microvascular etiology in isolated acute onset fourth nerve palsy. It is also important to ask whether there are any other neurologic symptoms that might help to localize the lesion. Before you assess extraocular movements, note the head position. Tilting of the head to one side, to help minimize diplopia, is a valuable clue to the presence of a fourth nerve palsy on the other side. If there is a head tilt, ask to inspect old photographs; if a fourth nerve palsy is ultimately diagnosed, the presence of the head tilt in these photographs suggests that the palsy is either congenital or long-standing. After assessing extraocular movements, use the Parks-Bielschowsky three-step test to confirm the diagnosis of fourth nerve palsy. The first step of this test is to determine the side of the hypertropia. The next step is to show that the hypertropia increases

155

Lee AG, ed. Curbside Consultation in Neuro-Ophthalmology: 49 Clinical Questions, Second Edition (pp 155-157). © 2015 SLACK Incorporated.

156  Question 37 Figure 37-1. This woman presented with vertical-torsional diplopia and was noted to have a right hypertropia that increased with leftward gaze. The hypertropia was also (A) increased by head tilt to the right, while it was (B) minimized by head tilt to the left, confirming the diagnosis of right fourth nerve palsy.

A

B

with adduction. The final step is to show that the hypertropia also increases with head tilt to the ipsilateral side (Figure 37-1A). In contrast, the hypertropia will be minimized with head tilt to the contralateral side (Figure 37-1B). In a patient with right hypertropia due to a right fourth nerve palsy, you would expect the vertical deviation to be greatest with leftward gaze and with head tilt to the right. In subtle cases, it may be helpful to quantify the deviation at each stage of the test using vertical prisms or the Maddox rod. Prisms can also be used to demonstrate an increased vertical fusion amplitude (of more than 10 prism diopters), a finding that is often associated with long-standing fourth nerve palsy. Extorsion of the affected eye(s) can be demonstrated and quantified with double Maddox rods. Alternatively, use of Lancaster red-green glasses, examination of the fundus, or fundus photographs may confirm that the eye is extorted. Beware of the patient with unilateral superior oblique weakness who fixates with the paretic eye. These patients may appear to have a contralateral superior rectus palsy when versions are tested due to the phenomenon of inhibitional palsy of the contralateral antagonist. Ductions of the contralateral superior rectus muscle usually will be normal. Many patients with fourth nerve palsy due to head trauma actually have bilateral fourth nerve palsies. They will usually have hypertropia of each eye on adduction (ie, a right hypertropia with leftward gaze and a left hypertropia with rightward gaze) and the Parks-Bielschowsky test might be positive bilaterally. A measured torsional misalignment of more than 10 degrees on double Maddox rod testing is particularly suggestive of bilateral fourth nerve palsies. During the examination, look for signs that help to localize the lesion or suggest another cause for the diplopia. For example, a lesion in the region of the trochlear nucleus in the midbrain might produce an ipsilateral Horner syndrome, as well as a contralateral fourth nerve palsy. The presence of other cranial nerve palsies ipsilateral to a fourth nerve palsy suggests a cavernous sinus, superior orbital fissure, or orbital apex lesion. The presence of brainstem signs or intorsion of the hypertropic eye suggests a skew deviation rather than a fourth nerve palsy. The presence of areflexia or ataxia may suggest Miller-Fisher syndrome. The presence of fluctuating signs, such as ptosis, suggests myasthenia gravis, while the presence of exophthalmos or lid retraction suggests thyroid eye disease. Neuroimaging is not essential in patients with congenital fourth nerve palsy, isolated fourth nerve palsy due to head trauma, or isolated acute onset fourth nerve palsy in association with vascular risk factors. Neuroimaging should be performed in all other patients and in those with a presumed microvascular etiology who do not recover or begin to improve within 3 months, although it will rarely be abnormal if there are no other neurologic signs. In patients older than age 50 years with a presumed microvascular etiology, you should ask about symptoms of giant cell arteritis and investigate urgently if any are present. Treatment is directed toward the underlying cause. Although most patients with a microvascular fourth nerve palsy will completely recover within 3 months, you should address their vascular risk factors and consider starting low-dose aspirin to reduce the risk of future vascular events. A Fresnel prism can minimize diplopia in patients who do not completely recover. Alternatively,

What Is the Appropriate Evaluation of a Fourth Nerve Palsy?  157 patients with disabling diplopia that is both long-standing and stable could be offered surgery; inferior oblique myectomy or recession is the most appropriate procedure in these cases.

Summary ●









Fourth nerve palsy, when unilateral, results in an ipsilateral hypertropia that increases with adduction and head tilt toward the affected side. Most isolated fourth nerve palsies are posttraumatic, congenital (but may decompensate and become symptomatic later in life), or microvascular in etiology. Bilateral fourth nerve palsies are most often posttraumatic and present with alternating hypertropia (ie, right hypertropia on left gaze and left hypertropia on right gaze) and significant torsional misalignment. Neuroimaging should be performed in patients with progressive fourth nerve palsy or presumed microvascular fourth nerve palsy that does not recover within 3 months. Treatment of fourth nerve palsy depends on the etiology and degree of ocular misalignment.

Bibliography Brazis PW. Palsies of the trochlear nerve: diagnosis and localization – recent concepts. Mayo Clin Proc. 1993;68:501-109. Dickey CL, Scott WE, Cline RA. Oblique muscle palsies fixating with the paretic eye. Surv Ophthalmol. 1988;33:97-107. Lemos J, Eggenberger E. Clinical utility and assessment of cyclodeviation. Curr Opin Ophthalmol. 2013;24:558-565. Mollan SP, Edwards JH, Price A, Abbott J, Burdon MA. Aetiology and outcomes of adult superior oblique palsies: a modern series. Eye. 2009;23:640-644. Tamhankar MA, Biousse V, Ying GS, et al. Isolated third, fourth, and sixth cranial nerve palsies from presumed microvascular versus other causes: a prospective study. Ophthalmology. 2013;120:2264-2269.

38 QUESTION

WHAT IS THE APPROPRIATE EVALUATION IN A PATIENT SUSPECTED OF HAVING A SIXTH NERVE PALSY? Steven R. Hamilton, MD A 60-year-old man has binocular horizontal diplopia. He does not abduct OD well and has an incomitant esotropia. Now what? The first step in evaluating a patient with diplopia is to acquire a solid and focused history. Your patient should describe binocular diplopia that is predominantly horizontal, present at distance more than at near, and typically worse looking to one side. There may be diplopia only on looking to one side, as while driving, and none with reading if the palsy is mild. Worrisome symptoms for intracranial disease, such as tumor, include intermittent or gradual onset, particularly with progression, persistent pain over weeks, ipsilateral facial numbness or weakness, hearing changes, or vertigo. One usually suspects a sixth nerve palsy based on the patient’s history and the initial finding on examination of extraocular motility of an abduction deficit. It is important to be aware that all abduction deficits are not due to sixth nerve palsy and that a number of other possibilities should be considered (Table 38-1). Those occurring most frequently are a restrictive ophthalmopathy due to thyroid eye disease or myopathy from myasthenia gravis. You should look for orbital signs such as proptosis, chemosis, and lid edema during the examination. If abducting saccades are not obviously slow, one should perform forced ductions to rule out a restrictive process or consider orbital imaging with high-resolution computed tomography (CT) or magnetic resonance imaging (MRI). Myasthenia can mimic any cranial nerve palsy; you should consider checking for a binding type of acetylcholine receptor antibody level or performing a Tensilon test, particularly if there is any history or finding of ptosis of either or both eyes. In my experience, myasthenia rarely presents with an isolated abduction deficit; it more commonly affects adduction, mimicking an internuclear ophthalmoplegia or partial third nerve palsy. Once you determine that the patient has a probable sixth nerve palsy, the workup should proceed based on the patient’s age, the duration of symptoms, and the presence of any other worrisome symptoms (as noted previously) or orbital or other neurological signs. The latter would include any other cranial nerve abnormalities or dysfunction of the oculosympathetic pathway

159

Lee AG, ed. Curbside Consultation in Neuro-Ophthalmology: 49 Clinical Questions, Second Edition (pp 159-162). © 2015 SLACK Incorporated.

160  Question 38

Table 38-1

Etiology of Abduction Deficit ● ● ● ● ● ● ● ●

Sixth nerve palsy Restrictive ophthalmopathy, such as thyroid eye disease Ocular myasthenia gravis Congenital: Duane or Möbius syndrome Nonspecific orbital inflammation (pseudotumor) Entrapment of the medial rectus muscle from trauma Spasm of the near reflex Giant cell arteritis

(Horner syndrome). If other neurologic signs are present, the patient should undergo a contrastenhanced brain MRI. If there are any orbital signs such as proptosis, injection, chemosis, or a bruit, you should obtain an orbital and brain MRI with contrast, as well as magnetic resonance angiography (MRA) and magnetic resonance venography (MRV). If the patient is 50 years of age or older with a sudden onset of symptoms and no orbital or neurological signs other than a sixth nerve palsy and has known vasculopathic risk factors (eg, diabetes mellitus, hypertension, or hyperlipidemia), you can presume an ischemic basis to the palsy. Be aware, however, that there may be up to a 5% possibility of another etiology in this group based on a recently published multicenter prospective series of patients. Regular follow-up checks at monthly intervals should be done documenting gradual resolution of the palsy. If any new signs or symptoms develop during this time or the palsy does not resolve within 3 months or progresses, a brain MRI with contrast must be performed. You should also consider screening any patient older than age 50 years for the possibility of giant cell arteritis and obtain a Westergren erythrocyte sedimentation rate and C-reactive protein. Never forget that a patient is allowed only one ocular motor nerve palsy at a time on the basis of benign ischemia. Multiple cranial nerve palsies demand prompt neuroimaging with MRI with contrast with careful attention to the region of the superior orbital fissure and cavernous sinus. If there is any concern about an orbital process in a patient without known vascular risk factors, I believe that careful neuroimaging with MRI of the brain including contrast and orbital views is mandatory. This comprises all sixth nerve palsies in young adults and children since a high percentage of childhood sixth nerve palsies are secondary to compression from tumor (Table 38-2). MRA should be considered on occasion even with a normal brain MRI if aneurysm is suspected. A special case should be made for bilateral sixth nerve palsies, even if relatively asymmetrical in degree. This finding should raise concern for increased intracranial pressure or meningeal disease (eg, inflammation, infection, neoplasm) and warrant immediate MRI with contrast; if the result is negative, probably a diagnostic lumbar puncture with assessment of opening pressure and cerebrospinal fluid profile should be done. Management of a transient sixth nerve palsy may include temporary patching of the eye or fogging of a spectacle lens with semiopaque tape. I have found that ischemic sixth nerve palsies do not really need a temporary prism, given their short duration. For chronic sixth nerve palsy, prisms may be helpful, although ultimately strabismus surgery may be the best option. Surgery should not be considered until the palsy has been followed and remains stable for 6 to 12 months. I have had

What Is the Appropriate Evaluation in a Patient With a Sixth Nerve Palsy?  161

Table 38-2

Localization of Nonisolated Sixth Nerve Palsies Sixth Nerve Palsy With the Following:

Localization

Examples

Ipsilateral gaze palsy with or without ipsilateral seventh nerve palsy

Sixth nucleus in the pons

Stroke, tumor, multiple sclerosis

Ipsilateral fifth or seventh nerve palsy, Horner syndrome, or contralateral hemiparesis

Sixth nerve fascicle in the pons

Stroke, tumor, multiple sclerosis

Ipsilateral deafness, vertigo, fifth or seventh nerve palsy

Cerebellopontine (CP) angle

Tumor in CP angle

Ipsilateral otitis media with or without mastoiditis signs

Petrous apex

Mastoiditis

Ipsilateral third, fourth, or fifth nerve palsy or Horner syndrome

Cavernous sinus

Tumor, vascular fistula, thrombosis, infection, inflammation, aneurysm

Optic nerve dysfunction with or without third, fourth, or fifth cranial nerve dysfunction

Orbital apex

Tumor, infection, inflammation, carotid cavernous sinus fistula

Orbital signs (eg, proptosis)

Orbit

Tumor, infection, inflammation

Bilateral palsies

Nonlocalized

Often due to increased intracranial pressure or meningeal processes

Adapted from Pane A, Burdon M, Miller NR. The Neuro-ophthalmology Survival Guide. St. Louis, MO: Mosby; 2006:230-231.

good success with surgery even for compressive etiologies such as cavernous sinus meningiomas, which are considered inoperable. Botulinum toxin can be used as a temporizing measure, but the diplopia will return as it dissipates.

Summary ●



Sixth nerve palsy is one of many causes of an abduction deficit; one must be familiar with the differential diagnosis. Brain MRI (with or without orbital views) is indicated in all patients with acute sixth nerve palsies who have no known vascular risk factors.

162  Question 38 ●



Management of a transient (eg, ischemic) sixth nerve palsy may include temporary patching of the eye or fogging of a spectacle lens with semiopaque tape. Strabismus surgery may be the best option for patients with chronic deviations who fail or are intolerant of conservative measures.

Bibliography Glaser JS, Bachynski B. Infranuclear disorders of eye movement. In: Glaser JS. Neuro-ophthalmology. 2nd ed. Philadelphia: JB Lippincott; 1990:366. Miller NR, Newman NJ, Biousse V, Kerrison JB. Nuclear and infranuclear ocular motility disorders. In: Walsh and Hoyt’s Clinical Neuro-Ophthalmology: The Essentials. 2nd ed. New York: JB Lippincot; 2008:404. Pane A, Burdon M, Miller NR. The Neuro-Ophthalmology Survival Guide. St. Louis: Mosby; 2006:230-231. Tamhankar MA, Biousse V, Ying GS, et al. Isolated third, fourth, and sixth cranial nerve palsies from presumed microvascular versus other causes. Ophthalmology. 2013;120:2264-2269.

39 QUESTION

HOW DO I EVALUATE A PATIENT WITH MULTIPLE OCULAR MOTOR CRANIAL NERVE PALSIES? Marc Dinkin, MD and Gregory S. Kosmorsky, DO

A 65-year-old woman presents with headaches and binocular oblique double vision. Neuroophthalmological examination reveals limitation of abduction of both eyes. Furthermore, the left eye will not adduct, elevate, or depress, and there is left ptosis. The left pupil is 2 mm larger than the right. What should I do now? This patient presents with what appears to be bilateral abducens palsies and a left, pupilinvolving ocular motor palsy. Paresis of each of these nerves alone would generate a list of specific diagnoses. For example, ocular motor nerve palsies in patients of this age group are often microvascular, especially if there is a history of hypertension or diabetes mellitus; however, when the pupil is involved, compression by a posterior communicating artery aneurysm must be urgently ruled out. Abducens palsies may also be ischemic, but when they occur bilaterally, you have to consider increased intracranial pressure, which tends to compress or stretch the nerve as it courses through the Dorello canal, a short tunnel that leads it from the subarachnoid space to the cavernous sinus. In this case, however, how can we explain involvement of all three nerves? When we see damage to some combination of cranial nerves III, IV, and VI, we think of four possible localizations, the first being the brainstem. Although each nerve travels at a different level (III, upper midbrain; IV, lower midbrain; VI, pons), multifocal diseases, such as demyelination, metastases, and patchy ischemia, and necrosis from thiamine deficiency could all cause simultaneous dysfunction. In Keane’s analysis of 979 cases of multiple cranial neuropathies (509 of which included two or more ocular motor palsies), 22% were due to brainstem disease, but only one of these cases lacked neighborhood signs (upper motor neuron weakness, internuclear ophthalmoplegia, or nystagmus) that clearly identified the brainstem as the site of injury. Therefore, as a rule, if you see multiple ocular motor palsies but no specific brainstem signs, the localization is likely elsewhere. Utilize plain T2-weighted magnetic resonance imaging (MRI) to visualize brainstem disease, as fluid-attenuated inversion recovery (FLAIR) MRI sequences are less sensitive and computed tomography (CT) is obfuscated by bony artifact. Upon leaving the brainstem, the ocular motor nerves all travel through the subarachnoid space. Any disease that affects the cerebrospinal fluid may cause cranial nerve dysfunction, including

163

Lee AG, ed. Curbside Consultation in Neuro-Ophthalmology: 49 Clinical Questions, Second Edition (pp 163-167). © 2015 SLACK Incorporated.

164  Question 39 infections (Lyme disease, syphilis, and fungal meningitis), inflammation (sarcoidosis), and neoplasm (lymphoma, leukemia, and carcinomatous meningitis). In Keane’s review, only 10% of cases resulted from involvement of the subarachnoid space. The acute inflammatory demyelinating neuropathies, including Guillain-Barré syndrome and the Miller Fisher variant, cause generalized nerve injury, but their effects on the nerves as they traverse the subarachnoid space are evidenced by elevated protein in the cerebrospinal fluid. Red flags for disease of the subarachnoid space include peripheral nerve involvement and signs of elevated intracranial pressure, such as papilledema. The three nerves finally come together in the cavernous sinus (CS), where cranial nerves III and IV travel along the lateral wall and cranial nerve VI moves in the substance of the cavernous sinus and alongside the internal carotid artery and sympathetics. In Keane’s review, 25% of the cases were due to CS disease, but this percentage would likely be much higher had he confined his analysis to ocular motor palsies. Whenever ocular nerve palsies are accompanied by dysfunction of the first two branches of the trigeminal nerve or a Horner syndrome, think of the CS. Also, look for signs of poor venous drainage from the eye: elevated intraocular pressure, injection, and chemosis. In cases of carotid-cavernous fistulas, you might see a conjunctival corkscrew vessel or hear an orbital bruit with your stethoscope. Other entities include tumors (meningiomas or large pituitary adenomas), idiopathic inflammation, and CS thrombosis, which may reflect a life-threatening infection. Pituitary apoplexy may cause an acute chiasmopathy along with ophthalmoplegia. When CS disease is suspected, ask your radiologist for thin cuts through the sinus with and without contrast. We find coronal T2-weighted imaging to be the most useful since the nerves can be well visualized. As the ocular motor nerves leave the CS, they coalesce at the orbital apex. The orbital apex syndrome can look much like the CS syndrome in that cranial nerves III, IV, and VI and the sympathetic fibers may all be affected, but the difference is that V2 is spared and the optic nerve may be involved. Look for proptosis or resistance to retropulsion as evidence of a retrobulbar lesion. Orbital apex tumors include benign meningiomas and hemangiomas but also metastases and lymphoma. The most menacing possibility, however, is mucormycosis, which must be considered in any patient who is diabetic or immunosuppressed. This process often affects diabetics with ketosis, and an eschar may be seen in the nose. Rapid disease progression and death will ensue if the condition is left untreated. Finally, don’t forget the mimickers of multiple ocular motor palsies. Orbital disease can produce ophthalmoparesis in multiple axes, suggestive of involvement of cranial nerves III, IV, and/ or VI, when in fact the disease is in the extraocular muscles themselves. Look for lid retraction and a delay of lid depression with downward gaze (lid lag) instead of ptosis, as well as proptosis, engorgement of vessels over the extraocular muscles, and periorbital edema as evidence of thyroid eye disease. Idiopathic inflammation, tumors, and orbital varices may also be responsible. Trauma to the bony orbit may entrap and restrict multiple extraocular muscles, but as with many orbital entities, the eye will not move with forced ductions. Contrast-enhanced MRI with thin cuts through the orbit and fat saturation is ideal. We have seen many cases where orbital pathology is missed by brain MRI alone. Myasthenia gravis (MG) may also mimic any single or combination of ocular motor palsies. Fatigable motility and ptosis are suggestive, as are curtaining and improvement with rest, ice, or intravenous edrophonium (Tensilon) or pyridostigmine. Look for evidence of systemic weakness and consider obtaining acetylcholine receptor antibody titers. A decremental response with repetitive nerve stimulation suggests MG, but single-fiber electromyography remains the most sensitive test, albeit not widely available. See Table 39-1 for a summary of the associated symptoms and signs as well as the differential diagnosis of multiple ocular motor palsies, organized by location. As for our 65-year-old woman with bilateral abducens palsies and a left ocular motor palsy, see Figure 39-1 for further work up and diagnosis.

How Do I Evaluate a Patient With Multiple Ocular Motor Cranial Nerve Palsies?  165

Table 39-1

Acquired Multiple Ocular Motor Cranial Nerve Palsies: Differential Diagnosis and Signs by Location Location

Differential Diagnosis

Associated Symptoms and Signs

Brainstem

Brainstem stroke Hemorrhage Paraneoplastic disease Demyelination Tumor Thiamine deficiency

Nystagmus Internuclear ophthalmoplegia Corticospinal tract dysfunction Oculopalatal myoclonus Facial myokymia

Subarachnoid Sarcoidosis space Syphilis Lyme disease Lymphoma/leukemia Carcinomatosis

Headache Papilledema Photophobia Pulsatile tinnitus Systemic peripheral neuropathies

Cavernous sinus

Headache Corkscrew conjunctival vessels Orbital bruit Increased intraocular pressure Chemosis Horner syndrome Numbness along V1 and V2

Vascular Intracavernous carotid artery aneurysm Carotid-cavernous sinus fistula Carotid sinus thrombosis Neoplasm Primary intracranial tumor Pituitary adenoma Meningioma Craniopharyngioma Hemangioma Local metastases Nasopharyngeal tumor Squamous cell carcinoma Distant metastases Lymphoma Multiple myeloma Carcinoma Infectious Bacterial Sinusitis Mucocele Periostitis ● ● ● ●

● ●

● ● ●

● ● ●

(continued)

166  Question 39

Table 39-1 (continued)

Acquired Multiple Ocular Motor Cranial Nerve Palsies: Differential Diagnosis and Signs by Location Location

Differential Diagnosis

Associated Symptoms and Signs

Viral Herpes zoster Fungal Mucormycosis Spirochetal Treponema pallidum Mycobacterial Mycobacterium tuberculosis Inflammatory Sarcoidosis Wegener granulomatosis Tolosa-Hunt syndrome ●







Orbital apex/ superior orbital fissure

Idiopathic orbital inflammation Contiguous sinusitis Mucormycosis or other fungal infection Meningioma Metastatic tumor Lymphoma/leukemia

Horner syndrome Optic neuropathy Proptosis Orbital hypoesthesia

Orbital (mimicking cranial nerve palsies)

Thyroid eye disease Idiopathic orbital inflammation Tumor Orbital varix Trauma Myopathy Myasthenia gravis (at neuromuscular junction)

Proptosis Resistance to retropulsion Positive forced duction test Increased intraocular pressure on upgaze

Diffuse, nonlocalizable

Diabetic ophthalmoplegia Giant cell arteritis Guillain-Barré/Miller Fisher syndrome

Diabetic retinopathy Headache, scalp tenderness Areflexia, weakness, or ataxia

How Do I Evaluate a Patient With Multiple Ocular Motor Cranial Nerve Palsies?  167 Figure 39-1. Let us now return to our original case: The lack of brainstem or cavernous sinus signs and presence of a headache suggested a meningeal etiology. A contrast MRI was therefore obtained and showed enhancement of multiple cranial nerves including III (A, yellow arrows), IV, and VI (B), all affected bilaterally. The trigeminal nerve (V) also enhanced avidly (C). Lumbar puncture was therefore performed and revealed a pleocytosis with leukemic cells. With appropriate chemotherapy, the diplopia resolved. Note the diffusion positive signal in the lacrimal glands (D), a sign suggestive of infiltration by leukemia or lymphoma.

Summary ●







In the setting of multiple cranial neuropathies, neuroimaging is mandatory. Do not make your radiologist play “guess what I’m thinking” but instead specify the topographic area of concern and the diagnostic possibilities. Look for signs and symptoms that can point toward the brainstem, subarachnoid space, cavernous sinus, or orbital apex and guide your choices for imaging and serological studies. Contrast-enhanced cranial MRI is the initial diagnostic procedure of choice, but include thin cuts through the cavernous sinus when its involvement is suspected. Screen for mimickers such as ocular myasthenia or orbital muscle disease.

Bibliography Bennett JL, Pelak VS. Palsies of the third, fourth, and sixth cranial nerves. Ophthalmol Clin North Am. 2001;14(1):169-185. Bianchi-Marzoli S, Brancato R. Third, fourth, and sixth cranial nerve palsies. Curr Opin Ophthalmol. 1997;8(6):45-51. Eshaugh CG, Siatkowski RM, Smith JL, Kline LB. Simultaneous, multiple cranial neuropathies in diabetes mellitus. J Neuroophthalmol. 1995;15(4):219-224. Hamilton SR. Neuro-ophthalmology of eye movement disorders. Curr Opin Ophthalmol. 1999;10(6):405-310. Johnston JL. Parasellar syndromes. Curr Neurol Neurosci Rep. 2002;2(5):423-431. Keane JR. Multiple cranial nerve palsies. Analysis of 979 cases. Arch Neurol. 2005;62:1714-1717.

40 QUESTION

WHAT IS BLEPHAROSPASM? Sushma Yalamanchili, MD and Angelina Espino, MD

A 60-year-old woman with no significant medical history presents to your clinic with uncontrolled blinking. She started noticing the symptom 5 years earlier, but it was infrequent. However, now the excessive blinking is causing significant visual impairment. The blinking is sometimes accompanied by cocontraction of the facial muscles. How should this patient be evaluated? Is an imaging study warranted? What treatments can be offered? Blepharospasm refers to a spasm of the eyelid or, more specifically, the preseptal, pretarsal, and periorbital orbicularis oculi. There are many causes of blepharospasm, including more generalized dystonias, hemifacial spasm, and ocular irritation (such as dry eye disease). When it occurs primarily and in isolation, the term benign essential blepharospasm (BEB) is applied. BEB is a focal dystonia, or involuntary muscle contraction, of the orbicularis oculi without any associated neurological lesion. BEB commonly affects women in the fifth to seventh decades of life and begins gradually. It affects 20,000 to 50,000 people in the United States, with a prevalence of 1.2 to 5 per 100,000. Most cases are bilateral from onset. However, they may be asymmetrical or initially unilateral, with fellow eye involvement following shortly thereafter. At onset, BEB may be minimally symptomatic and is often first noticed by a spouse or family member. Most cases present with photophobia or ocular irritation; these sensory symptoms may even precede the development of the eyelid spasms. Initially, frequent blinking may last anywhere from seconds to 20 minutes. However, with time, frequent blinking proceeds to involuntary spasms and forceful contractions of the orbicularis, rendering the patient functionally blind. The orbicularis oculi muscles are not exclusively affected in BEB. In some cases, episodes of BEB may be associated with spasms of the procerus and corrugator supercilii muscles. Disease progression varies greatly. In some cases, it never advances beyond being a nuisance, whereas in others it can cause significant functional and social impairment. The most common exacerbating factor is light and these patients may find some relief with filtered glasses. Other triggers include wind, noise, eye and head movements, stress, television, and reading. Patients sometimes develop tricks that reduce the frequency and severity of spasms, such as yawning, whistling, chewing, coughing, eating, neck stretching, rubbing the eyelids, and covering the eyes.

169

Lee AG, ed. Curbside Consultation in Neuro-Ophthalmology: 49 Clinical Questions, Second Edition (pp 169-172). © 2015 SLACK Incorporated.

170  Question 40 Spasms also disappear during sleep. The majority of patients with BEB present with additional spasms of the face or body. Since the nature of the disease is progressive, some patients later develop spasms of the lower face and neck (Meige syndrome) or dystonias affecting other facial regions (segmental cranial dystonia or craniocervical dystonia). The cause of BEB is not clear. Historically, it was believed to be psychogenic in origin. However, BEB is now known to be an organic and multifactorial disease, with both environmental and genetic factors contributing to its onset. A defect in the neurological circuit activity—which includes a sensory, a motor, and a central control component—has been proposed. Several environmental factors, such as light or ocular surface irritation, activate the sensory mechanism, which then sends impulses to the control center in the central nervous system. A defect in this central control mechanism secondarily activates the motor component, which includes the facial nucleus, facial nerve, and facial muscles innervated by its temporal division (orbicularis oculi, corrugator, and procerus muscles). Within the central nervous system, most studies implicate the basal ganglia and associated dopamine insufficiency as the underlying cause of BEB. One third of patients with BEB have at least one first- or second- degree family member with a movement disorder, suggesting a genetic predisposition in some patients. Evaluation of blepharospasm should begin with a complete history to identify the potential causative factors while eliminating other disease entities from the differential diagnosis. On examination, involuntary contraction of the orbicularis oculi muscle should be observed. If the muscular spasms are restricted to one side of the face, the diagnosis is likely to be hemifacial spasm and not BEB. If spasms are present in only one group of fibers within the orbicularis oculi, the diagnosis is then ocular myokymia. If bilateral eyelid spasms are associated with spasms of the midface, the accurate diagnosis is Meige syndrome (also known as idiopathic cranial-cervical dystonia). Difficulty opening the eyelids when the orbicularis oculi is not in forceful contraction might indicate apraxia of the eyelid opening. The latter can be a feature of BEB but may also occur as an isolated finding. Medications that can be associated with dystonia should also be recorded. Common ones include those used to treat Parkinson’s disease and most neuroleptics. Initial evaluation should include a careful ocular examination. Assessment for dry eye disease includes slit-lamp evaluation of the cornea and measurement of tear production. Misdirected lashes (trichiasis and distichiasis), blepharitis, and signs of allergic conjunctivitis should be noted. If any of the above signs are detected, blepharospasm may be secondary to irritation. Such irritants may exacerbate the disease even in patients with BEB. A complete neurological history should be taken and patients should be examined with special attention to the cranial nerves. Compressive lesions of the facial nerve, and even less commonly brainstem neoplasm, may result in spasms of the facial musculature, including the orbicularis oculi. In such cases, dysfunction of adjacent nerves can be encountered. In cases associated with any other neurological findings, further evaluation is warranted. In summary, the diagnosis of BEB is one of exclusion. A careful history and examination should rule out other dystonic movement disorders that may simulate blepharospasm. BEB can then be confirmed by a negative neurological examination and unremarkable neuroimaging. Our patient is also experiencing occasional facial spasms, which raises the possibility of a more diffuse process. The two syndromes that should be included in the differential are Meige syndrome and hemifacial spasm. Meige syndrome, or craniocervical dystonia, is a bilateral movement disorder that, in addition to blepharospasm, can involve muscular contractions of the jaw muscles (jaw clenching or mouth opening), lips (grimacing), and tongue (protrusion). However, patients with blepharospasm can have variable degrees of facial and neck involvement; therefore, the distinction between BEB and Meige syndrome is not always clear. These disorders may prove to represent extremes of the same disease. Regardless, degenerative diseases and compressive lesions can present with movement disorders resembling Meige syndrome, and an imaging study might be considered.

What Is Blepharospasm?  171 On the other hand, the distinguishing feature of hemifacial spasm is that it is invariably unilateral and, unlike BEB, the spasms are present during sleep. Muscles supplied by the facial nerve are affected. Spasms of the lower face (orbicularis oris) and neck (platysma) are common. An imaging study should also be ordered in this case as a small aberrant artery may be compressing the root of the facial nerve as it exits the brainstem. Blepharospasm is a chronic, progressive, debilitating disease. Its management is complex; the most effective conventional treatments include education and support (provided by the Benign Essential Blepharospasm Research Foundation [BEBRF]), pharmacotherapy, botulinum toxin (Botox) injections, and surgical intervention. The BEBRF was established in 1981 to search for a cure for BEB and related disorders. The foundation disseminates knowledge and promotes awareness throughout the world. It has been shown to help patients with BEB more than any other resource. The first step in the treatment of blepharospasm should address any underlying ocular irritation. Eyelid hygiene to decrease irritation and treat blepharitis should be encouraged. Dry eye treatment has also been shown to help patients. Consideration can also be given to rose-tinted glasses, which block ultraviolet light and decrease light sensitivity. Although controversial, chromatic lenses (FL-41) designed specifically for the treatment of blepharospasm are commercially available. Botulinum toxin injection is the mainstay of therapy for most patients with BEB. This toxin blocks the release of acetylcholine from the presynaptic nerve terminal of the neuromuscular junction and, therefore, weakens the muscles. Xeomin is an FDA-approved form of botulinum toxin. Muscle weakness starts 2 to 7 days after injection and lasts at least 3 months in 90% of patients. Numerous variations of dosage and injection location have been proposed. However, the precise treatment strategy is tailored based on spasm location. For example, some may require treatment of the brow depressors (glabella), whereas others may not. Those with more muscle mass or stronger contractions may require higher doses. A typical starting dose is 10 units per eye and, if needed, 20 units for the medial brow depressors. It is recommended that the starting dose should not exceed 20 units per eye to avoid side effects. If this is insufficient, the dose is increased with subsequent treatments. The interval of treatment varies and should be tailored to individual patient needs. As previously mentioned, injected botulinum toxin is usually effective for 3 months in most patients; however, it sometimes lasts up to 6 months or more. The control center involved in blepharospasm is unknown; therefore, targeted pharmacological therapy is still not available. Current drug therapy is designed on the basis of the three proposed neurotransmitter dysfunctions: cholinergic excess, GABA hypofunction, and dopamine excess. There have been only inconsistent anecdotal reports of successful treatment with systemic medications. The effects, however, are partial and short-lived. Pharmacotherapy is thus worth trying in patients resistant or opposed to treatment with botulinum toxin. More commonly used medications include benzodiazepines (clonazepam, diazepam, lorazepam, oxazepam), gabapentin, baclofen, carbamazepine, levodopa and carbidopa, and benztropine. Surgery may be considered as a last resort for patients with debilitating disease who fail maximal medical therapy or cannot tolerate it. Orbicularis oculi myectomy is effective when correctly performed. It consists of removing the pretarsal, preseptal, and the orbital portions of the orbicularis oculi muscle. However, it can be disfiguring and is fraught with complications. Simple removal of excess eyelid skin (blepharoplasty) may allow the brow to be recruited for eyelid elevation. However, the effect of this is limited and probably inconsequential in most severe cases.

172  Question 40

Summary ●

BEB is a focal dystonia, or involuntary muscle contraction, of the orbicularis oculi without any associated neurological lesion.



BEB commonly affects women in the fifth to seventh decades of life and begins gradually.



Its cause is unknown but it is thought to involve a dysfunction in the central control circuit.



Evaluation should include a complete history, as well as a complete neurological and ophthalmic exam. Some patients may warrant an imaging study to rule out compressive lesions.



The differential diagnosis includes lid myokymia, Meige syndrome, and hemifacial spasm.



Treatment should begin with patient education and support.





Botulinum toxin (Botox, Xeomin) injections are the first-line interventional treatment. Pharmacotherapy can be used as second-line treatment. Surgical treatments are generally considered as a last resort.

Bibliography Baker RS, Andersen AH, Morecraft RJ, Smith CD. A functional magnetic resonance imaging study in patients with benign essential blepharospasm. J Neuroopthalmol. 2003;23:11-15. Ben Simon GJ, McCann JD. Benign essential blepharospasm. Int Ophthalmol Clin. 2005;45(3):49-75. Coscarelli JM. Essential blepharospasm. Semin Ophthalmol. 2010;25(3):104-108. McCann JD, Ugurbas SH, Goldberg RA. Benign essential blepharospasm. Int Ophthalmol Clin. 2002;42(2):113-121.

41 QUESTION

WHAT IS HEMIFACIAL SPASM? Andrew R. Harrison, MD; Benin Barahimi, MD; and Michael S. Lee, MD

The patient is a 40-year-old man who reports a 2-year history of episodic involuntary closure of his left eye and a 6-month history of contractions of the left side of his face. His general health is excellent and he takes no medication regularly. With the exception of the involuntary movement of the left side of his face, his examination is normal. What evaluation is necessary? We think that your patient might have hemifacial spasm (HFS), which manifests as involuntary episodic contractions of the muscles innervated by the facial nerve. It is almost always unilateral and the peak presentation occurs between the ages of 40 to 60 years; women are more commonly affected. HFS often begins with isolated involvement of the orbicularis oculi and over time gradually spreads to the ipsilateral facial muscles. The spasms tend to be synchronous (ie, all of the muscles contract simultaneously). Coinciding contraction of the orbicularis oculi and frontalis muscle (also known as the Babinski-2 sign) is highly specific for HFS and results in eyelid closure with simultaneous brow elevation. If the zygomaticus major and minor muscles also become involved, the angle of the mouth will elevate as well. Spasms increase in frequency with stress, fatigue, anxiety, and voluntary facial contractions. You can often bring out the spasms by having the patient forcefully close his or her eyes. One of the features distinguishing HFS from other involuntary facial movements is that the spasms continue during sleep. The most common cause of HFS is compression of the facial nerve by an anomalous or aberrant vessel as it exits the brainstem. Although patients with HFS initially appear normal between contractions, they may develop facial weakness with long-standing HFS due to compression of the seventh nerve. Posterior fossa tumors have been found in less than 1%. In making the diagnosis of HFS, other involuntary facial movements should be ruled out. Unilateral twitching of the facial muscles may occur following a seventh nerve palsy secondary to aberrant regeneration of the peripheral nerve fibers with synkinesis. The patient will have a history of Bell’s palsy and, if you look closely, you will note that the contractions tend to be asynchronous. For example, eye closure may occur moments before the corner of the mouth elevates, or vice versa. Eyelid myokymia appears as involuntary undulating movements of one eyelid at a time. The eye does not close and the twitches last only seconds. Tics usually occur in males in the second decade of life. These are not typically isolated to unilateral facial contraction. There may be

173

Lee AG, ed. Curbside Consultation in Neuro-Ophthalmology: 49 Clinical Questions, Second Edition (pp 173-175). © 2015 SLACK Incorporated.

174  Question 41 Figure 41-1. Injection sites for botulinum toxin A in patients with HFS. The amount given (2.5 to 5 units per injection site) and location of injection are provided. Typical (X) and optional (O) sites depend on the muscles involved and desired outcome. The black line over the nasolabial fold indicates the location of hyaluronic acid injection to soften facial asymmetry.

intermittent eye deviation, shrugging, or throat clearing. These tics do not persist during sleep and often resolve spontaneously by age 20 years. Blepharospasm involves both eyes and may appear as excessive blinking or forceful eyelid closure. Finally, neuroleptic agents may also produce unusual facial movements that are generally bilateral and asynchronous. Look in the patient’s mouth, since the tongue is often involved, distinguishing this condition from typical HFS. Your patient’s age and clinical history are consistent with HFS. The overwhelming consensus is that HFS is due to irritation of the seventh nerve by an aberrant vessel; however, there are several well-documented cases of HFS from tumors. We believe that magnetic resonance imaging (MRI) and magnetic resonance angiography (MRA) with attention to the cerebellopontine angle and facial nerve should be performed. These scans will make it possible to rule out tumors of the posterior fossa. If such tumors are present, imaging may demonstrate vascular compression of the pons or facial nerve. We generally treat debilitating HFS with subcutaneous injections of botulinum toxin type A (Figure 41-1). We usually start with 2.5 units per injection and titrate as needed. A preinjection topical anesthetic or an ice pack may reduce pain. We try to avoid the area around and below the nasolabial fold because of the risk that weakness of the orbicularis oris may cause drooling. Patients should understand that double vision and ptosis may occur in 2% to 8% of cases. For this reason, they should avoid rubbing the injection sites or lying down for the first several hours after they leave the office to avoid unwanted spread of the botulinum toxin. Bruising is common and often inevitable. We advise patients to avoid aspirin and nonsteroidal anti-inflammatory drugs for 1  week prior to injection when possible. For the first few weeks after injection, we encourage aggressive lubrication with artificial tears and ointment at bedtime for exposure keratopathy secondary to lagophthalmos. Improvement in spasms occurs in approximately 3 to 7 days and

What Is Hemifacial Spasm?  175 typically persists for 3 to 6 months, after which the injections must be repeated. We do not favor oral medications such as benzodiazepines (eg, clonazepam), anticonvulsants (eg, carbamazepine), or muscle relaxants (eg, baclofen). We have found that these are not very effective and result in undesirable side effects. In cases with asymmetry of the lower face even after neurotoxin injection, hyaluronic acid fillers can be used. These soften the nasolabial fold and the weight of the material can even dampen the involuntary movements. We also inform patients that microvascular decompression is another treatment option. With a suboccipital craniectomy, a Teflon sponge can be placed between the compressive vessel and the seventh nerve. This has a high rate of success (up to 90%) and is curative in many cases, but it also carries a risk of facial palsy, ipsilateral deafness, vertigo, and stroke. Recurrences of HFS after surgery have been reported in up to 20% of patients. In our experience, most patients prefer botulinum toxin injections.

Summary ●

Hemifacial spasm is a clinical diagnosis.



Magnetic resonance imaging is useful to exclude a compressive lesion.



Injections of botulinum toxin are useful in symptomatic patients.



Microvascular decompression is another therapeutic option but has significant risks.

Bibliography Borodic, GE. Use of fillers as adjunct therapy for the treatment of lower face hemifacial spasm. Ophthal Plast Reconstr Surg. 2013;29(2):225-226. Pawlowski M, Gess B, Evers S. Babinski-2 sign in hemifacial spasm. Mov Disord. 2013;28:1298-1300. Samii M, Gunther T, Iaconetta G, et al. Microvascular decompression to treat hemifacial spasm: long term results for a consecutive series of 143 patients. Neurosurgery. 2002;50:712-718. Tan NC, Chan LL, Tan EK. Hemifacial spasm and involuntary facial movements. Q J Med. 2002;95:493-500.

42 QUESTION

HOW DO YOU DEAL WITH NONORGANIC VISUAL LOSS? Robert R b t L. L Lesser, L MD A 13-year-old girl reports that her vision is “ just not clear.” She noted this problem at the beginning of the school year and has trouble “reading for a long time.” The examination shows that her visual acuity is 20/50 in the right eye and 20/20 in the left, with an inferior visual field defect in the right eye (Figure 42-1). The remainder of her examination is normal with no afferent pupillary defect; the optic nerves and maculae look healthy. Another eye care provider has prescribed reading glasses, but these were of little help. What are the next steps in evaluating this patient? When I am dealing with unexplained visual loss, I recommend considering the subtle but recognizable causes. The major considerations are listed in Table 42-1. If no abnormality is found on the clinical examination, the possibility of a nonorganic cause becomes more likely. You should develop a specific “game plan” in approaching a patient with a potentially nonorganic basis for diminished vision. I recommend that you develop a sympathetic but direct approach in proving that the patient’s visual function is normal. Numerous testing techniques have been proposed in dealing with this clinical problem. It is essential that you select a series of these techniques and become facile in performing them. Showmanship might even be required, but above all reassurance and encouragement of the patient are keys to success. My favorite tests to detect nonorganic visual loss from the simplest to most complex include the following.

Stereo Acuity Testing Sixty seconds of arc is equivalent to 20/40, while 40 seconds indicates 20/20 acuity.

Near-Distance Disparity Look for a disparity in near and distance vision. If a patient can read 20/20 at near and 20/50 at distance, the visual loss is not physiological.

177

Lee AG, ed. Curbside Consultation in Neuro-Ophthalmology: 49 Clinical Questions, Second Edition (pp 177-181). © 2015 SLACK Incorporated.

178  Question 42 Figure 42-1. Automated visual field demonstrates inferior visual field loss in the right eye.

Prism Dissociation Test Two alternate techniques are useful. The first involves placing a 4-diopter prism base-down in front of the “bad eye” and a 0.5 prism diopter base-down in front of the “good eye.” The patient is then asked if he or she sees 2 lines. In this case, the patient will be shown the 20/20 lines and asked to read each of them. He or she is then asked to read the top line forward and the bottom line backward. If your patient can do that, you have demonstrated 20/20 vision from the “bad eye” (Figure 42-2A). This is one of my favorite tests because it can be done quickly. Patients are usually fooled by this test because they are complaining of visual loss, not diplopia. An alternate method is to place a 4-diopter prism base-down in front of the “better eye” with both eyes open. The patient is again asked what he or she sees on the Snellen chart. If 2 lines of

How Do You Deal With Nonorganic Visual Loss?  179

Table 42-1

Subtle or Easily Missed Causes of Unexplained Visual Loss Cornea 1. Irregular astigmatism 2. Keratoconus 3. Anterior basement membrane dystrophy

Lens 1. Subtle cataracts̶oil droplet, minimal nuclear sclerosis

Retina 1. 2. 3. 4. 5.

Occult macular dystrophy Cone dystrophy Paraneoplastic retinopathy Early Stargardt disease Occult epiretinal membrane, macular hole, or edema

Optic Nerve 1. Toxic or nutritional optic neuropathy 2. Autosomal dominant optic atrophy

letters are seen and both lines are read correctly, then the patient has 20/20 vision in both eyes. Should the patient see only one line, he or she may have an organic basis for diminished vision in one eye (Figure 42-2B). Regardless of the technique, this test must be carried out in an efficient and encouraging manner.

Microscopic Killer Refraction (Toothpaste Refraction—Squeeze It Out of Them) Start at the 20/10 level for refraction and work up slowly, using various “aids,” such as red-green colors on the chart, rotating prism, Jackson cross cylinder, and lenses with minimal power (eg, 60.25 or 60.50). Then work up and go to the 20/15 line and repeat the same process. Use various aids to suggest that they will help, like a pinhole, Risley prism, and “telescopic” lens. You often need to take your time so you can wear the patient down—be creative and persuasive. By the time you get to the 20/20 level, the patient is instructed to read the “large letters.” Show the patient multiple 20/20 lines and ask him or her to read the top line only. Using these techniques, with a little encouragement and positive reinforcement, the patient will often be able to read the 20/20 line.

180  Question 42

Figure 42-2. Prism dissociation test. (A) With good vision in each eye, the patient will be able to read each 20/20 line. (B) With poor vision in one eye, only one clear image of the 20/20 line will be seen. Often, a second blurred image will be seen, depending on the level of vision in the eye with an organic cause for reduced acuity.

Asymmetrical Fogging Blur both eyes with the phoropter and then gradually dial lenses back to the correct refraction in front of the “bad eye” while keeping the other eye blurred. When patients read 20/20 from the “bad eye,” you have made the diagnosis. The problem with this test is that patients will often close one eye, having figured out how to beat the test. You can also use Polaroid lenses or red-green filters to dissociate the 2 eyes.

Different Distances Measure acuity at different distances. If the patient can see 20/10 at 10 feet, that is equivalent to seeing 20/20 at 20 feet.

Saccade Envy Visual Field Test The saccade envy test is a good way to prove that your patient’s field loss is not real. First, perform confrontation visual field testing (Figure 42-3A). Next, ask the patient to look at your finger in the periphery and then back at your nose. Say that these are some of the fastest saccades that you have ever seen! Praise these fast eye movements and bring some office personnel or other physicians in to observe and admire these unusually fast saccades. In demonstrating these “amazing” saccades, hold your finger in different “nonseeing areas” (Figure 42-3B). If the patient can see your fingers in these areas when testing saccades, you have documented that the visual field loss has no organic basis.

Electrophysiological Testing If the above examination techniques are inconclusive, then electrophysiological studies, such as electroretinography (ERG) and visual evoked potentials (VEPs) are helpful. It is essential to monitor visual fixation during these tests, since you can get a false-positive result if the patient

How Do You Deal With Nonorganic Visual Loss?  181 Figure 42-3. (A) Confrontation visual field testing with the patient reporting an inability to see fingers in the inferior visual field. (B) Patient looking in the direction of the inferior visual field when told that saccades are being tested, confirming that the patient indeed can see in the inferior visual field.

does not look at the stimulus correctly. If full-field ERG and pattern-reversal VEPs are normal, it is likely that the patient has nonorganic visual loss. Sometimes, multifocal ERG testing may also be required to exclude a localized macular disorder. Always remember that nonorganic visual loss is a diagnosis of exclusion. The patient may have an organic cause for diminished visual acuity that may be subtle and challenging to demonstrate. Furthermore, the patient may have a superimposed “functional overlay,” magnifying and embellishing the true deficit. You must tease apart the organic from the nonorganic! You must be certain not to overlook the cause of the patient’s visual complaint; if there is any doubt, 1 or 2 follow-up visits may be advisable to confirm the diagnosis. Most patients can be treated with reassurance, often during the initial examination, as well as 1 to 2 follow-up visits. A confrontational approach is usually counterproductive. It is essential to provide the patient with a “way out.” Although psychiatric referral is usually not needed, physicians should be aware of the possibility of physical or sexual abuse associated with this condition in children. After seeing a child, you should inform the pediatrician of the diagnosis.

Summary ●

Most patients with nonorganic visual loss can be treated with reassurance, often during the initial examination as well as at follow-up visits.



A confrontational approach is usually counterproductive.



Provide the patient with a “way out.”



Be aware of the possibility of physical or sexual abuse associated with this condition in children.

Bibliography Bain KE, Beatty S, Lloyd C. Nonorganic visual loss in children. Eye. 2000;14:770-772. Bengtzen R, Woodward M, Lynn MJ, Newman NJ, Biousse V. The “sunglasses sign” predicts nonorganic visual loss in neuro-ophthalmologic practice. Neurology. 2008;70:218-221. Golnik KC, Lee AG, Eggenberger ER. The monocular vertical prism dissociation test. Am J Ophthalmol. 2004;137:135-137. Lim SA, Siatkowski RM, Farris BK. Functional visual loss in adults and children: patient characteristics, management, and outcomes. Ophthalmology. 2005;112:1821-1828. Trobe JD. The Neurology of Vision. New York: Oxford University Press; 2001:369-388.

43 QUESTION

HOW DO YOU DIAGNOSE AND MANAGE MIGRAINE AURA? Robert R b tH H. S Spector, t MD A 46-year-old ophthalmologist was operating when he noticed a vague, ill-defined cloud in his vision; by covering each eye, he realized that it was a central right homonymous defect. Over the next 10 minutes, the cloud enlarged, moved to his right, and seemed to move behind a flickering, colorful edge, which together with the cloud blocked vision. He excused himself from surgery, closed his eyes for 10 minutes, and the visual symptoms resolved. Headache never followed. His history included headaches typically provoked by sleeplessness and hunger. Aura is defined as a sensory warning heralding the onset of a more explicit neurological event, like a migraine or a major motor seizure. Visual auras in migraine may manifest either as a transient negative scotoma (eg, transient homonymous hemianopia or monocular visual loss or a transient positive scotoma) that usually comprises a luminous, colorful, geometric display of sparkling, dazzling, dancing, or flickering lights arrayed in homonymous portions of the visual field. Although most migrainous auras announce the onset of a headache, 13% of migraineurs have an aura without headache, a syndrome that used to be called acephalgic migraine but now is classified as migraine aura without headache (MAWH). Few, if any, other conditions produce a migraine experience. I will describe the hallmark features of a migrainous aura that may aid in the differential diagnosis—the “red flags” that warrant a more extensive search for other causes of a similar experience. This will be followed by a short discussion regarding treatment of this migraine variant. A typical migrainous aura is not static but rather moves across the visual field. It begins as a small, flickering, ill-defined spot eccentric to fixation that, over 20 to 60 minutes, enlarges and marches across the homonymous portions of visual field, leaving behind a wake of blindness that slowly fills in as the scintillation fades into the temporal periphery of the ipsilateral eye. The enlargement and march of the visual hallucination in homonymous portions of the visual field are indigenous to migraine and do not occur, for instance, in somewhat similar conditions, such as an occipital seizure or transient ischemic attack (TIA). Retinal migraine is defined by the International Headache Society as attacks of fully reversible monocular visual disturbance associated with migraine headache and a normal neuro-ophthalmological examination between attacks.

183

Lee AG, ed. Curbside Consultation in Neuro-Ophthalmology: 49 Clinical Questions, Second Edition (pp 183-186). © 2015 SLACK Incorporated.

184  Question 43

Table 43-1

Nuances of the Migraine Aura ●







Aura that is preceded or followed by another type of migraine accompaniment, such as paresthesias (pins and needles) that gradually march up one extremity and cross to the other side Attacks of MAWH interspersed with other types of migraine syndromes (eg, migraine headache without aura) Auras that historically occur in close temporal relationship to well known migraine-precipitating circumstances, such as consuming red wine, chocolate, pressed meats, MSG or aspartame, or the auras recur just before or after onset of a menstrual period If the patient says that he or she has had 2 or more identical episodes that have recurred months or years apart, with full recovery each time.

It is mandatory that other causes of these monocular phenomena be excluded before establishing migraine as the diagnosis, an adage that should be even more strictly enforced in individuals with atherosclerotic risk factors. If patients claim that their experience was monocular, ask them to cover each eye independently at the onset of the next “attack” because they will often see a nasal defect or display in the less involved eye, thus establishing the symptoms as homonymous and not monocular. In addition to the pathognomonic “march” and “enlargement” of the migrainous aura, additional nuances of the history help to differentiate the MAWH syndrome (Table 43-1). Virtually all patients with transient visual phenomena see a physician after their vision has returned to normal. Accurate diagnosis, therefore, is predicated on meticulous history taking and a physical examination. Be sure to include auscultation of the orbits, cranial vault, and neck vessels to listen for a bruit; also include direct and indirect ophthalmoscopy of each eye to look for intravascular plugs or plaques, isolated or multiple retinal infarctions, or signs of venous stasis retinopathy. Finally, quantitative visual field testing should be done to detect an asymptomatic defect that topographically localizes to the occipital lobes. Either an atypical history or physical findings will engender an evaluation of several conditions, including focal epilepsy, recurrent microemboli, structural lesions of the occipital lobe (arteriovenous malformation or brain tumor), hematological disorders, and collagen vascular disease. A lower threshold for investigating the differential diagnosis can be argued in the elderly and very young because they often cannot recount the small but characteristic details of their visual experience. That said, if the description of a transient visual event is sufficiently characteristic of migraine (buildup, march and duration), I have yet to find an alternative etiology. This, over the years, has led me to the conclusion that if the examiner takes the time to exact specific details of the event and medical history, few non-migrainous conditions will remain that truly mimic a characteristic MAWH. Occipital epileptogenic discharges often produce tonic deviation of the eyes, vomiting, and a unilateral or generalized convulsion. Thromboembolism may indeed cause a transient homonymous hemianopia and should be considered in individuals with atherosclerotic risk factors (smoking, obesity, diabetes mellitus, hypertension, dyslipidemia, family predilection for premature vascular disease) or cardiac conditions that may cause thromboembolism (eg, valvular damage,

How Do You Diagnose and Manage Migraine Aura?  185 recent myocardial infarction, patent foramen ovale, and myocardiopathy). To the clinical adage that vertebrobasilar artery disease causes transient homonymous hemianopia and blindness should be added the fact that it occurs as the isolated manifestation of a TIA in less than 2% of cases. Although buildup, march, and duration help to distinguish a migraine aura, 25% of migraineurs state that their visual loss or display becomes maximal immediately. The absence of buildup should not exclude migraine but should, at least, bring into question the differential diagnosis. Remember that a thromboembolic TIA will cause simultaneous sensory loss on the face, arm, and leg, while in migraine the sensory symptoms slowly march and spread over the face, fingers, or hand and may even migrate and cross to the contralateral face and hand over 30 minutes. In general, the duration of a migrainous aura lasts 15 to 25 minutes, whereas 95% of TIAs last less than 15 minutes. The indications for MRI or MRA of the brain with and without contrast in individuals with transient visual symptoms with or without headache include atypical history or physical findings, rapidly increasing frequency and/or severity of auras alone or headaches, first or “worst” headache ever experienced, thunderclap or abrupt-onset incapacitating headache, new onset of headache or MAWH after age 50 years, and headache(s) that are refractory to traditional treatment. MRI and MRA may be normal, which does not definitely exclude a non-migrainous condition, such as thrombocythemia, thrombotic thrombocytopenia, polycythemia, and other hyperviscosity states or clotting disorders. MAWH can occur in individuals of any age but more commonly occurs in females older than age 45 years. Although not listed as such in the International Classification of Headache Disorders, pediatric acephalgic migraines are listed along with other childhood periodic syndromes as “migraine equivalents,” which may be good predictors of the future development of typical migraines. Persistent migraine aura without infarction (PAWOI) is a little-known condition first described under the designation prolonged migraine aura status. PAWOI is said to be a possible cause of a variety of neurological symptoms, including visual snow, loss of vision, and increased afterimages. It is unclear which medical examinations are useful in diagnosing PAWOI; at present, it is diagnosed solely based on the patient’s present and past symptoms. Auras usually need no treatment. Reportedly agents including acute magnesium, rapid-acting nonsteroidal anti-inflammatory drugs such as meclofenamate or naproxen, sublingual nitroglycerin or nifedipine, or a beta-agonist inhalant have been used. However, sublingual nifedipine is no longer used because of the risk of profound hypotension. Calcium channel blockers and the antiepileptic drugs have had some success in patients with frequent MAWH (valproic acid, gabapentin, and topiramate). However, know that the prophylactic migraine medications are designed to prevent headache, not visual aura. Triptans should never be used to treat an aura. Oral triptans do not act fast enough to affect an aura, and if the rapid-acting injectable sumatriptan is given during the aura, it may not abort a subsequent headache. Because they may cause vascular constriction, triptans need to be used with great caution in older patients, who may have vascular disease, hypertension, or other cardiovascular risk factors. I find that explaining the etiology of their phenomena to patients (and their families) avoids the need to use any other therapy. Anecdotally, asking a patient to simply rebreathe his or her own air may abbreviate or abort the visual symptoms, allegedly because increased blood carbon dioxide levels cause cerebral vasodilation and theoretically should reverse the occipital dysfunction, presumably caused by migrainous cerebral vasoconstriction. However, I have never found this maneuver to be successful. Once confident that you are dealing with the diagnosis of MAWH based on accumulating information from a careful history physical examination, the treatment of choice, in my opinion, is reassurance, reassurance, and more reassurance. Medications are available to be taken at the first sign of an impending migraine attack. They attempt to prevent the headache from developing or to reduce the severity of the attack. Many of

186  Question 43 these medications cause the constriction of blood vessels and cannot be given to patients at risk of heart attack or other vasculopathies; they include ergot derivatives such as DHE-45, serotonin agonists (triptans), inhaled isoproterenol, and sublingual nitroglycerin. The medications come in various preparations to enable administration via different routes. For example, patients who experience vomiting and cannot keep pills down may benefit from a nasal spray or injection.

Summary ●





Clinical history that differentiates a migrainous visual aura from other causes of an attack of transient visual loss or transient visual hallucinations Worrisome history or physical findings that should alert you to investigate alternative etiologies Answer the questions when and if a CT or MRI scan of the head are indicated in a person with a transient or positive scotoma

Bibliography Evans RW, Tietjen GE. Migrainous aura versus transient ischemic attack in an elderly migraineur. Headache. 2001; 41:201-203. Fernandez-Torre JL. Epileptic auras: classification, pathophysiology, practical usefulness, differential diagnosis and controversials. Rev Neurol. 2002;34(10):977-983. Savitz SI, Caplan LR. Vertebrobasilar disease. N Engl J Med. 2005;352:2618-2626. Silberstein SD, LIpson RB, Dodick DW, eds. Pain. In: Wolff’s Headache and Other Head Pain. 8th edition. New York: Oxford University Press: 2007: 63-95 Spector RH. Migraine. Focal Points. 2000;28:1-12. Swartz RH, Kern RZ. Migraine is associated with magnetic resonance imaging white matter abnormalities. Arch Neurol. 2004;61:1366-1368. Tuxhorn IEB. Somatosensory auras in focal epilepsy: a clinical, video EEG and MRI study. European J Epilepsy - Seizure. 2005;14(4):262-268.

44 QUESTION

HOW DO I RECOGNIZE LEBER HEREDITARY OPTIC NEUROPATHY? Nancy J. Newman, MD The patient is a 22-year-old man in college who went out drinking over the weekend with his buddies. On Monday, he noted a loss of vision in the right eye to counting fingers. He was found to have a right relative afferent pupillary defect and the nerve looked swollen on the right. A visual field examination was performed and showed a dense central scotoma on the right; the left was normal. Optic neuritis was initially suspected, and he was scheduled for magnetic resonance imaging (MRI) later in the week. Now, however, he has called to say that he has vision loss in his left eye as well. Interestingly, the same thing happened to his brother at age 25 years, a year earlier, and his vision never really recovered. Any ideas? This young man has suffered severe central visual loss in the right eye. The right relative afferent pupillary defect, the dense central scotoma, and the abnormal-appearing right optic nerve all led you to the appropriate localization of the problem to the right optic nerve. It was most certainly reasonable, in a person of this age with an acute unilateral optic neuropathy, to consider the possibility of inflammation of the optic nerve (ie, optic neuritis). However, if there was no pain at the time of visual loss, optic neuritis would be much less likely (over 90% of patients with optic neuritis will experience pain, usually on eye movement). Similarly, since optic neuritis tends to affect females far more often than males, I am always a bit wary of making a definitive diagnosis of typical idiopathic demyelinating optic neuritis in a male, especially when there is no complaint of pain. The real spoiler in this case is the loss of vision in the other eye within the week. Although optic neuritis can present in both eyes (and indeed the Optic Neuritis Treatment Trial showed that concurrent involvement in the fellow eye was not at all unusual, occurring in more than 40% of patients), involvement of the second eye broadens the differential diagnosis to include infectious and other inflammatory etiologies of optic neuritis (such as syphilis, sarcoidosis, and neuromyelitis optica), as well as other disease categories including rapidly infiltrating neoplasms (such as the leukemias, lymphomas, and malignant optic nerve gliomas), compressive lesions with rapid expansion, such as pituitary apoplexy, or even vascular etiologies. However, the clinical presentation is most suggestive of Leber hereditary optic neuropathy (LHON), and the history of a brother who suffered similar visual loss at age 25 years essentially clinches the diagnosis.

187

Lee AG, ed. Curbside Consultation in Neuro-Ophthalmology: 49 Clinical Questions, Second Edition (pp 187-189). © 2015 SLACK Incorporated.

188  Question 44 LHON is a maternally inherited disorder, usually affecting individuals between the ages of 15  and 35 years, with a male predominance of 4 to 1. Onset is typically painless and acute or subacute, with simultaneous onset in both eyes in about 50% of cases or sequential onset with an interval of days, weeks, or months. Essentially, all patients have bilateral involvement by 1 year, typically by 6 months. Deterioration of vision in each eye has usually reached its nadir by 3 to 4  months, with most patients having visual acuities worse than 20/200. Visual field defects, as in this patient, are usually central or cecocentral scotomas. The classic funduscopic appearance in some patients with LHON seen acutely is that of pseudoedema of the optic nerve head, in which the disc looks elevated, hyperemic, and swollen, but there is no late leakage on fluorescein angiography to indicate true disc edema. Small telangiectatic vessels may be seen on the disc, and other retinal vessels may appear tortuous. These funduscopic changes may be seen not only in affected patients but also in presymptomatic individuals and even in asymptomatic maternal relatives who never lose vision. However, many patients with LHON never manifest these funduscopic features, even when examined at the time of acute visual loss, occasionally resulting in an erroneous diagnosis of nonorganic visual loss in these patients. Ultimately, after several weeks, pallor of the optic nerve head and loss of the nerve fiber layer supervenes, especially involving the papillomacular bundle. MRI is usually normal. Typically, visual loss from LHON is permanent, although spontaneous recovery of even excellent central visual acuity can occur, especially in those patients who suffer their initial visual loss at a young age, usually younger than 20 years, and who harbor one of the less common mitochondrial DNA mutations (see below). Visual recovery does not usually occur for at least 6 months to a year after visual loss and often years later. Optic neuropathy is typically the only manifestation of the disease, although there are individuals and pedigrees with cardiac conduction defects and minor neurological abnormalities. LHON is inherited maternally; all offspring of a woman carrying the trait will inherit the trait, but only the females can pass it on to the subsequent generation. Maternal inheritance results when the genetic defect is in DNA that resides in the cytoplasm of the cell rather than the nucleus; hence it is passed on via the mother’s egg. This DNA resides in the mitochondria of the cell and is called mitochondrial DNA (mtDNA). Three point mutations in the mtDNA are considered to be causal in about 90% of all cases of LHON and are designated “primary LHON mutations”: the mutation at position 11778 within the mtDNA accounts for about 70% of LHON cases, the mutation at 3460 accounts for about 13% of LHON cases, and the mutation at 14484 accounts for about 14% of LHON cases. Clinically, patients with visual loss from the 3 primary LHON mutations differ only in their propensity for visual recovery—patients with the 14484 mutation have a far greater chance of spontaneous recovery of vision (up to 70% of cases) than those patients with the 11778 or 3460 mutations (as low as 4% of cases). The understanding of the genetic basis of LHON and the availability of molecular diagnosis on individual patients has allowed us to make a definitive diagnosis of this disorder even in patients without a family history of visual loss. However, we still do not know the exact location of the initial pathology within the optic nerve, the pathophysiology of optic nerve dysfunction, the explanation of the disease’s peculiar timing and focal pathology, or the determinants of expression. Indeed, the majority of patients who harbor a LHON mtDNA mutation will never express visual loss, with approximately 20% to 50% of at-risk males and 4% to 30% of at-risk females losing vision during their lifetimes. Furthermore, the male predominance in this disorder cannot be explained by maternal inheritance and has recently been linked to a probable disease-modifying factor on the X chromosome. Environmental factors, both internal and external, may play a role in expression. Systemic illnesses, nutritional deficiencies, medications, or toxins that stress or directly inhibit mitochondrial metabolism could conceivably initiate or increase phenotypic expression of LHON. For example, tobacco use may be more prevalent among those individuals who have lost vision from LHON; therefore, smoking cessation among at-risk carriers should be advised.

How Do I Recognize Leber Hereditary Optic Neuropathy?  189 The relatively easy access to laboratory testing for the 3 primary LHON mutations on blood samples allows us to consider the diagnosis of LHON not only in typical cases such as the one presented here, but also in any unusual or unexplained case of bilateral optic neuropathy. I would most certainly screen this young man for the primary LHON mutations. However, if his brother has already been tested and proved positive for a LHON mutation, there is no reason to confirm this on the patient or his maternal relatives because all maternally related family members will have the mutation. I would certainly proceed with the planned MRI; although other causes of his optic neuropathy are far less likely, they have very different management implications. I would also obtain an electrocardiogram to screen for cardiac conduction defects. Attempts to treat or prevent the acute phase of LHON visual loss with systemic or topical medications or vitamin supplementation have mostly proven ineffective. There have been anecdotal reports of improvement in vision with idebenone (a synthetic analog of coenzyme Q10). A recent randomized controlled trial of idebenone in the treatment of patients with LHON within 5 years of visual loss failed to show a meaningful difference with treatment overall but suggested that idebenone might be helpful in those patients treated early in their course of visual loss, when visual function is still asymmetrical. Other similar therapies are in the developmental phase and gene therapies via intravitreal injection are on the horizon. I recommend idebenone (if available) for patients with LHON within their first year of visual loss. I also strongly recommend cessation of smoking, as I would anyway for general health reasons, and advise against excessive alcohol use. An assessment by a low-vision specialist is often helpful, especially given that a fair amount of peripheral vision may be retained. The importance of genetic counseling of patients with LHON and their families cannot be overstated. The prognosis for visual recovery should be communicated based on the specific mutation harbored. Most importantly, it should be explained to this patient and his brother that neither of them can pass this disease on to their children. However, all their female maternal relatives, whether affected with visual loss or not, will pass on the mutation, and hence the risk of visual loss, to all of their offspring.

Summary ●







LHON is a clinical diagnosis confirmed by mitochondrial DNA genetic testing. In patients with LHON, treatment with idebenone may be considered within a year of visual loss onset. Recommend cessation of smoking and excessive alcohol use. The prognosis for visual recovery should be communicated based on the specific mitochondrial DNA mutation.

Bibliography Newman NJ. Hereditary optic neuropathies. In: Miller NR, Newman NJ, Biousse V, Kerrison JB, eds. Walsh & Hoyt’s Clinical Neuro-Ophthalmology. 6th ed. Philadelphia: Lippincott Williams & Wilkins; 2005:465-501. Newman NJ. Treatment of hereditary optic neuropathies. Nat Rev Neurol. 2012;8:545-556. Yu-Wai-Man P, Griffiths RG, Chinnery PF. Mitochondrial optic neuropathies—disease mechanisms and therapeutic strategies. Prog Retin Eye Res. 2011;30:81-114.

45 QUESTION

HOW DO I MANAGE AN ORBITAL APEX SYNDROME? Roger R oger E E. T Turbin, urb bin M MD, D F FACS ACS A 35-year-old man with diabetes mellitus and a history of diabetic ketoacidosis (DKA) presents with new onset painful ophthalmoplegia and loss of vision in his left eye. There might be a bit of proptosis as well. What should I be worried about and how soon does he need to be seen? This clinical scenario represents a neuro-ophthalmic emergency and should be managed as an immediate, life-threatening, invasive fungal infection until proven otherwise. In my experience, the situation is commoner on the wards, in the setting of DKA or other immunosuppressive comorbid conditions (eg, leukemia, chemotherapy, long-term corticosteroids, broad-spectrum antibiotic therapy, chronic dialysis, solid organ or bone marrow transplant recipients) than in patients who “walk into the office.” I hospitalize or utilize the emergency department to coordinate otolaryngologic, neurosurgical, neurologic, medical, and infectious disease service consultation, as well as to obtain immediate neuroimaging to include contrast-enhanced studies of the head, sinuses, and orbit if renal failure does not preclude MRI or CT contrast. I typically recommend the institution of broad-spectrum antifungal therapy (amphotericin or posaconazole), often prior to biopsy confirmation. The differences between orbital apex syndrome (OAS) (involving cranial nerves II, III, IV, VI, and V1), cavernous sinus syndrome, and superior orbital fissure syndrome (absence of optic nerve involvement) do not significantly affect diagnostic considerations but may guide the diagnostic biopsy and the surgical decision concerning the extent of debridement after the diagnosis is established. Fungal infection may originate from or extend beyond ophthalmic and orbital structures to involve the cavernous sinus, paranasal sinuses, or cerebrovascular structures at presentation despite intact bone. The differential diagnosis of OAS also includes other infectious (bacterial and especially viral infections such as herpes zoster; also mucocele), inflammatory, thrombotic (aseptic, septic, fistula, or vascular malformation), as well as vasculitic or neoplastic (benign, malignant) lesions. However, these entities remain a diagnosis of exclusion in a sick patient with diabetes mellitus after ruling out invasive fungal infection. I perform a complete neuro-ophthalmic examination, including assessment of the pupil for anisocoria or relative afferent pupillary defect, assessment of proptosis and orbital congestion, motility analysis (cranial nerves III, IV, and VI versus myopathic restrictive dysmotility),

191

Lee AG, ed. Curbside Consultation in Neuro-Ophthalmology: 49 Clinical Questions, Second Edition (pp 191-196). © 2015 SLACK Incorporated.

192  Question 45 Figure 45-1. Endoscopic photograph of necrotic middle turbinate and ethmoid mucosa in a patient with biopsy-proven Rhizopus (Mucor), although initial intraoperative analysis was suggestive of Aspergillus nigricans.

fundoscopic examination (optic nerve and retinal perfusion, choroidal folds, cherry-red spot), and other neurological symptoms (including sympathetic dysfunction). In addition to facial and lower cranial nerve function, I always evaluate trigeminal nerve function (corneal anesthesia, facial numbness), as areas of numbness (cranial nerves V1, V2, and V3) typically represent more widespread disease. In addition, I inspect the nasal and oropharyngeal mucosa, including the soft and hard palate, to detect areas of early necrosis (Figure 45-1). Emergent ear-nose-throat consultation, endoscopic evaluation, and photography is warranted for management, serial examination, and extended follow-up. Occasionally the affected patient may present with very mild proptosis, visual loss, or an ocular motility disorder (Figure 45-2). The soft tissue details of MRI and bone details of CT are both useful and frequently complementary, as is vascular imaging (CTA, MRA, MRV, or cerebral angiography) in selected patients. I pay special attention to the location of concurrent paranasal disease and the destruction of the fat plane posterior to the maxillary sinus at the level of the pterygopalatine fossa and pterygomaxillary fissure. The latter is a frequently overlooked but critical sign that sinus opacification represents more than just incidental sinusitis and may indicate invasive fungal infection (Figure 45-3). Infection may spread from the adjacent sinuses despite intact bone to cause increased signal and enhancement in the orbital apex, extraocular muscles, and orbital tissues. Bone destruction may be present and rarely the sinuses may even initially appear normal. In addition, hypointense “black” signal on both T1- and T2-weighted MRI images may serve as a radiographic hint that fungus is present because of the signal change (paramagnetic susceptibility artifact) from the manganese, magnesium, iron, or calcium concretions in some species of fungus (Figure 45-4). This dark signal on MRI may be mistaken for the hypointensity of air if careful attention is not paid to the sequences and concomitant comparison with orbital CT is not made. The taxonomy of invasive fungal infection may be confusing. The term Mucormycosis describes any fungal infection of the order Mucorales, which belongs to the class Zygomycetes and may be termed zygomycosis. The terms Rhizopus, Rhizomucor, Mucor, and Absidia refer to the genus. Rhizopus oryzae is the predominant pathogenic species, accounting for 60% of all forms of mucormycosis and 90% of Rhinocerebral forms. Aspergillosis refers to infection by Deuteromycetes, an imperfect fungus without a sexual reproductive phase. The three main disease-causing species are Aspergillus fumigatus, A flavus, and A niger. The early differentiation between the two most frequent invasive fungal pathogens, Aspergillus and Zygomycetes, has become increasingly important given the trend toward first-line use of therapies not based on amphotericin B in aspergillosis (Table 45-1). However, the initial differentiation of aspergillosis versus zygomycosis is not always

How Do I Manage an Orbital Apex Syndrome?  193 Figure 45-2. This patient developed DKA and was initially diagnosed as having viral meningitis due to mild CSF pleocytosis. Treated with broadspectrum antibiotics and insulin, he subsequently progressed from a very mild proptosis and ocular motility disorder to severe abnormalities. At the time of the photograph, early in his course, he had preserved visual acuity, minimal left proptosis, and mild exotropia due to involvement of his left medial rectus. Sinus and orbital biopsy confirmed Rhizopus arrhizus, sensitive to amphotericin B, micafungin, and posaconazole. His ocular motility worsened diffusely, his left vision decreased to 20/400, he developed a left relative afferent pupillary defect, and he experienced multiple septic emboli causing severe expressive aphasia. He underwent three sinus endoscopic débridements, transcaruncular medial orbital debridement, systemic amphotericin B, and oral posaconazole therapy, as well as direct irrigation of amphotericin B to the sinuses and orbit via retrobulbar injection. Three months after discharge, his vision had recovered to 20/25, his ocular motility abnormality was limited to only a modest left adduction defect, and his expressive aphasia was improving, allowing him to speak in simple sentences. He remains able to live independently.

Figure 45-3. Axial CT scan after administration of contrast shows the loss of the normal fat plane behind the left maxillary sinus in the region of the pterygopalatine fossa (large arrow) despite apparently intact bone in a patient with biopsy proven Rhizopus. Compare this area with the normal black signal of the intact fat plane on the right (small arrow). There is also abnormal thickening and enhancement in the soft tissue of the face anterior to the maxillary sinus, as well as the left turbinate, maxillary sinus, and ethmoids (not shown).

clinically straightforward; I have been involved in a number of cases in which even the intraoperative differentiation of aspergillosis versus zygomycosis was reversed at the time of culture identification. The misidentification may potentially lead to either inappropriate therapy with or the delay of the use of species-directed voriconazole. In additional, a noninvasive allergic form of chronic fungal sinusitis in atopic individuals exists (predominantly due to Aspergillus spp.), which is treated with debridement and corticosteroid therapy. The distinction between allergic and invasive sino-orbital disease is critical since corticosteroids are contraindicated in invasive fungal disease. The chronic allergic form (ie, allergic fungal sinusitis) may sometimes present a confusing acute presentation if a mucocele ruptures and causes a secondary orbital cellulitis. Historically, patients with invasive fungal infection of the orbital apex suffer severe morbidity and frequent mortality. Survival requires early diagnosis and immediate treatment. Experienced authors have discussed treating infected tissues as a “malignancy” with extirpation of all involved tissues. This results in frequent exenteration or even more extensive deforming craniofacial resection in patients with other comorbid conditions. I have moved away from this paradigm, utilizing

194  Question 45

Figure 45-4. (A) The patient illustrated in Figure 45-2, with biops-proven rhino-sino-orbital Rhizopus arrhizus, has opacification of the posterior ethmoid sinus, as shown on this coronal T2-weighted MRI image. Paramagnetic susceptibility artifact from fungal hyphae may produce hypointense or “blacker” signal on T1- and T2-weighted MRI studies than expected. The fungal signal is hypointense (white arrow) within the more typical bright signal of diseased sinus tissue. Depending on the mineral content, the signal may even simulate air. (B) T1-weighted axial contrast-enhanced MRI image again shows dark areas within the diseased sinus (white arrow). Abnormal signal (white arrowhead) is suggestive of infiltration at the left orbital apex.

surgery as an early diagnostic modality with a more limited resection of tissue that is clearly ischemic or necrotic. It is my usual current practice to use, in addition to broad-spectrum or species-directed systemic antifungal therapy (Table 45-1), adjuvant direct antifungal application. It has also been my experience that despite the standard textbook discussion citing that pathogens are easily differentiated based on microscopic morphology, the thinner, septate, acute branchingangled Aspergillus hyphae may swell with frozen preparation and frequently be misinterpreted as Mucor (see Figure 45-1 discussion). I consider surgical and systemic antifungal treatment supportive therapy until the primary immunosuppressive state can be reversed. If the underlying immunosuppressive state is not reversed with aggressive therapy, early insulin, and so on, it is very difficult to halt disease progression. In addition, the pathological local environment at the site of infection may preclude adequate local antibiotic delivery to necrotic tissue through affected blood vessels. In addition, in Mucor infections, local changes produce acidotic conditions that promote fungal proliferation even after systemic DKA is corrected. I irrigate the affected soft tissues and sinuses intraoperatively with 1 L of 0.25 to 0.50 mg/mL amphotericin B prepared by the pharmacy for surgical irrigation. I also typically perform postoperative retrobulbar or peribulbar injection of 2 to 6 mL of amphotericin B at 2 mg/mL prepared for injection in a sterile hood, favoring lower volumes in nonblind eyes to prevent injection related orbital compartment syndrome. I will direct the retrobulbar needle into affected areas, administer a test dose with blood pressure monitoring, and consider a peribulbar lidocaine injection if the patient is intolerant. Some authors have advocated an indwelling orbital catheter for irrigation, which theoretically might decrease the chance of an intradural injection through the nerve sheath or an ocular perforation, but it adds the addition risk of an indwelling orbital foreign body. I do

How Do I Manage an Orbital Apex Syndrome?  195

Table 45-1

Treatment Guidelines Infection

Drug

Dosage/Duration Alternatives

Aspergillosis

Voriconazole

6 mg/kg IV q12h x 1 day, then 4 mg/kg IV bid or 200 to 300 mg PO bid until resolved

Zygomycosis

Amphotericin Ba Lipid formulation amphotericin Bb

a

Lipid formulation of amphotericin Ba Posaconazole 200 mg PO tid-qid caspofungin 70 mg IV x 1 day, then 50 mg IV once/day Micafungin 100 to 150 mg IV daily Posaconazole 150 to 200 mg PO qid x 6 to 10 weeks

Amphotericin B deoxycholate 0.7 to 1.0 mg/kg IV

b

Lipid formulation of amphotericin B

Amphotericin B lipid complex 5 mg/kg IV (Abelcet) Liposomal amphotericin B 3 to 5 mg/kg IV (AmBisome) Amphotericin B cholesteryl sulfate complex 3 to 4 mg/kg IV (Amphotec) Adapted from Treatment guidelines: antifungal drugs. Med Lett Drugs Ther. 2012;10(120):61-68.

leave a postoperative catheter in place to irrigate the sinuses; before irrigation, this may require an application of topical anesthetic spray and the availability of an emesis basin. Please note, however, that these recommendations for the topical and local administration amphotericin B involve off-label uses.

Summary ●









An orbital apex syndrome in patients who are diabetic or immunocompromised may represent a life-threatening fungal infection and remains the primary diagnosis of exclusion. High clinical suspicion and early diagnosis of fungal infection is required to prevent morbidity and mortality. Diagnosis and treatment require the coordination of a multispecialty approach. Advances in systemic antifungal therapy provide potential alternatives to nephrotoxic monotherapy with amphotericin B. In fungal rhino-sino-orbital infection, biopsy, limited debridement, and systemic antifungal therapy coupled with adjuvant local antifungal therapy may provide an alternative to mutilating extirpative surgical procedures.

196  Question 45

Bibliography Hebrecht R, Denning DW, et al. Voriconazole versus amphotericin B for primary therapy of invasive aspergillosis. N Engl J Med. 2002;347:408-415. Treatment guidelines: antifungal drugs. Med Lett Drugs Ther. 2012;10(120):61-68. Turbin RE, Khoobiar SA, Langer P, et al. Adjuvant therapy for invasive sino-orbital fungal infection. J Neuroophthalmol. 2002;22(2):178-179.

46 QUESTION

HOW DO I EVALUATE AND MANAGE IDIOPATHIC ORBITAL INFLAMMATORY SYNDROME? Timothy T imotthy JJames ames M McCulley, cC Cullley M MD D An 18-year-old otherwise healthy young woman presents with a 1-week history of progressive painful periocular swelling. She denies a history of trauma or systemic illness. Her visual acuity is intact, but when her eyelid is held open she reports double vision when looking up. She is afebrile with no other abnormality identified on examination. The photos in Figure 46-1 are of two patients who presented as described above with roughly 1 week of painful periocular swelling. In approaching such patients with inflamed orbits, I often find the most troubling aspect to be diagnosis. This is because of the combination of the lack of confirmatory tests and the potential consequences should the wrong diagnosis be made. Table 46-1 lists some of the more frequent causes of orbital inflammation. When no cause is identified, as a diagnosis of exclusion, we label patients with the term orbital pseudotumor. However, with modern imaging, even difficult orbital inflammatory disease no longer masquerades as neoplastic disease, at least not for long. Therefore, I prefer and use the more descriptive term idiopathic orbital inflammatory syndrome (IOIS). Presumably, all orbital inflammation has a potentially identifiable cause, and as our diagnostic abilities improve, the term IOIS will predictably be used less. For example, in recent years a subgroup of IgG4-related inflammation has been identified. Just a few years ago, such individuals would have been diagnosed with IOIS. Nevertheless, we still are often unable to determine a specific diagnosis for patients with orbital inflammation, and the term IOIS remains apropos. In evaluating patients with an inflamed orbit, a primary goal is to identify those with infectious and/or systemic disease. As with all patients, the importance of a thorough history must be stressed. A relatively short duration of hours to days suggests either orbital inflammatory syndrome or an infection. A subacute presentation of days to weeks is more characteristic of Graves’ disease or a neoplastic process. Also, note any history of rheumatological disease, which might be an associated with soft tissue inflammation (eg, sarcoidosis, granulomatosis with polyangiitis (Wegener granulomatosis), polyarteritis nodosa, rheumatoid arthritis, and xanthogranulomatous disease). A history of thyroid disorder should be noted. Pain and tenderness are important indicators. Nontender swelling is more concerning for neoplasm, although some infections and

197

Lee AG, ed. Curbside Consultation in Neuro-Ophthalmology: 49 Clinical Questions, Second Edition (pp 197-200). © 2015 SLACK Incorporated.

198  Question 46 Figure 46-1. External photographs of two patients: (A) patient A and (B) patient B. Both patients presented with less than 1 week of progressive periocular swelling, erythema, and pain. Both were afebrile and otherwise healthy.

Table 46-1

Differential Diagnosis of an Inflamed Orbit Category

Subcategory

No identifiable cause Inflammatory

Specific Detail Idiopathic orbital inflammatory disease

Vasculitis

Wegener granulomatosis Systemic lupus erythematosus Hepatitis-related vasculitis Polyarteritis nodosa Rheumatoid arthritis

Miscellaneous

Sarcoidosis Graves disease

Infectious

Bacterial

Staphylococcus aureus Streptococcus pyogenes

Fungal

Mucor (mucormycosis) Aspergillus (aspergillosis)

Parasitic

Taenia solium (cysticercosis) Trichinella (trichinosis) Microfilaria

Echinococcus Neoplastic

Lymphoproliferative disease Ruptured dermoid Rhabdomyosarcoma

Vascular and lymphatic

Carotid cavernous fistula Lymphangioma

How Do I Evaluate and Manage Idiopathic Orbital Inflammatory Syndrome?  199 Figure 46-2. Imaging studies of (A) patient A and (B) patient B. In patient A, computed tomography demonstrates sinusitis with an adjacent subperiosteal abscess. In patient B, magnetic resonance imaging demonstrates clear sinuses and no fluid collection. Inflammation is seen to extend from the lacrimal gland to the superior orbit.

inflammatory syndromes can also cause little or no pain. Concern for infection increases in some high-risk patients: those who are immunosuppressed, with inadequately controlled diabetes mellitus, with a history of periocular trauma or perhaps an insect bite, and with a history of sinus disease. Unfortunately, helpful history is often lacking, as in the two patients illustrated in Figures 46-1 and 46-2. With just about any patient presenting with orbital inflammation, imaging is obtained on the same day. Figure 46-2 shows the imaging of our two patients. For most diseases, I prefer magnetic resonance imaging (MRI), but computed tomography (CT) is often adequate, at least for an initial screening. A primary goal of this initial step is to identify infection. The presence of sinus disease strongly suggests an infectious etiology, and an abscess is, of course, indicative of infection. In patient A (see Figures 46-1A and 46-2A), opaque sinuses with an adjacent abscess confirm the diagnosis of infection. This patient fared well after being admitted to the hospital for open drainage of the abscess followed by intravenous antibiotic therapy. The two findings most suggestive of noninfectious inflammation are the lack of sinus disease and focal inflammation. By focal inflammation, I mean inflammation centered on a specific structure, such as a muscle (myositis), the optic nerve (optic perineuritis), or Tenon capsule. In patient B (see Figures 46-1B and 46-2B), the sinuses were clear and inflammation involved and extended from the lacrimal gland (see Figure 46-2A). Bear in mind that there is no finding (or lack of a finding) on imaging that can with absolute certainty establish the presence or absence of infection. In patients with a first episode of IOIS—unless otherwise indicated by findings on history, physical, or imaging—no further diagnostic evaluation may be needed. When the initial evaluation is indicative of IOIS, management is tailored to severity. If there is visual loss (eg, an optic neuropathy) or vision appears to be threatened, I will admit patients to the hospital for intravenous steroid therapy (usually 250 mg of methylprednisolone 4 times/day) for 3 to 5 days, followed by oral therapy (roughly 1 mg/kg of prednisone). If vision is intact and the patient is reliable, I will treat him or her as an outpatient. The duration and rapidity of the steroid taper is determined by disease severity and response. In most cases of idiopathic IOIS, complete resolution can be expected within 1 or at most 2 weeks. Patient B responded very well to oral steroids, with a complete resolution in less than 2 weeks. Once at baseline, I will initiate a slow taper over the course of 1 to 2 months. It is important not to decrease the steroid dose too quickly and

200  Question 46 to bump it back up should symptoms return during the taper. When steroid therapy is ineffective, alternate “steroid-sparing” immunosuppressive medications (eg, methotrexate, rituximab) may be helpful. Radiation therapy has also been advocated, but I have not found it to be effective and view it as an absolute last resort. In some patients, a more extensive diagnostic evaluation may be beneficial. These include the following: patients with an atypical initial presentation, history suggestive of infection or systemic syndromes, failure to respond to steroids, and recurrent disease. A biopsy may yield a diagnosis or at least narrow down the categories of disease, guiding a more focused evaluation. The decision of whether to biopsy largely depends on how accessible the involved tissue is. A chest CT can help in assessing a number of conditions (eg, sarcoidosis, tuberculosis, lymphoproliferative disease). Although tuberculosis rarely presents with isolated orbital inflammation, a purified protein derivative is easy to obtain. Serological evaluation might include a complete blood count, angiotensin converting enzyme, lysozyme, cytoplasmic and perinuclear anti-neutrophil cytoplasmic antibodies, antinuclear antibody, rheumatoid factor, thyroid function, and a hepatitis panel. Of course, if history or examination is in any way suggestive of a specific disease, a targeted evaluation might be appropriate. It is also useful to remember that with recurrent or persistent disease, a repeat of some or all of the above may yield different results the second time around.

Summary ●



IOIS is a diagnosis of exclusion. When evaluating a patient with an inflamed orbit, a primary goal is to identify those with infectious and/or systemic disease.



A thorough history often helps in identifying underlying diseases.



Pain and an acute or subacute presentation are important indicators of IOIS.







Imaging is important to obtain in all patients with an inflamed orbit. I prefer MRI, but CT is often adequate. In patients with recurrent or atypical disease, a more extensive diagnostic evaluation may be beneficial. This may include serologic testing and/or biopsy. Management is tailored to severity. If there is visual loss (an optic neuropathy) or vision appears to be threatened, use IV steroid therapy (usually 250 mg methylprednisolone QID). If vision is intact and the patient is reliable, treat them as an outpatient (roughly 1 mg/kg prednisone). The duration and rapidity of the steroid taper is determined by disease severity and response.

Bibliography Kapur R, Sepahdari AR, Mafee MF, et al. MR imaging of orbital inflammatory syndrome, orbital cellulitis, and orbital lymphoid lesions: the role of diffusion-weighted imaging. Am J Neuroradiol. 2009;30:64-70. Rootman J. Diseases of the Orbit: A Multidisciplinary Approach. Philadelphia,PA: Lippincott Williams & Wilkins; 2003:455499. Yoon MK, McCulley TJ. Orbital disease in neuro-ophthalmology. Neurol Clin. 2010;28:679-699.

47 QUESTION

HOW DO I MANAGE THE LOW-FLOW CAROTID CAVERNOUS FISTULA? LLeah eah h LLevi, evii M MBBS BBS The patient is a 65-year-old elderly hypertensive woman with a chronic red eye. Topical antihistamines, antibiotics, antivirals, and prednisone have been tried but the eye just stays red. The vessels seem engorged, dilated, and tortuous. Now she is complaining of double vision and a “ bulging” eye. What should be done? In your patient, the engorged vessels, complaints of eye bulging, and diplopia tell you that this is not simple conjunctivitis. You should consider low-flow dural cavernous sinus fistula, which is a communication between one or more dural branches of the carotid artery and the cavernous sinus. As in your patient, this is generally painless with mild findings that at first can easily be mistaken for conjunctivitis or other common causes of red eye (Figure 47-1). Unlike high-flow fistulas, the patient often does not have a pulsatile or audible bruit. Since the venous drainage of the eye and the orbit is impaired and under arterial pressure, you should systematically look for signs that reflect this increased venous pressure in 3 areas: (1) the eye itself, (2) the orbit, and (3) the cavernous sinus.

Eye Tortuous dilated conjunctival and episcleral vessels (see Figures 47-1 and 47-2) Lid swelling and conjunctival chemosis (see Figure 47-2B) Asymmetrical intraocular pressure (higher on the affected side) Increased pulse of applanation mires on the affected side Retinal venous congestion (Figure 47-3) The most common signs of a low-flow dural cavernous sinus fistula are found during ophthalmological examination. In the eye, the main effect of the increased venous pressure is the tortuosity of the conjunctival and episcleral vessels. There may even be some mild lid swelling and conjunctival chemosis. The increased episcleral venous pressure, in turn, leads to increased intraocular pressure as compared with the unaffected eye, even into the abnormal range. A very useful sign I look ● ● ● ● ●

201

Lee AG, ed. Curbside Consultation in Neuro-Ophthalmology: 49 Clinical Questions, Second Edition (pp 201-204). © 2015 SLACK Incorporated.

202  Question 47 Figure 47-1. Mild red eye on the left in a patient with a low-flow dural fistula. In this case there was also a left Horner syndrome.

Figure 47-2. (A) Dilated and tortuous conjunctival and episcleral veins. This patient’s fistula closed spontaneously after a plane flight. (B) A different patient demonstrating conjunctival chemosis. This patient had bilateral red eye and chemosis and was treated as having “chronic conjunctivitis” until he developed a sixth nerve paresis.

A

B

Figure 47-3. Signs of retinal venous congestion: dilated tortuous veins and hemorrhages. In this case, there is also some mild disc edema.

for is increased pulsation of the Goldmann applanation mires on the affected side. Since the episcleral veins are under arterial pressure, there is a larger change in intraocular pressure from systole to diastole (increased pulse pressure). This sign helps me to differentiate a fistula from other conditions that may have congestion as part of the clinical picture, such as thyroid-associated orbitopathy or orbital inflammation. I also look for increased congestion and tortuosity of the retinal veins.

Orbit Proptosis (frequently absent or mild) Congestion of the extraocular muscles (uncommon) In the orbit, the increased venous pressure may cause proptosis on the affected side from orbital congestion. The extraocular muscles may be engorged in some cases, and this can lead to diplopia. ● ●

How Do I Manage the Low-Flow Carotid Cavernous Fistula?  203 Figure 47-4. Orbital CT scan shows enlarged superior ophthalmic vein in the left orbit compared with the right orbit (see arrows).

Figure 47-5. Dilated superior ophthalmic vein (arrow) in a right dural cavernous fistula. Note the normal diameter of the contralateral superior ophthalmic vein (arrowhead).

Cavernous Sinus Ocular motor cranial nerve palsies (sixth the most common) Horner syndrome Your patient’s complaint of diplopia is from the effects of the increased venous pressure in the cavernous sinus, where the sixth nerve is commonly affected. In low-flow fistulas, the third and fourth nerves may be involved, but this is less common. Other possible signs to look for are an ipsilateral Horner syndrome (see Figure 47-1). If the clinical findings suggest a low-flow dural cavernous sinus fistula, I usually obtain orbital imaging with contrast to confirm dilation of the superior ophthalmic vein (Figures 47-4 and 47-5). If you decide to obtain a magnetic resonance imaging (MRI) scan, make sure you ask for fat-saturated views of the orbits so that you can see the details of the superior ophthalmic vein. In ordering the imaging, I like to specify on the request that I suspect a fistula and to look for a dilated superior ophthalmic vein. MR angiography and computed tomography (CT) angiography can also be ordered to diagnose and better define the fistula. The choice of angiography modality is somewhat institution dependent, so it is worth speaking to the neuroradiologist regarding this. Catheter angiography is still the definitive test for demonstrating the exact anatomy of the fistula. However, since up to 50% of low-flow dural fistulas close spontaneously and without complications within several months and since angiography carries a small but definite risk, I usually ● ●

204  Question 47 do not obtain angiography when I first see the patient. This can be done by the interventional neuroradiologist if the patient is referred for treatment. Considering the high rate of spontaneous closure of low-flow dural cavernous fistulas, you can usually manage the patient conservatively as long as the symptoms and findings are mild and not progressive or a danger to vision. If your patient has no concerning signs such as retinal venous congestion and has an intraocular pressure that can be controlled medically and is causing no visual field defects, then you can simply follow the patient, obtaining baseline visual field testing and fundus photography. Ocular coherence tomography looking for defects in the retinal nerve fiber layer is an additional option for baseline and follow-up evaluation. I like to see the patient every few weeks to check the intraocular pressure, repeat the visual field testing, and make sure there are no signs of increasing retinal venous congestion. If your patient has proptosis and corneal exposure, I recommend lubricating drops and ointment. If your patient has diplopia, you can temporize with patching, a Fresnel prism, or orthoptics depending on the clinical situation. If the patient develops progressive and concerning signs—such as increasing venous congestion or intraocular pressure that is uncontrolled medically or is causing visual field or defects in the retinal nerve fiber layer—or if the fistula does not close within a few months and the patient is symptomatic owing to orbital congestion or diplopia, I will refer to an interventional neuroradiologist for definitive treatment. Treatment consists of closing the communication between the arterial and venous systems. Depending on the exact anatomy, this can be done either by introducing embolic materials into the arterial branches feeding the fistula or via the veins draining the fistula. In appropriate cases, embolic materials such as coils may be introduced via the superior ophthalmic vein by the interventional neuroradiologist coordinating with an orbital surgeon. Although the first-line approach to treatment is endovascular, some complex cases have been successfully treated with stereotactic radiosurgery. After closure of the fistula, whether spontaneously or after treatment, I like to document that the clinical findings have improved or resolved (as is usually the case) and manage any mild residual problems. Occasionally, the closure may not be complete or there may be a recurrence, so I like to follow these patients periodically, being prepared to manage mild findings or refer again to interventional neuroradiology for additional treatment.

Summary ●





Patients with proptosis, ophthalmoplegia, and signs of orbital venous congestion should be evaluated for carotid cavernous sinus fistula. Consider referral of symptomatic patients to an interventional neuroradiologist for definitive evaluation and possible treatment. Closure of the communication between the arterial and venous systems can be done using endovascular techniques.

Bibliography Choi BS, Park JW, Kim JL, et al. Treatment strategy based on multimodal management outcome of cavernous sinus dural arteriovenous fistula (CSDAVF). Neurointervention. 2011;6(1):6-12. Liu HM, Wang YH, Chen YF, Cheng JS, Yip PK, Tu YK. Long-term clinical outcome of spontaneous carotid cavernous sinus fistulae supplied by dural branches of the internal carotid artery. Neuroradiology. 2001;43:1007-1014. Miller NR. Dural carotid-cavernous fistulas: epidemiology, clinical presentation, and management. Neurosurg Clin North Am. 2012;23(1):179-192.

48 QUESTION

HOW DO I EVALUATE AND TREAT RADIATION OPTIC NEUROPATHY? Anne A nne A Abel, bell MD MD and dM Michael ich haell S S. LLee, ee M MD D A 73-year-old man with a history of metastatic bladder cancer presents with 1 week of severe painless vision loss in the right eye. Ten months prior, he underwent whole brain radiation followed closely by Gamma Knife radiation to metastatic brain lesions. He denies headache and systemic signs of giant cell arteritis. How should I evaluate and treat this patient? Radiation optic neuropathy (RON) results from delayed radionecrosis of the anterior visual pathways following radiation therapy to intracranial, skull base, and paranasal sinus tumors. It presents as painless vision loss in one or both eyes months to years after radiation therapy (mean 18 months). The retrobulbar optic nerve and chiasm are more commonly affected, in which case the optic disc initially appears normal. Progressive optic nerve pallor develops over 6 to 8 weeks. RON is a diagnosis of exclusion, so make sure you consider tumor recurrence, secondary malignancy, and other infectious and inflammatory acute optic neuropathies. Keep in mind that with RON or other forms of optic neuritis the affected optic nerves or chiasm will enhance with gadolinium on MRI (Figure 48-1). Unfortunately, the prognosis for radiation optic neuropathy is poor and there is no proven treatment. The majority of patients (85%) end up with 20/200 vision or worse in the affected eye.

Evaluation In patients with a history of radiation therapy to the head and neck who present with an acute optic neuropathy, I first want to know how much radiation they received. The risk of RON increases with higher doses. Total fractionated external beam radiation greater than 50 gray (Gy) and individual fractions greater than 2.0 Gy are considered to pose a higher risk. Therefore, if the patient had 18 Gy over 10 days, RON would seem unlikely. Stereotactic radiosurgery involves higher doses in one to three fractions. In these cases, single doses of radiation between 8 and 10 Gy are considered relatively safe. A lower threshold for radiation injury occurs among patients with diabetes mellitus, patients undergoing concomitant chemotherapy, and patients with a history of

205

Lee AG, ed. Curbside Consultation in Neuro-Ophthalmology: 49 Clinical Questions, Second Edition (pp 205-207). © 2015 SLACK Incorporated.

206  Question 48 Figure 48-1. Radiation optic neuropathy. This 63-yearold man presented with vision loss in the left eye 18 months after radiation treatment with 6660 cGy for nasopharyngeal leiomyosarcoma. Visual acuity was 20/20 in the right eye and hand motion in the left eye. Fundus exam showed pallor of both optic nerves. Superior and inferior arcuate defects were present on the visual field exam in the right eye. Axial T1 MRI with fat suppression after gadolinium shows bilateral prechiasmal optic nerve enhancement (arrows). He received methylprednisolone 1 g IV for 3 days. Twelve months later, visual acuity and visual fields remained stable while optic nerve enhancement had nearly resolved.

compressive tumors (ie, pituitary adenomas), so we may consider RON at lower doses in these patients. Ask any neuro-ophthalmologist and you will be told that a comprehensive neuro-ophthalmic examination is in order, including a careful evaluation of the pupil, color vision, and visual field testing. A relative afferent pupillary defect will occur only if radiation injury is unilateral or asymmetrical. In any older patient with an optic neuropathy, giant cell arteritis (GCA) should always cross your mind. You may want to inquire about systemic symptoms of GCA and check the erythrocyte sedimentation rate and C-reactive protein. If the optic disc is swollen, this may not be RON and you should think more about anterior ischemic optic neuropathy (AION). Generally, a swollen optic disc occurs in eyes with a history of ocular radiation, such as brachytherapy, rather than skull base radiation. I would order a brain and orbit MRI with gadolinium to rule out recurrent tumor or secondary malignancy and to look for optic nerve/chiasmal enhancement. If the vision loss is severe, I would arrange for same-day imaging; otherwise, it may be reasonable to get the MRI within several days. Many of these patients have recent surveillance imaging as part of their oncology or neurosurgical follow-up. You could ask the neuroradiologist if the optic nerves enhance on these scans. Keep in mind that enhancement has been reported on surveillance imaging prior to the onset of visual symptoms in RON.

Treatment Unfortunately, there is no proven treatment for RON. Therapies have attempted to affect downstream tissue ischemia, such as free radical damage to vessel walls and the optic nerve support cells. Proposed but unproven therapies have included intravenous corticosteroids, systemic anticoagulation, hyperbaric oxygen (HBO) therapy, and intravitreal and intravenous vascular endothelial growth factor inhibitors (anti-VEGF). Recently, an isolated case of intravenous bevacizumab was reported to dramatically improve both visual field and visual acuity in a patient with radiation optic neuropathy. This has not yet been studied in a randomized trial. Each of these therapies has associated risk, and none have been studied in a randomized, masked, controlled trial. Moreover, the natural history of RON is unknown, and, albeit rarely, spontaneous recovery has been reported.

How Do I Evaluate and Treat Radiation Optic Neuropathy?  207 HBO is the most controversial treatment for RON and it deserves special mention. HBO for RON has not been studied prospectively; only anecdotal, retrospective reports exist. Although HBO is an FDA-approved treatment for delayed radiation injury in soft tissue and bone, no study has shown the same benefit for any type of neurological injury. Cochrane Reviews found no benefit of HBO in acute stroke, traumatic brain injury, multiple sclerosis, or peripheral neuropathy. Among optic neuropathies, one retrospective case-controlled study reported no efficacy for HBO in AION. Although common side effects are low risk (lenticular myopia, barotrauma), seizures and pulmonary toxicity can occur after HBO. Moreover, the described HBO protocols for RON are time-consuming and expensive. The HBO protocol involves 30 dives of 90 minutes each, costing upward of $16,000. Thus patients could conceivably incur large medical bills for an unproven therapy. Although I have not personally observed improvement in RON with corticosteroids, I treat with 3 days of intravenous corticosteroids (methylprednisolone 1 gram IV daily) in case the enhancement on MRI represents an inflammatory optic neuropathy. Once I have made the diagnosis of RON, I typically see patients back in 4 to 6 weeks, extending follow-up intervals over time. Eventually, the optic nerve or chiasmal enhancement on MRI will resolve; however, I do not typically order follow-up imaging.

Summary ●

RON is a diagnosis of exclusion.



Occurs months to years (average 18 months) after high dose radiation.



Need to rule out malignancy, infectious and inflammatory optic neuropathy including GCA.



MRI will show optic nerve enhancement.



No proven treatment, but should consider IV steroids.



Prognosis is guarded.

Acknowledgment This work was supported by an unrestricted grant from Research to Prevent Blindness (New York, NY) and the Lions Club of Minnesota.

Bibliography Farooq O, Lincoff NS, Saikali N, et al. Novel treatment for radiation optic neuropathy with intravenous bevacizumab. J Neuroophthalmol. 2012;32:321-324. Lee MS, Borruat FX. Should patients with radiation-induced optic neuropathy receive any treatment? J NeuroOphthalmol. 2011;31:83-88. Lessell S. Friendly fire: neurogenic visual loss from radiation therapy. J Neuro-Ophthalmol. 2004;24:243-250. Levy RL Miller NR. Hyperbaric oxygen therapy for radiation-induced optic neuropathy. Ann Acad Med Singapore. 2006;35:151-157.

49 QUESTION

WHAT SHOULD I DO WITH A SEVENTH NERVE PALSY? Matthew M attthew JJ. T Thurtell, hurttelll M MBBS, BBS F FRACP RACP A 32-year-old man presents with an acute onset of left-sided facial weakness. How should I evaluate and treat this patient? The seventh, or facial, nerve is predominantly an efferent motor nerve, innervating the facial muscles and stapedius muscle in the middle ear. However, it also has sensory and autonomic functions; it conveys taste sensation from the anterior two thirds of the tongue (via the chorda tympani), afferent sensation from part of the external auditory canal and soft palate, and efferent parasympathetic innervation to the lacrimal and salivary glands. The nerve arises from the pons. Its fascicle passes near the sixth nerve nucleus and fascicle, medial longitudinal fasciculus, and corticospinal tracts. After exiting the pons, the nerve passes through the cerebellopontine angle to the internal auditory meatus of the temporal bone, where it is adjacent to the eighth nerve. When I see a patient with a seventh nerve palsy, I start by taking a history to determine the timing of the onset of symptoms. I ask if the patient has noted ipsilateral loss of taste sensation, increased sensitivity to sounds or noise (ie, hyperacusis), or any other neurologic or auditory symptoms; their presence or absence will guide my examination and localization of the lesion. When I examine the patient, I note their facial appearance; unilateral seventh nerve palsy produces a flattened nasolabial fold, decreased forehead wrinkling, and a widened palpebral fissure. I evaluate the strength of the facial muscles by asking patients to raise their eyebrows, smile, puff their cheeks out, and close their eyes tightly. I note if the upper and lower facial muscles are equally affected; sparing of the upper facial muscles might indicate an upper motor neuron lesion, whereas involvement of all facial muscles indicates a lower motor neuron lesion. I pay close attention to the patient’s ability to close the eye. If there is significant orbicularis oculi weakness, there will be lagophthalmos (inability to close the eye), poor apposition of the lower eyelid to the eye, and upward eye deviation (Bell’s phenomenon) with attempted eye closure. The risk of exposure keratopathy is much greater if there is lagophthalmos and an inadequate Bell’s phenomenon. I assess taste sensation by placing salt or sugar crystals on the ipsilateral side of the tongue. If the patient has facial paralysis without any other deficits, I palpate for a mass in the parotid gland. The next step in evaluation is to complete the ophthalmic and neurologic examinations. During the ophthalmic examination, I measure visual acuity, look for signs of exposure keratopathy, and

209

Lee AG, ed. Curbside Consultation in Neuro-Ophthalmology: 49 Clinical Questions, Second Edition (pp 209-211). © 2015 SLACK Incorporated.

210  Question 49 Figure 49-1. Left seventh nerve palsy (A) associated with left horizontal gaze palsy and left internuclear ophthalmoplegia (one-and-ahalf syndrome) (B), caused by a small plaque of demyelination involving the left facial colliculus (C).

A

B

C

examine the fundus for optic disc edema, as a seventh nerve palsy can occur with raised intracranial pressure. During the neurologic examination, I check the other cranial nerves; other cranial nerve palsies can occur with a cerebellopontine angle lesion or meningeal process. I assess ocular motility, as there might be a horizontal gaze palsy (sixth nerve nuclear lesion), one-and-a-half syndrome, or abduction deficit (sixth nerve palsy) with a pontine lesion (see Figure 49-1 for an example). I also evaluate eighth nerve function, checking for ipsilateral hearing loss, nystagmus (with quick phases to the contralateral side), and loss of the ipsilateral vestibulo-ocular reflex. I look for external auditory canal and palatal vesicles, which can occur in Ramsay Hunt syndrome. During my neurologic examination, I evaluate power, coordination, and the reflexes since abnormalities in these may indicate a central lesion, neuropathy, myopathy, or neuromuscular junction problem. I next consider what further investigations are needed. If the patient has other neurologic signs, MRI of the brain with contrast is mandatory. I have a low threshold for consulting a neurologist in such patients since a lumbar puncture, other special investigations (eg, nerve conduction studies), or urgent intervention may be required. If the patient has an acute isolated unilateral seventh nerve palsy, the likely diagnosis is Bell’s palsy. Bell’s palsy has an acute onset, with maximum paralysis occurring within 3 to 4 days. Some patients report preceding pain and many report facial fullness or numbness. MRI with contrast shows enhancement of the facial nerve or geniculate ganglion. Other investigations are usually not required, although testing for possible inflammatory and infectious causes (eg, sarcoidosis or Lyme disease) might be indicated depending on the clinical scenario. The majority of patients will spontaneously recover their seventh nerve function. However, because of the chance of incomplete recovery, I routinely recommend treatment with oral steroids (prednisone 60 mg daily for 5 days followed by a 5-day taper), which has been shown to increase the probability of recovery in multiple clinical trials. As Bell’s palsy is thought to be caused by reactivation of herpes simplex virus in the geniculate ganglion, I mention that antivirals (eg, acyclovir) could be given, but that they only modestly increase the probability of recovery. If the patient has lagophthalmos, I recommend aggressive ocular lubrication and, if necessary, taping the eye closed at night, with close ophthalmic follow-up. If the patient develops significant exposure keratopathy, I recommend tarsorrhaphy to protect the cornea until seventh nerve function recovers. If the patient has a poor recovery of facial power, I refer them to a surgeon with expertise in seventh nerve grafting techniques.

What Should I Do With a Seventh Nerve Palsy?  211

Summary ●









The seventh nerve is predominantly an efferent motor nerve, innervating the facial muscles and stapedius muscle, but it also has sensory and autonomic functions. Seventh nerve palsy results in weakness of the ipsilateral facial muscles, although sparing of the upper facial muscles may indicate an upper rather than lower motor neuron lesion. Patients with seventh nerve palsy are at risk of exposure keratopathy, especially if there is lagophthalmos and an inadequate Bell’s phenomenon. The most common cause of an acute isolated seventh nerve palsy is Bell’s palsy. Clinical trials have found that the outcome of Bell’s palsy improves with steroids (oral prednisone) and perhaps with antiviral treatment.

Bibliography Bhatti MT, Schiffman JS, Pass AF, Tang RA. Neuro-ophthalmologic complications and manifestations of upper and lower motor neuron facial paresis. Curr Neurol Neurosci Rep. 2010;10:448-458. Gilden DH. Bell’s palsy. N Engl J Med. 2004;351:1323-1331. Gronseth GS, Paduga R. Evidence-based guideline update: steroids and antivirals for Bell palsy. Neurology. 2012;79:22092213. Sullivan FM, Swan IR, Donnan PT, et al. Early treatment with prednisolone or acyclovir in Bell’s palsy. N Engl J Med. 2007;357:1598-1607.

FINANCIAL DISCLOSURES

Dr. Anne Abel has no financial or proprietary interest in the materials presented herein. Dr. Benin Barahimi has no financial or proprietary interest in the materials presented herein. Dr. M. Tariq Bhatti has no financial or proprietary interest in the materials presented herein. Dr. Valérie Biousse has no financial or proprietary interest in the materials presented herein. Dr. Mark Borchert has no financial or proprietary interest in the materials presented herein. Dr. Swaraj Bose has no financial or proprietary interest in the materials presented herein. Dr. Emily M. Bratton has no financial or proprietary interest in the materials presented herein. Dr. Paul W. Brazis has no financial or proprietary interest in the materials presented herein. Dr. Jodie M. Burton has participated in advisory boards and received honoraria and educational support from Novartis, TEVA Beuroscience Canada, Biogen Idec, EMD Serono, and Genzyme. Dr. Tom Carlow has no financial or proprietary interest in the materials presented herein. Dr. Sophia Chung received research support from Eli Lilly & Co. Dr. Kimberly Cockerham receives a DOD grant and is a paid consultant for Lumenis. Dr. James Corbett has no financial or proprietary interest in the materials presented herein. Dr. Wayne Cornblath has no financial or proprietary interest in the materials presented herein.

213

214  Financial Disclosures Dr. Fiona Costello has no financial or proprietary interest in the materials presented herein. Dr. Roberto A. Cruz has no financial or proprietary interest in the materials presented herein. Dr. Kathleen B. Digre was supported in part by an unrestricted grant from Research to Prevent Blindness, Inc, New York, NY, USA, to the Department of Ophthalmology and Visual Sciences, University of Utah, Salt Lake City, Utah, USA. Dr. Marc Dinkin has no financial or proprietary interest in the materials presented herein. Dr. James A. Dixon has no financial or proprietary interest in the materials presented herein. Dr. Eric Eggenberger is a consultant for Acorda, Berlex Inc, Biogen Inc, Genzyme, Novartis Pharmaceuticals Corporation, Serono, and Teva Pharmaceutical Industries Ltd. He receives lecture fees from Berlex, Biogen, and Teva Pharmaceutical. He also receives grant support from Biogen, Novartis Pharmaceuticals, Serono, and Teva Pharmaceutical. Dr. Angelina Espino has no financial or proprietary interest in the materials presented herein. Dr. Julie Falardeau has no financial or proprietary interest in the materials presented herein. Dr. Steven E. Feldon has no financial disclosures relevant to the content of the chapter. Dr. Feldon is a consultant to the American University Professors of Ophthalmology (EVP) and he has grants from Research to Prevent Blindness and National Eye Institute. He has a patent currently licensed to Lumetrics (no product, no sales) and has < 5% ownership of a start-up company AcuRet (no product, no sales). Dr. Rod Foroozan has no financial or proprietary interest in the materials presented herein. Dr. Deborah I. Friedman is a speaker for Allergan. She received grant support from Merck, National Eye Institute, and GammaCore. She is a consultant for Avanir and is on the advisory board for Avanir, Supernus, Teva Pharmaceuticals. Dr. Friedman is on the editorial board for Neurology Reviews and is a contributing author for MedLink Neurology. She has also served as an expert witness for both plaintiff and defense counsel in cases of idiopathic intracranial hypertension. Dr. James A. Garrity has no financial or proprietary interest in the materials presented herein. Dr. Christopher C. Glisson has received speaker and consultant honorarium from Biogen-Idec, Novartis, Lundbeck, and Questcor. None of the topics contained within this manuscript are relevant to these relationships. Dr. Karl Golnik has no financial or proprietary interest in the materials presented herein. Dr. Aaron Grant has no financial or proprietary interest in the materials presented herein. Dr. Jennifer K. Hall has no financial or proprietary interest in the materials presented herein. Dr. Steven R. Hamilton is a paid consultant for Novartis and Teva Pharmaceuticals.

Financial Disclosures  215 Dr. Andrew R. Harrison is co-owner of Neuro-ophthalmix, LLC. Dr. Jonathan C. Horton has no financial or proprietary interest in the materials presented herein. Dr. Randy Kardon receives funding (grants) from NEI R009040554; R01 EY018853; Department of Defense; TATRC funding (grants); VA Rehabilitation Research and Development; and Novartis steering committee for OCTiMS multicenter study. Dr. David I. Kaufman is an adjudicator for Lilly on an AION registry research study. Dr. Aki Kawasaki received book royalties from Cambridge University Press in the past year. Dr. Sachin Kedar has no financial or proprietary interest in the materials presented herein. Dr. Lanning B. Kline has no financial or proprietary interest in the materials presented herein. Dr. Melissa W. Ko has no financial or proprietary interest in the materials presented herein. Dr. Gregory S. Kosmorsky has no financial or proprietary interest in the materials presented herein. Dr. Byron L. Lam has no financial or proprietary interest in the materials presented herein. Dr. Jacqueline Leavitt has no financial or proprietary interest in the materials presented herein. Dr. Andrew G. Lee has no financial or proprietary interest in the materials presented herein. Dr. Michael S. Lee is co-owner of Neuro-ophthalmix, LLC. Dr. Robert L. Lesser has no financial or proprietary interest in the materials presented herein. Dr. Leah Levi has no financial or proprietary interest in the materials presented herein. Dr. Grant T. Liu has no financial or proprietary interest in the materials presented herein. Dr. Amina Malik has no financial or proprietary interest in the materials presented herein. Dr. Timothy James McCulley has no financial or proprietary interest in the materials presented herein. Dr. Neil R. Miller has no financial or proprietary interest in the materials presented herein. Dr. Lina Nagia has no financial or proprietary interest in the materials presented herein. Dr. Nancy J. Newman is a consultant for Santhera, Gensight, and Trius. Dr. Steve Newman has no financial or proprietary interest in the materials presented herein. Dr. Victoria S. Pelak receives royalties from Up-to-Date, Inc.

216  Financial Disclosures Dr. Janet C. Rucker has no financial or proprietary interest in the materials presented herein. Dr. Alfredo A. Sadun is the principle investigator of a lab that receives grant support for research in LHON from Edison Pharmaceuticals and Stealth Peptides. He has no equity in either, receives no consulting payments, nor is he personally compensated. Dr. Robert H. Spector has no financial or proprietary interest in the materials presented herein. Dr. Madhura A. Tamhankar has no financial or proprietary interest in the materials presented herein. Dr. Rosa Ana Tang is part of the speaker's bureau of EMD Serono and Genzyme. Dr. Matthew J. Thurtell has no financial or proprietary interest in the materials presented herein. Dr. Robert L. Tomsak has no financial or proprietary interest in the materials presented herein. Dr. Roger E. Turbin has no financial or proprietary interest in the materials presented herein. Dr. Michael S. Vaphiades has performed consulting work for Mallinckrodt Pharmaceuticals. Dr. Nicholas Volpe has no financial or proprietary interest in the materials presented herein. Dr. Michael Wall has no financial or proprietary interest in the materials presented herein. Dr. Michelle Y. Wang has no financial or proprietary interest in the materials presented herein. Dr. Sushma Yalamanchili has no financial or proprietary interest in the materials presented herein.